You are on page 1of 46

VISIONIAS

www.visionias.in
ANSWERS & EXPLANATIONS
GENERAL STUDIES (P) TEST – 2982 (2020)

Q 1.A
• भारतीय चीनी उद्योग भारत का दूसरा सबसे बडा कृ षि-आधाररत उद्योग है। इस उद्योग हेतु मूल कच्चा माल गन्ना है, षिसकी
कु छ षिषिष्ट षििेिताएं हैं:
o यह भार ह्रास िाला कच्चा माल है। इसे लंबे समय तक भंडाररत नहीं ककया िा सकता है क्योंकक समय के साथ इसमें सुक्रोि
की मात्रा कम हो िाती है। इसे लंबी दूरी तक पररिहषनत नहीं ककया िा सकता है। इसीषलए चीनी षमलें गन्ना उत्पादन
क्षेत्रों में ही षथथत होती हैं। इसषलए कथन 1 सही है।
• भारत गन्ना और चीनी, दोनों का षिश्व में दूसरा सबसे बडा उत्पादक राष्ट्र है तथा चीनी के कु ल िैषश्वक उत्पादन में से इसका
योगदान 8 प्रषतित है। ििष 2015 में भारत में गन्ने का िार्िषक उत्पादन 341,400 हिार मीरिक टन (TMT) अनुमाषनत था।
ब्रािील गन्ना उत्पादन की सूची में प्रथम थथान पर है, यह 739,300 हिार मीरिक टन का िार्िषक उत्पादन करता है। इसके
अषतररक्त गन्ने से खांडसारी और गुड भी बनाए िाते हैं। इसषलए कथन 2 सही नहीं है।

Q 2.A
• ओरांग राष्ट्रीय उद्यान
o ओरांग राष्ट्रीय उद्यान या रािीि गांधी ओरांग राष्ट्रीय उद्यान, षिसे षमनी कािीरंगा राष्ट्रीय उद्यान भी कहा िाता है, असम
के दरांग और सोषनतपुर षिलों में कािीरं गा राष्ट्रीय उद्यान से लगभग 100 ककमी की दूरी पर षथथत है।
o ओरांग राष्ट्रीय उद्यान 78.81 िगष ककमी के क्षेत्र में षिथतृत है। यह ब्रह्मपुत्र नदी के उत्तरी ककनारे पर षथथत है। इसषलए युग्म
1 सही सुमषे लत है।
o उद्यान का संपूर्ष क्षेत्र िल षनकाय, पूिी षहमालय के आर्द्ष पर्षपाती िन, पूिी आर्द्ष िलोढ़ घास के मैदान, पूिी दलदली िन,
सिाना घासभूषम, षनम्नीकृ त घासभूषम, आर्द्ष रे तीले क्षेत्र और िुष्क रे तीले क्षेत्र में षिभाषित है।
• पन्ना राष्ट्रीय उद्यान
o पन्ना राष्ट्रीय उद्यान मध्यप्रदेि के पन्ना और छतरपुर षिलों में षथथत है। इसे ििष 1981 में राष्ट्रीय उद्यान घोषित ककया
गया। उद्यान को षिश्व भर में इसके िन्यिीिों के षलए िाना िाता है, षिनमें बाघ, षहरर्, बारहससंगा, षगद्ध, भेषडए,
सचंकारा और अनेक अन्य िन्यिीि सषममषलत हैं।
o इस आरषक्षत िन क्षेत्र से होकर के न नदी प्रिाषहत होती है तथा नदी घाटी के अपने मागष में आकिषक िलप्रपातों का षनमाषर्
करती है। इस राष्ट्रीय उद्यान की िैि-षिषिधता अत्यंत समृद्ध है। इसषलए युग्म 2 सही सुमषे लत है।
o इस क्षेत्र में पाई िाने िाली िनथपषत षमषित प्रकार की होती है, षिसमें असमतल क्षेत्र की कं टीली झाषडयों और घास से
लेकर चट्टानी भू-पररदृश्य के सैकडों प्रिाषतयों के िृक्ष और झाषडयााँ िाषमल हैं।
• षिम कॉबेट राष्ट्रीय उद्यान
o षिम कॉबेट राष्ट्रीय उद्यान भारत का सबसे पुराना राष्ट्रीय उद्यान है और इसकी थथापना ििष 1936 में हेली राष्ट्रीय उद्यान
के रूप में संकटग्रथत बंगाल टाइगर के संरक्षर् के षलए की गई थी।
o यह उत्तराखंड के नैनीताल और पौडी गढ़िाल षिलों में अिषथथत है और इसका नाम प्रषसद्ध षिकारी और प्रकृ षतषिद षिम
कॉबेट के नाम पर रखा गया है। यह उद्यान प्रोिेक्ट टाइगर पहल के अंतगषत िाषमल पहला उद्यान है।

1 www.visionias.in ©Vision IAS


o षिम कॉबेट राष्ट्रीय उद्यान में पाए िाने िाले अन्य थतनधारी िीिों में षहरर् की प्रिाषतयां (काकड या बार्किं ग षडयर,
सांभर, पाढ़ा या हॉग षडयर और चीतल), थलॉथ षबयर, षहमालयी काले भालू, भारतीय धूसर नेिले (Indian Grey
Mongoose), ऊदषबलाि, हाथी, पीले गदषन िाले नेिले (Yellow Throated Martens), घोरल (बकरी-मृग िैसा िंत)ु ,
भारतीय पैंगोषलन तथा लंगरू और रीसस बंदर िाषमल हैं।
o रामगंगा नदी कॉबेट राष्ट्रीय उद्यान की िीिनरे खा है। यह उद्यान के सभी िल षनकायों में सबसे बडी और महत्िपूर्ष है।
इसषलए युग्म 3 सही सुमषे लत है।
• िलदापाडा राष्ट्रीय उद्यान
o िलदापाडा राष्ट्रीय उद्यान पषिम बंगाल के अलीपुरद्वार षिले में षथथत है। िलदापाडा राष्ट्रीय उद्यान को िलदापाडा
िन्यिीि अभयारण्य भी कहा िाता है।
o यहााँ षिषभन्न प्रकार के पादप और िंतु पाए िाते हैं। इस उद्यान में पाई िाने िाली कु छ प्रमुख पादप प्रिाषतयााँ हैं- लंबे साल
(सखुआ/साखू) और िीिम के िृक्ष। अन्य प्रिाषतयों में फनष, झाषडयााँ और लंबी घास भी िाषमल हैं।
o िलदापाडा का मुख्य आकिषर् एक सींग िाला भारतीय गैंडा है। यह राष्ट्रीय उद्यान एक सींग िाले भारतीय गैंडे, हाषथयों,
भारतीय बाइसन, पाढ़ा, चीतल, सांभर, काकड, िंगली सुअर, बाघ, तेंदआ
ु , िंगली षबषललयों, लेपडष कै ट, कफसिंग कै ट,
भारतीय सीिेट, भारतीय षििाल षगलहरी, भारतीय पैंगोषलन, षहषथपड हेयर (एक प्रकार का खरगोि), भारतीय साही
आकद का आिास थथल है।
o उद्यान पूिी षहमालय की तराई और टोसाष नदी के तट पर षथथत है। इसषलए युग्म 4 सही सुमषे लत नहीं है।
• 358 ककमी लंबी टोसाष नदी षतब्बत की चुंबी घाटी से षनकलती है, िहााँ इसे माचु (Machu) कहा िाता है। इसके पिात् पषिम
बंगाल में प्रिेि करने से पूिष यह भूटान से प्रिाषहत होती है, िहााँ इसे एमो चु (Amo Chu) कहा िाता है। यह नदी चीन में
लगभग 113 ककमी और भूटान में 145 ककमी की दूरी तक प्रिाषहत होती है।
• पषिम बंगाल में, टोसाष चाय उत्पादक दलससंगपाडा (िलपाईगुडी) और िलदापाडा राष्ट्रीय उद्यान (अलीपुरद्वार षिला) से
प्रिाषहत होती है। इसके पिात् यह बांग्लादेि में प्रिेि करती है, िहााँ इसे कलिनी नाम से िाना िाता है और बांग्लादेि में यह
ब्रह्मपुत्र नदी में षमल िाती है।

Q 3.C
• कथन 1 सही है: थथानांतरी कृ षि (झूम कृ षि) एक कृ षि प्रर्ाली है, षिसमें भूषम के ककसी भाग पर कु छ समय तक खेती की िाती
है, कफर मृदा की उिषरता कम होने पर उसे परती छोड कदया िाता है और िहााँ प्राकृ षतक िनथपषतयों को उगने कदया िाता है ,
उसकी िगह कृ िक अन्य ककसी भूखंड पर खेती करता है।
• कथन 2 सही है: भूमध्यरे खीय क्षेत्रों की िनसंख्या अत्यंत कम होती है तथा इस क्षेत्र के िनों में, आकदम लोग/िनिाषतयां
आमतौर पर षिकारी और भोिन संग्राहक होते हैं। इन उष्र्करटबंधीय क्षेत्रों, षििेिकर अफ्रीका, दषक्षर् और मध्य अमेररका
तथा दषक्षर्-पूिष एषिया में षनिास करने िाली षिषभन्न िनिाषतयों में आकदम/पुरातन िैली की कृ षि या थथानांतरी (झूम) कृ षि
अषधक प्रचषलत है। यह उष्र्करटबंधीय क्षेत्र में षिषभन्न नामों से प्रचषलत है, उदाहरर्ाथष भारत के पूिोत्तर राज्यों में झूम कृ षि,
मध्य अमेररका और मेषक्सको में षमलपा, इं डोनेषिया और मलेषिया में लदांग।
Q 4.D
• भारतीय िलक्षेत्र में समुर्द्ी थतनधारी िीिों की पच्चीस प्रिाषतयााँ पाई िाती हैं, िो दो कु लों षसटेषिया (Cetacea) और
साइरे षनया (Sirenia) से समबंषधत हैं। सी काऊ (डु गोंग डु गोन) मन्नार की खाडी, कच्छ की खाडी और अंडमान षनकोबार के
तटीय िल में पाई िाती है। साइरे षनया कु ल की एक प्रिाषत डु गोंग और षसटेषिया की 30-34 प्रिाषतयााँ (डॉषलफन, व्हेलस और
सील, पॉरपॉइज़), षिनमें गंगा नदी डॉषलफन भी िाषमल है, भारतीय उपमहाद्वीप के िल में पाई िाती हैं। भारतीय तटों पर
पाए िाने िाले समुर्द्ी थतनधाररयों की सभी प्रिाषतयों को भारतीय (िन्यिीि) संरक्षर् अषधषनयम (1972) के अंतगषत संरक्षर्
प्राप्त है।
• पॉरपॉइज़ पूर्त
ष ः िलीय समुर्द्ी थतनधाररयों का समूह है, षिसमें से सभी फॉकोषनडी पररिार के अंतगषत िगीकृ त ककए गए हैं।
पोरपॉइज़ की सात प्रिाषतयााँ षिद्यमान हैं। िे छोटे दााँतों िाले व्हेल (थमाल टूथेड व्हेल) हैं, िो महासागरीय डॉषलफन से

2 www.visionias.in ©Vision IAS


https://t.me/UPSC_PDF Download From > https://upscpdf.com https://t.me/UPSC_PDF

षनकटता से संबंषधत हैं। पॉरपॉइज़ के दााँत फािडे सदृि (सपाट) िबकक डॉषलफन के दााँत नुकीले होते हैं। डॉषलफन में हुक के
आकार का या मुडा हुआ पृष्ठीय पंख (dorsal fin) होता है (उन प्रिाषतयों को छोडकर षिनमें पृष्ठीय पंख नहीं होता है), िबकक
पॉरपॉइज़ का पृष्ठीय पंख अषधक षत्रकोर्ीय होता है। साथ ही सामान्यतः डॉषलफन का िरीर अपेक्षाकृ त अषधक पतला होता है
िबकक पॉरपॉइज़ थोडे थथूल होते हैं। डॉषलफन इंफ्राऑडषर षसटेषिया (Cetacea) के अंतगषत आने िाले िलीय थतनधाररयों का
सामान्य नाम है।
• डु गोंग मध्यम आकार का समुर्द्ी थतनधारी िीि है। यह साइरे षनया कु ल की चार िीषित प्रिाषतयों में से एक है, षिसमें मैनाटी
की तीन प्रिाषतयााँ भी िाषमल हैं। यह कभी षिषिधतापूर्ष रहे डु गोंषगडाए (Dugongidae) पररिार का एकमात्र िीषित
सदथय है; इसका सबसे षनकटतम आधुषनक संबंषधत िीि ‘थटेलसष सी काऊ’ 18िीं िताब्दी में अषधक षिकार के कारर् लुप्त हो
गया।

Q 5.D
• कथन 1 सही नहीं है: मानि संसाधन षिकास मंत्रालय ने उच्च षिक्षा में बेहतर अषधगम पररर्ामों के षलए प्रौद्योषगकी का
उपयोग करने हेतु एक नई PPP योिना नेिनल एिुकेिनल एलायन्स फॉर टेक्नोलॉिी (NEAT) की घोिर्ा की। अनेक थटाटष-
अप कं पषनयां ऐसी प्रौद्योषगककयों पर पहले से कायष कर रही हैं। अतः MHRD ऐसे प्रयासों की पहचान करेगी और उन्हें एक
साझे मंच पर लेकर आएगी, ताकक षिक्षार्थषयों को इस तक सुगमता से पहुाँच प्राप्त हो सके । युिाओं को षिषक्षत करना एक राष्ट्रीय
प्रयास है तथा MHRD, PPP मॉडल के िररए ऐसी प्रौद्योषगकी षिकषसत करने िाली िैक्षषर्क तकनीकी (EdTech) कं पषनयों
के साथ राष्ट्रीय गठबंधन प्रथताषित करती है।
• कथन 2 सही है: इसका उद्देश्य षिक्षाथी की आिश्यकता के अनुरूप अषधगम की प्रकक्रया को अषधक िैयषक्तकृ त
(personalised) और अनुकूषलत बनाने के षलए आर्टषकफषियल इं टेषलिेंस (कृ षत्रम बुषद्धमत्ता) का उपयोग करना है। इसके षलए
षिक्षार्थषयों की षिषिधता को संबोषधत करने हेतु अनुकूली अषधगम (Adaptive Learning) में तकनीकों के षिकास की
आिश्यकता है।
• कथन 3 सही नहीं है। NEAT कायषक्रम का कायाषन्ियन करने िाली एिेंसी AICTE होगी। योिना को MHRD द्वारा गरठत
सिोच्च सषमषत के मागषदिषन में प्रिाषसत ककया िाएगा।
• नोट: राष्ट्रीय ज्ञान नेटिकष (National Knowledge Network: NKN) पररयोिना का लक्ष्य एक िृहत और सुदढ़ृ भारतीय
नेटिकष थथाषपत करना है, िो सुरषक्षत और षिश्वसनीय कनेषक्टषिटी प्रदान करने में सक्षम होगा। NKN एक अत्याधुषनक, मलटी-
गीगाषबट पैन-इं षडयन ररसोसष िेयररं ग नेटिकष है, षिसका लक्ष्य सभी राष्ट्रीय षिश्वषिद्यालयों, महाषिद्यालयों और िोध संथथानों
को देिव्यापी िचुअ
ष ल क्लासरूम के संचालन के षलए षडषिटल रूप से कनेक्ट करना है।

Q 6.A
• हाल ही में िैज्ञाषनक तथा औद्योषगक अनुसंधान पररिद-राष्ट्रीय भूभौषतकीय अनुसंधान संथथान (CSIR-NGRI) द्वारा
पयाषिरर्ीय क्षषत का आकलन करने हेतु नीषत षनमाषताओं के उपयोग के षलए पहली बार ‘षियोके षमकल बेसलाइन एटलस ऑफ
इं षडया’ िारी ककया गया है। इसषलए कथन 2 सही नहीं है।
• इस एटलस में संपर्
ू ष भारत में ऊपरी और षनचली मृदा परतों में षिद्यमान धातुओं, ऑक्साइडों और तत्िों के 45 मानषचत्र
समाषिष्ट ककए गए हैं। उललेखनीय है कक यह एटलस पृथ्िी की सतह में होने िाले रासायषनक पररितषनों का आकलन करने हेतु
देि की भािी पीढ़ी के षलए एक संदभष के रूप में कायष करे गा। ये मानषचत्र प्रदूिर् उत्पन्न करने िाले उद्योगों या अन्य षनकायों से
भषिष्य में होने िाले संदि
ू र् को ज्ञात करने में सहायता करें गे। इसषलए कथन 1 सही है।
• यह मृदा में षििाक्त रसायनों की षिद्यमानता और षिषभन्न थथानों पर उनकी सांर्द्ता की मात्रा में षभन्नता को ज्ञात करने में
सहायता करे गा। यह इस तथ्य की पहचान करने में भी सहायक षसद्ध होगा कक मृदा प्रदूिर् भूिल संदि ू र् में ककस प्रकार
योगदान करता है।
• ये मानषचत्र प्रत्येक पौधे की िृषद्ध के षलए आिश्यक सूक्ष्म पोिक तत्िों से समृद्ध उपयुक्त मृदा का चयन करने में भी सहायक
होंगे।

3 www.visionias.in ©Vision IAS

Google it:- https://upscpdf.com


https://t.me/UPSC_PDF Download From > https://upscpdf.com https://t.me/UPSC_PDF

• इससे पूिष, प्रदूिर्कताष द्वारा पयाषिरर् को पहुंचाई गई क्षषत को अथिीकार ककए िाने की षथथषत में उसका उत्तरदाषयत्ि
प्रमाषर्त करने की कोई षिषध उपलब्ध नहीं थी। परन्तु ितषमान में बेसलाइन मैप्स एटलस इसे प्रमाषर्त करने में सहायता
करे गा।
• CSIR-NGRI: राष्ट्रीय भूभौषतकीय अनुसंधान संथथान (NGRI) एक भूिैज्ञाषनक िोध संगठन है, षिसकी थथापना ििष 1961
में िैज्ञाषनक तथा औद्योषगक अनुसंधान पररिद (CSIR) के अधीन की गई थी।

Q 7.D
• एयरोसोल िायुमंडल में षनलंषबत सूक्ष्म कर् होते हैं। िब ये कर् पयाषप्त रूप से िृहत् आकार ग्रहर् कर लेते हैं तो उनकी थपष्ट
उपषथथषत दृषष्टगोचर होने लगती है, क्योंकक िे सूयष के प्रकाि का प्रकीर्षन और अििोिर् करते हैं। उनके द्वारा सूयष के प्रकाि का
प्रकीर्षन ककए िाने से दृश्यता कम हो सकती है (धुध
ं की षथथषत में) और सूयोदय एिं सूयाषथत के समय आकाि में लाल िर्ष की
आभा दृषष्टगोचर होती है।
• एयरोसोल पृथ्िी के षिककरर् बिट तथा िलिायु को प्रत्यक्ष और अप्रत्यक्ष रूप से प्रभाषित करते हैं। प्रत्यक्ष प्रभाि के रूप में,
एयरोसोल सूयष के प्रकाि का पुनः अंतररक्ष की ओर प्रकीर्षन कर देते हैं। अप्रत्यक्ष प्रभाि के रूप में, षनचले िायुमड
ं ल में
एयरोसोल मेघों के कर्ों के आकार को पररिर्तषत कर सकते हैं। इस प्रकार मेघों द्वारा सूयष के प्रकाि को परािर्तषत और
अििोषित करने की प्रकक्रया पररिर्तषत हो िाती है, षिससे पृथ्िी का ऊिाष बिट प्रभाषित होता है।
• एयरोसोल रासायषनक अषभकक्रयाओं (षििमांगी रासायषनकी) के षलए आधार थथलों के रूप में भी कायष कर सकते हैं। इन
अषभकक्रयाओं में से कु छ समतापमंडल में ओिोन परत के क्षरर् के षलए उत्तरदायी हैं। ध्रुिीय क्षेत्रों में िीतकाल के दौरान
एयरोसोल षिकषसत होकर ध्रुिीय समतापमंडलीय मेघों का षनमाषर् करते हैं। इन मेघ कर्ों के षिथतृत सतह क्षेत्र रासायषनक
अषभकक्रयाओं के षलए आधार थथल प्रदान करते हैं। इन प्रषतकक्रयाओं से िृहत् मात्रा में अषभकक्रयािील क्लोरीन का षनमाषर् और
पररर्ामथिरूप अंतत: समतापमंडल में ओिोन परत का क्षरर् होता है।
• इसषलए सभी कथन सही हैं।

Q 8.C
• कथन 1 सही नहीं है: धूम-कोहरा (smog) दो िब्दों- धूम और कोहरे से व्युत्पन्न हुआ है। धूम-कोहरे के दो प्रकार होते हैं-
सामान्य धूम कोहरा और प्रकाि-रासायषनक धूम कोहरा। सामान्य धूम-कोहरा िीत नम िलिायु में षनर्मषत होता है। यह धुएं,
कोहरे और सलफर डाई ऑक्साइड का षमिर् है। रासायषनक रूप से यह एक अपचायक षमिर् होता है और इसषलए इसे
‘अपचायक धूम-कोहरा’ भी कहा िाता है।
• कथन 2 सही नहीं है और कथन 3 सही है: प्रकाि-रासायषनक धूम-कोहरा उष्र्, िुष्क और साफ धूपमयी िलिायु में षनर्मषत
होता है। प्रकाि-रासायषनक धूम-कोहरा िाहनों और कारखानों से उत्सर्िषत नाइिोिन के ऑक्साइडों और हाइड्रोकाबषनों पर
सूयष के प्रकाि की अषभकक्रया के कारर् उत्पन्न होता है। चूाँकक इसमें ऑक्सीकारक अषभकमषकों की सांर्द्ता उच्च होती है, अतः इसे
ऑक्सीकारक धूम-कोहरा भी कहा िाता है।

Q 9.C
• के रल के कयाल या पििल, अरब सागर तट (मालाबार तट) के समानांतर षिद्यमान लिर्ीय लैगन
ू ों और झीलों के साथ-साथ
अंतयोषित नहरों, नकदयों और इनलेट्स का नेटिकष है। इसषलए कथन (c) सही उत्तर है।
• उनका उपयोग मत्थयन, अंतःथथलीय नौकायन के षलए ककया िाता है और साथ ही ये पयषटकों के षलए आकिषर् का कें र्द् भी हैं।
प्रषत ििष के रल के पुन्नामदा कयाल में प्रषसद्ध नेहरू िॉफी िलामकाली (बोट रे स) का आयोिन ककया िाता है। के रल में पििल
(बैकिाटसष) के रल के एक छोर से दूसरे छोर तक षिथताररत 1500 ककमी लंबी नहरों (मानि षनर्मषत और प्राकृ षतक), 38 नकदयों
और 5 बडी झीलों का एक नेटिकष है। कयाल या पििल के रल के प्रमुख पयषटन थथलों में से एक हैं और ये के रल राज्य की
षिषिष्ट षििेिता हैं। इन्हें परं परागत रूप से मुख्य पररिहन मागष के रूप में उपयोग ककया िाता रहा है। पििल के रल आने िाले
पयषटकों के षलए आकिषक अनुभि प्रदान करते हैं।

4 www.visionias.in ©Vision IAS

Google it:- https://upscpdf.com


https://t.me/UPSC_PDF Download From > https://upscpdf.com https://t.me/UPSC_PDF

Q 10.B
• हाल ही में ईथट इं षडया कं पनी पर रानी चेन्नममा की षििय के थमरर् में ककत्तूर उत्सि का आयोिन ककया गया था। इस महोत्सि
में खेलकू द, सांथकृ षतक कायषक्रम ि रानी चेन्नममा के राज्य के षििय में िानकार व्यषक्तयों के व्याख्यानों का आयोिन ककया गया
था।
• ककत्तूर की रानी चेन्नममा एक िीर योद्धा थीं, षिन्होंने 19िीं िताब्दी के आरंभ में षब्ररटि सेनाओं के षिरुद्ध षिर्द्ोह का नेतृत्ि
ककया। िह 1824 ई. में षब्ररटि ईथट इं षडया कं पनी के षिरुद्ध सिस्त्र षिर्द्ोह का नेतृत्ि करने िाली प्रथम भारतीय िाषसका थीं।
यद्यषप इस षिर्द्ोह के दौरान उनकी सेना की संख्या अंग्रि
े ी सेना की तुलना में कम होने के कारर् अंततः उन्हें बंदी बना षलया
गया था, तथाषप भारत में षब्ररटि िासन के षिरुद्ध षिर्द्ोह का नेतृत्ि करने के षलए उनका थमरर् ककया िाता है।
• उनका षििाह 15 ििष की आयु में ककत्तूर के रािा मललसिष देसाई से हुआ और िह ककत्तूर की रानी बनीं। 1816 ई. में रािा
मललसिष देसाई की और 1824 ई. में उनके पुत्र की मृत्यु हो गई। ककत्तूर की रानी के रूप में चेन्नममा ने अपने पुत्र की मृत्यु के
पिात् षििसलंगप्पा को ककत्तूर के ससंहासन का उत्तराषधकारी बनाने के उद्देश्य से गोद षलया। षब्ररटि ईथट इं षडया कं पनी ने
चेन्नममा की इस कारषिाई को थिीकार नहीं ककया और व्यपगत नीषत के आधार पर राज्य से षििसलंगप्पा के षनिाषसन का आदेि
कदया।
• रानी चेन्नममा ने षििसलंगप्पा को रािससंहासन से िंषचत करने के षब्ररटि आदेि को चुनौती दी षिसके कारर् युद्ध प्रारं भ हो
गया। इसषलए कथन 1 सही नहीं है।
• थितंत्रता आंदोलन के दौरान, षब्ररटि सेनाओं के षिरुद्ध उनका साहषसक प्रषतरोध कई प्रेरर्ादायक नाटकों, लोक गीतों (लािर्ी)
और कहाषनयों का षििय बन गया। षब्ररटि सेनाओं के षिरुद्ध रानी चेन्नममा की प्रथम षििय को प्रषत ििष अक्टूबर में ककत्तूर
उत्सि के रूप में आयोषित ककया िाता है। इसषलए कथन 2 सही है।

Q 11.C
• हाल ही में भारत सरकार ने षहमालय के िल-सोतों या चश्मों (Springs) का पुनरुद्धार करने के षलए िल अभयारण्य कायषक्रम
आरंभ करने का षनर्षय ककया है। इस प्रयोिनाथष आठ षहमालयी राज्यों में िल-सोतों की पहचान की गई है। इस कायषक्रम की
षनम्नषलषखत षििेिताएं हैं:
o सूखते िा रहे िल-सोतों की पहचान करने और उनके षििय में सूचनाओं के संकलन के साथ लगभग 1 लाख िल-सोतों की
षिथतृत सूची तैयार करना।
o प्रथम ििष में सिाषषधक सुभेद्य षिलों को लषक्षत करते हुए आगामी पांच ििों तक षचषननत िल-सोतों का पुनरुद्धार करना।
o भारतीय षहमालयी क्षेत्र (IHR) के सभी 12 राज्यों में ग्राम पंचायतों, राज्य सरकारों षििेिकर राज्य िन षिभागों के
सहयोग से कायाषन्ियन।
• इसषलए कथन (c) सही उत्तर है।

Q 12.D
• ज्ञातव्य है कक िायुमंडलीय दाब में षभन्नता के कारर् िायु गषतमान होती है। इस क्षैषति गषतमान िायु को पिन कहा िाता है।
पिन उच्च दाब से षनम्न दाब िाले क्षेत्र की ओर प्रिाषहत होती है। भूतल पर धरातलीय षििमताओं के कारर् घिषर् उत्पन्न होता
है, षिससे पिन की गषत प्रभाषित होती है। इसके अषतररक्त, पृथ्िी का घूर्षन भी पिन की गषत को प्रभाषित करता है।
• पृथ्िी के घूर्न
ष द्वारा व्युत्पन्न बल को कोररऑषलस बल के रूप में िाना िाता है। इस प्रकार पृथ्िी के धरातल पर क्षैषति पिनें
तीन बलों के संयुक्त प्रभाि के प्रषत अनुकक्रया करती हैं, ये हैं- दाब प्रिर्ता बल, घिषर् बल और कोररऑषलस बल। इसके
अषतररक्त गुरुत्ि बल पिनों को िायुमंडल से धरातल की ओर अितषलत करता है।
• दाब प्रिर्ता बल: िायुमंडलीय दाब में षभन्नता से एक बल उत्पन्न होता है। दूरी के साथ दाब में पररितषन की दर दाब प्रिर्ता
कहलाती है। िहां समदाब रे खाएं एक-दूसरे के षनकट होती हैं िहां दाब प्रिर्ता अषधक और िहां समदाब रे खाएं एक-दूसरे से
दूर होती हैं िहां दाब प्रिर्ता कम होती है।
• घिषर् बल: यह पिन की गषत को प्रभाषित करता है। धरातल पर घिषर् सिाषषधक होता है और इसका प्रभाि धरातल से
सामान्यतया 1-3 ककमी की ऊंचाई तक षिथताररत होता है। समुर्द् की सतह पर घिषर् न्यूनतम होता है।
• कोररऑषलस बल: पृथ्िी का अपने अक्ष पर घूर्न
ष पिन की कदिा को प्रभाषित करता है। 1844 ई. में इसका षििरर् प्रथतुत
करने िाले एक फ्रांसीसी भौषतक िैज्ञाषनक के नाम पर इस बल को कोररऑषलस बल कहा िाता है। इस बल के प्रभाि से पिन

5 www.visionias.in ©Vision IAS

Google it:- https://upscpdf.com


https://t.me/UPSC_PDF Download From > https://upscpdf.com https://t.me/UPSC_PDF

उत्तरी गोलाद्धष में अपनी मूल कदिा से दाषहनी ओर तथा दषक्षर्ी गोलाद्धष में बाईं ओर षिक्षेषपत हो िाती है। पिन का िेग
अषधक होने पर षिक्षेपर् भी अषधक होता है।
o कोररऑषलस बल अक्षांि के कोर् के अनुक्रमानुपाती होता है। यह ध्रुिों के षनकट अषधकतम और षििुित िृत्त पर
अनुपषथथत होता है।
o कोररऑषलस बल, दाब प्रिर्ता बल के समकोर् पर कायष करता है। दाब प्रिर्ता बल समदाब रेखा के लंबित (समकोर्)
होता है। दाब प्रिर्ता षितनी अषधक होगी, पिन का िेग भी उतना ही अषधक होगा तथा पिन की कदिा भी उतनी ही
अषधक षिक्षेषपत होगी।
o इन दोनों बलों के एक दूसरे से समकोर् पर होने के कारर् षनम्न दाब िाले क्षेत्रों में पिन इसके चतुर्दषक प्रिाषहत होती है।
o षििुित िृत्त पर कोररऑषलस बल िून्य होता है और पिन समदाब रे खाओं के समकोर् पर प्रिाषहत होती है। अत: षनम्न
दाब क्षेत्र और अषधक गहन होने की बिाय पूररत हो िाता है। यही कारर् है कक षििुित िृत्त पर उष्र्करटबंधीय चक्रिात
षनर्मषत नहीं होते हैं।
Q 13.A
• उइग़र लोगों को चीन िनिादी गर्राज्य के षिनषियांग उइग़र थिायत्त क्षेत्र के मूल षनिाषसयों के रूप में पहचाना िाता है।
उन्हें चीन के आषधकाररक रूप से मान्यता प्राप्त 55 नृिातीय अलपसंख्यक समूहों में से एक माना िाता है। चीन द्वारा उइग़र
लोगों को के िल एक बहु-सांथकृ षतक राष्ट्र में एक क्षेत्रीय अलपसंख्यक के रूप में मान्यता प्रदान की गई है और चीन उनके थिदेिी
समूह होने के षिचार को मान्यता प्रदान नहीं करता है। इसषलए युग्म 1 सही सुमषे लत नहीं है।
• रोसहंग्या राज्यषिहीन षहन्द-आयष नृिातीय समूह हैं। ये मुख्य रूप से इथलाम धमष के अनुयायी हैं और उनमें से कु छ अलपसंख्यक
सहंदू धमष का भी अनुसरर् करते है तथा ये मयांमार के रखाइन प्रांत में षनिास करते हैं। ििष 2016-17 के संकट से पूिष मयांमार में
अनुमाषनत रूप से 1 षमषलयन रोसहंग्या षनिास कर रहे थे। संयक्त
ु राष्ट्र द्वारा ििष 2013 में षिश्व के सिाषषधक अत्याचार ग्रथत
अलपसंख्यकों के रूप में िर्र्षत ककए गए रोसहंग्या समुदाय को ििष 1982 के मयांमार राष्ट्रीयता कानून के अंतगषत नागररकता
प्रदान नहीं की गई है। इसषलए युग्म 2 सही सुमषे लत है।
• हािोंग भारतीय उपमहाद्वीप षििेितया पूिोत्तर भारतीय राज्यों और बांग्लादेि का देिि िनिातीय समुदाय है। हािोंग की
बहुसंख्यक आबादी भारत में षनिाषसत है। हािोंग मुख्य रूप से धान की कृ षि करने िाले ककसान हैं। हािोंग को भारत में
अनुसूषचत िनिाषत का दिाष प्राप्त है। भारत में हािोंग समुदाय के लोग मेघालय की गारो एिं खासी दोनों पहाषडयों पर षनिास
करते हैं। इनमें से अषधकांि मेघालय और बांग्लादेि सीमा से सटे साउथ-िेथट गारो षहलस षिले में रहते हैं। इसषलए युग्म 3 सही
सुमषे लत नहीं है।
Q 14.D
• षमलिाउकी डीप, षिसे षमलिाउकी डेप्थ के नाम से भी िाना िाता है, अटलांरटक महासागर का सबसे गहरा क्षेत्र और प्यूटो
ररको िेंच (गतष) का एक भाग है। इसषलए युग्म 1 सही सुमषे लत है।
• सुंडा गतष, षिसे िािा गतष के नाम से भी िाना िाता है, सुमात्रा के षनकट सहंद महासागर में अिषथथत एक महासागरीय गतष है।
यह उस थथान पर षनर्मषत है िहां ऑथिेषलयाई-कै प्रीकॉनष प्लेटें यूरेषियन प्लेट के एक भाग के नीचे प्रषिष्ठ (subduct) होती हैं।
इसषलए युग्म 2 सही सुमषे लत है।
• चैलि
ें र डीप पृथ्िी के समुर्द् अधथतल िलमंडल में ज्ञात सबसे गहरा क्षेत्र है। यह प्रिांत महासागर में मररयाना द्वीप समूह के
षनकट मररयाना गतष के दषक्षर्ी छोर पर षथथत है। इसषलए युग्म 3 सही सुमषे लत है।

Q 15.D
• सभी बषहिषषनक भूआकृ षतक प्रकक्रयाओं को एक सामान्य पद ‘अनाच्छादन’ (Denudation) के अंतगषत समाषिष्ट ककया िाता है।
'अनाच्छादन' िब्द का अथष है- षनरािृत्त करना या आिरर् हटाना। अपक्षय, िृहत् क्षरर्, संचलन, अपरदन, पररिहन आकद
'अनाच्छादन' प्रककया में सषममषलत होते हैं।
• अपक्षय (Weathering): अपक्षय चट्टानों के लघु खंडों में अपघटन की प्रकक्रया है। यह अपघटन तीन प्रकार की अपक्षय
प्रकक्रयाओं- भौषतक, रासायषनक और िैषिक अपक्षय द्वारा होता है।
• चट्टानों का अपघटन और खंडों में षिखंडन भौषतक अपक्षय द्वारा संपन्न होता है, रासायषनक अपक्षय में यह अपघटन चट्टान की
संरचना में रासायषनक पररितषन के कारर् होता है तथा िैषिक अपक्षय में िीि एिं पादप चट्टानों के अपघटन के षलए
उत्तरदायी होते हैं।

6 www.visionias.in ©Vision IAS

Google it:- https://upscpdf.com


https://t.me/UPSC_PDF Download From > https://upscpdf.com https://t.me/UPSC_PDF

• अपरदन (Erosion): नकदयों और षहमनदों िैसे प्राकृ षतक अषभकारकों के कारर् चट्टानों एिं अपक्षषयत सामग्री का संचलन
‘अपरदन’ कहलाता है।
• पररिहन (Transportation): अपरदन के कारर् उत्पन्न मलबे को िल, िायु आकद कारकों द्वारा एक थथान से दूसरे थथान पर
पहुंचाया िाना ‘पररिहन’ होता है।
• षनक्षेपर् (Deposition): यह पृथ्िी के कु छ भागों में मलबे का षनक्षेषपत हो िाना है, िहां यह निीन चट्टानों का षनमाषर् करने
के षलए संषचत हो सकता है।
• उललेखनीय है कक उपयुक्त
ष सभी अनाच्छादन प्रकक्रयाएं षिश्व के षिषभन्न भागों में षभन्न-षभन्न दरों पर एक साथ संपन्न होती हैं
तथा काफी हद तक उच्चािच की प्रकृ षत, चट्टान की संरचना, थथानीय िलिायु और मनुष्य के हथतक्षेप पर षनभषर करती हैं।
Q 16.C
• हाल ही में िन षिभाग एिं कु ड्डलोर मरीन पुषलस के अषधकाररयों को तट पर 5.88 ककलोग्राम का ‘एमबरषग्रस’ (कभी-कभी इसे
व्हेल की उलटी भी कहते हैं) नामक पदाथष प्राप्त हुआ था, िो प्रिाषहत होते हुए तट पर आ गया था। थपमष व्हेल
(Physetercatodon) की आंत से उत्पन्न होने िाला एमबरषग्रस एक ठोस मोमीय पदाथष है। पूिी संथकृ षतयों में एमबरषग्रस का
उपयोग औिषध एिं िबषत (potions) के षनमाषर् के साथ-साथ मसाले के रूप में भी ककया िाता है। पषिम संथकृ षतयों में इसका
उपयोग उत्कृ ष्ट इत्र की खुिबू को षथथर करने के षलए ककया िाता था। एमबरषग्रस समुर्द् में तैरता रहता है और मुख्य रूप से
चीन, िापान, अफ्रीका ि अमेररका के तटों पर तथा बहामास िैसे उष्र्करटबंधीय द्वीपों के तटों पर प्रिाषहत होते हुए पहुाँच
िाता है।
• तािा एमबरषग्रस काला ि मुलायम होता है तथा इसमें एक अरुषचकर गंध होती है। यद्यषप, िब यह सूय,ष हिा ि समुर्द्ी िल के
संपकष में आता है, तो यह हलके धूसर या पीले रं ग का हो िाता है तथा इस प्रकक्रया के दौरान इसमें एक भीनी ि रुषचर सुगध

षिकषसत हो िाती है। भारत में एमबरषग्रस का आयात ि षनयाषत प्रषतबंषधत है। ककसी के पास इस पदाथष का पाया िाना सीमा
िुलक अषधषनयम के अधीन एक दंडनीय अपराध है।
• थपमष व्हेल मुख्य रूप से षथिडों (squids) का षिकार करती हैं और इनका अनपचा षहथसा इनके द्वारा की गई उलटी के माध्यम
से बाहर षनकल िाता है। एमबरषग्रस का उपयोग मुख्य रूप से इत्र और सौंदयष प्रसाधन सामग्री के षनमाषर् में ककया िाता है और
इस पदाथष की कीमत का षनधाषरर् इसकी आयु के आधार पर होता है।
Q 17.C
• दषक्षर्-पषिम मानसून भारतीय प्रायद्वीप के दषक्षर्ी छोर के पास दो िाखाओं, अथाषत् अरब सागर िाखा और बंगाल की खाडी
की िाखा में षिभाषित हो िाता है। अत: यह भारत में दो िाखाओं में प्रिेि करता है - बंगाल की खाडी की िाखा एिं अरब
सागर की िाखा।

7 www.visionias.in ©Vision IAS

Google it:- https://upscpdf.com


https://t.me/UPSC_PDF Download From > https://upscpdf.com https://t.me/UPSC_PDF

• सिषप्रथम दषक्षर्-पषिम मानसून की प्रथम िाखा बंगाल की खाडी में उत्पन्न होती है िो उत्तर भारत के मैदानी क्षेत्रों में ििाष का
कारर् बनती है। मानसून की दूसरी िाखा अथाषत दषक्षर्-पषिम मानसून की अरब सागरीय िाखा द्वारा भारत के पषिमी तट
पर ििाष की िाती है। अरब सागरीय िाखा थार मरुथथल के षनम्न दाब िाले क्षेत्र की ओर प्रिाषहत होती है तथा बंगाल की
खाडी िाखा की तुलना में लगभग तीन गुना अषधक प्रबल होती है।
• अरब सागर से उत्पन्न होने िाली मानसूनी पिनें आगे तीन िाखाओं में षिभाषित हो िाती हैं:
o एक िाखा पषिमी घाट द्वारा बाषधत होती है। ये पिनें पषिमी घाट की ढलानों के सहारे आरोही क्रम में ऊपर उठती हैं
तथा पिषतीय ििाष पररघटना के पररर्ामथिरूप पिनाषभमुख ढालों पर 250 सेमी से 400 सेमी के बीच भारी ििाष होती
है। पषिमी घाट को पार करने के पिात ये पिनें नीचे की ओर अितषलत होती हैं और गमष होने लगती हैं। इससे इन पिनों
में आर्द्षता की कमी हो िाती है। पररर्ामथिरूप, पषिमी घाट के पूिष में इन पिनों से न्यूनतम ििाष होती है। कम ििाष िाला
यह क्षेत्र िृषष्ट-छाया क्षेत्र कहलाता है।
o अरब सागर से उत्पन्न होने िाली मानसून की दूसरी िाखा मुंबई के उत्तर में नमषदा और तापी नकदयों की घारटयों से प्रिेि
कर मध्य भारत के षिथतृत क्षेत्र में ििाष करती है। इस िाखा से छोटा नागपुर के पठार में 15 सेमी तक ििाष होती है। इसके
पिात यह गंगा के मैदान में प्रिेि करती है एिं बंगाल की खाडी से आने िाली मानसून की िाखा से षमल िाती है।
o इस मानसून की तीसरी िाखा सौराष्ट्र प्रायद्वीप एिं कच्छ में प्रिेि करती है। तत्पिात यह िाखा अरािली िेर्ी के
समानांतर प्रिाषहत होते हुए पषिम रािथथान से होकर गुिरती है और इस दौरान इस क्षेत्र में के िल अत्यलप ििाष ही
होती है। यह िाखा पंिाब ि हररयार्ा में बंगाल की खाडी की िाखा से षमल िाती है। ये दोनों िाखाएं एक-दूसरे के सहारे
प्रबषलत होकर पषिमी षहमालय (षििेि रूप से धमषिाला) में ििाष का कारर् बनती हैं।
• मानसून की अरब सागर िाखा षनम्नषलषखत दो कारर्ों से बंगाल की खाडी िाखा से अषधक प्रबल होती हैः
o अरब सागर िाखा का षिथतार बंगाल की खाडी िाखा से अषधक है।
o अरब सागर िाखा पूर्त
ष ः भारत की ओर प्रिाषहत होती है, िबकक बंगाल की खाडी िाखा का के िल एक भाग ही भारत में
प्रिेि करता है, िेि मयांमार, थाईलैंड ि मलेषिया की ओर प्रिाषहत होती है। अतः कथन 1 सही है।
• तषमलनाडु तट षनम्नषलषखत दो कारर्ों से इस मौसम में िुष्क बना रहता हैः
o तषमलनाडु तट दषक्षर्-पषिम मानसून की बंगाल की खाडी िाखा के समानांतर षथथत है।
o यह दषक्षर्-पषिम मानसून की अरब सागर िाखा के िृषष्टछाया क्षेत्र में षथथत है। अतः कथन 2 सही नहीं है।
• तषमलनाडु तट पर मानसून के षनितषन या िीतकाल के दौरान उत्तर-पूिी मानसून से ििाष होती है।
• मानसूनी ििाष एक बार में कु छ कदनों की कई आर्द्ष अिषधयों के रूप में होती है। इन आर्द्ष अिषधयों के बीच में कु छ िुष्क अिषधयााँ
भी आती हैं, षिन्हें षिच्छेद (breaks) कहा िाता है। ििाष के इन षिच्छेदों का संबंध उन चक्रिातीय अिदाबों से है, षिनका
षनमाषर् बंगाल की खाडी के िीिष पर होता है और िो मुख्य भूषम पर प्रिेि करते हैं।
• इन अिदाबों की बारमबारता और गहनता के अषतररक्त इनके द्वारा अपनाया गया मागष भी ििाष के थथाषनक षितरर् को
षनधाषररत करता है। अतः कथन 3 सही है।
Q 18.D
• कथन 1 सही नहीं हैः काटोसैट-3 इसरो द्वारा षनर्मषत एिं षिकषसत एक उन्नत भारतीय भू-अिलोकन उपग्रह (earth
observation satellites) है। यह हाई ररज़ॉलयूिन इमेसिंग क्षमता िाला तीसरी पीढ़ी का एक उन्नत उपग्रह है। इसमें 0.25
मीटर का एक पैनक्रोमैरटक ररज़ॉलयूिन है, िो इसे षिश्व का सबसे अषधक ररज़ॉलयूिन िाला इमेसिंग सैटेलाइट बनाता है। यह
काटोसैट िृंखला के षपछले पेलोड की तुलना में एक बडा सुधार है।
• कथन 2 सही नहीं हैः 1,600 ककलोग्राम से अषधक ििनी (ककसी नैनो उपग्रह का ििन 10 ककलोग्राम से कम होता है)
काटोसैट-3 व्यापक पैमाने पर नगर षनयोिन, ग्रामीर् संसाधन एिं अिसंरचना षिकास, तटीय भूषम का उपयोग तथा भूषम
आच्छादन आकद के षलए प्रयोक्ताओं की बढ़ती मांगों की पूर्तष करेगा। काटोसैट-3 को भूमध्य रे खा पर 97.5 षडग्री के झुकाि के
साथ 509 ककमी की कक्षा में थथाषपत ककया गया था। इसके संभाषित उपयोगों में मौसम मानषचत्रर्, काटोग्राफी या सैन्य रक्षा
तथा रर्नीषतक अनुप्रयोग सषममषलत हैं।
Q 19.B
• िल-प्रदूिर् मानि गषतषिषधयों से उत्पन्न होता है। षिषभन्न मागों के माध्यम से प्रदूिर् धरातल या भूिल तक पहुाँचता है।
• सबंदु स्रोत प्रदूिर् (एक ही स्रोत से उत्पन्न) के षिपरीत गैर-सबंद ु स्रोत प्रदूिर् कई स्रोतों से उत्पन्न होने िाला प्रदूिर् होता है।
सबंदु स्रोतों की पहचान करना ि उनकी षनगरानी करना अपेक्षाकृ त सरल होता है। परंतु प्रदूिर् के गैर -सबंदु स्रोतों की पहचान
8 www.visionias.in ©Vision IAS

Google it:- https://upscpdf.com


https://t.me/UPSC_PDF Download From > https://upscpdf.com https://t.me/UPSC_PDF

करना एिं उन्हें प्रबंषधत करना करठन होता है। उदाहरर् के षलए, कृ षि अपिाह (खेत, पिुओं ि फसल-भूषम से षनकलने िाला),
अमलीय ििाष, तूफान के िल का अपिाह (सडकों, पार्किं ग थथल ि लॉन से षनकलने िाला) आकद।
• सरलता से पहचाने िाने िाले स्रोत या प्रदूिर् के थथान को सबंद ु स्रोत कहा िाता है, िैसे, नगरपाषलका ि औद्योषगक षिसिषन
पाइप षिनके माध्यम से प्रदूिक िल-स्रोत में प्रिाषहत होते हैं।
• अतः षिकलप (b) सही है।

Q 20.D
• कथन 1 सही नहीं हैः इनर लाइन परषमट (ILP) एक आषधकाररक यात्रा दथतािेि (िैिल डॉक्यूमेंट) है, िो ककसी भारतीय
नागररक को एक सीषमत अिषध के षलए ILP प्रर्ाली के तहत संरषक्षत राज्य में आिक यात्रा (inward travel) की अनुमषत
प्रदान है। संरषक्षत राज्य के बाहर के भारतीय नागररकों को उन राज्यों में प्रिेि करने के षलए यह परषमट प्राप्त करना अषनिायष
है। यह दथतािेज़ भारत के अंतरराष्ट्रीय सीमा के पास षथथत कु छ क्षेत्रों में आिािाही को षिषनयषमत करने का सरकार द्वारा
ककया गया एक प्रयास है। यह “बंगाल ईथटनष फ्रंरटयर रे गल
ु ेिन एक्ट (BEFRA), 1873” पर आधाररत है। इस अषधषनयम का
षनमाषर् षब्ररटि सरकार द्वारा "षब्ररटि प्रिा" को इन "संरषक्षत क्षेत्रों" में प्रिेि करने से रोककर चाय, तेल तथा हाथी के व्यापार
के संदभष में षब्ररटि ताि के षहतों की रक्षा करने हेतु ककया गया था। प्रत्येक भारतीय नागररक, िो उन संरषक्षत क्षेत्रों का षनिासी
नहीं है, उसे ILP की आिश्यकता होती है। हालांकक, NRI, PIO काडष धारक तथा OCI काडष धारक ILP के षलए पात्र नहीं हैं।
उन्हें एक षििेि परषमट के षलए आिेदन करना होता है, षिसे संरषक्षत/प्रषतबंषधत क्षेत्र परषमट कहा िाता है।
• कथन 2 सही नहीं है: ILP संबषं धत राज्य सरकारों द्वारा िारी एक आषधकाररक यात्रा दथतािेि है।
• कथन 3 सही नहीं हैः यह प्रर्ाली ितषमान में के िल चार पूिोत्तर राज्यों- अरुर्ाचल प्रदेि, नागालैंड, षमिोरम एिं मषर्पुर में
लागू है तथा कोई भी भारतीय नागररक, िब तक कक िह उस राज्य से संबंषधत न हो, इन राज्यों में से ककसी में भी यात्रा नहीं
कर सकता है और न ही िह ILP में षनर्दषष्ट अिषध से अषधक िहां षनिास कर सकता है। ILP में मषर्पुर को भी सषममषलत करने
की घोिर्ा 10 कदसंबर, 2019 को की गई थी, षिसे 1 िनिरी, 2020 से लागू कर कदया गया है।

Q 21.C
• िैि रासायषनक ऑक्सीिन मांग (BOD):
o BOD को िायिीय पररषथथषतयों में िैि-षनम्नीकरर्ीय पदाथों के अपघटन के षलए सूक्ष्मिीिों द्वारा प्रयुक्त ऑक्सीिन की
आिश्यक मात्रा के रूप में पररभाषित ककया िाता है। यह िल की मयुषनषसपल या िैषिक गुर्ित्ता की क्षमता षनधाषररत
करने के षलए सिाषषधक प्रयोग ककया िाने िाला मापदंड है, अथाषत् यह िैि-षनम्नीकरर्ीय प्रदूिकों का मापन करता है।
o मानक BOD परीक्षर् 20 षडग्री सेषलसयस के तापमान पर 5 कदनों की िायिीय पररषथथषत में अपषिष्ट िल के नमूने में
षिद्यमान िैि-षनम्नीकरर्ीय पदाथष के षनम्नीकरर् के षलए सूक्ष्म िीिों द्वारा आिश्यक ऑक्सीिन की मात्रा षनधाषररत करता
है। इसका मापन mg/l में ककया िाता है।
o BOD का मापन उपचार सुषिधाओं को षडिाइन करने के षलए ककया िाता है। इसका उपयोग एषक्टिेरटड थलि इकाइयों
और रिकसलंग कफलटर इकाइयों के आकार के षनधाषरर् हेतु ककया िाता है। इसके अषतररक्त, इसका उपयोग षिषभन्न उपचार
इकाइयों की दक्षता षनधाषररत करने के षलए भी ककया िाता है।
• रासायषनक ऑक्सीिन मांग (COD):
o रासायषनक ऑक्सीिन मांग (COD) को पोटैषियम परमैंगनेट, पोटैषियम डाइक्रोमेट िैसे ऑक्सीकारक अषभकताषओं का
उपयोग करके अपषिष्ट िल में षिद्यमान काबषषनक और अकाबषषनक रसायनों के रासायषनक ऑक्सीकरर् के षलए आिश्यक
ऑक्सीिन की मात्रा के रूप में िर्र्षत ककया िाता है।
o COD की उपषथथषत ककसी अषतररक्त उपकरर् के षबना िैषिक पदाथों के त्िररत रासायषनक ऑक्सीकरर् को सुगम बनाती
है। यह भारी षििाक्त सीिेि में िैषिक भार के षनधाषरर् को संभि बनाने िाली एकमात्र षिषध है। इसषलए, यह िैि-
षनम्नीकरर्ीय प्रदूिकों और गैर िैि-षनम्नीकरर्ीय प्रदूिकों, दोनों का मापन करती है।

9 www.visionias.in ©Vision IAS

Google it:- https://upscpdf.com


https://t.me/UPSC_PDF Download From > https://upscpdf.com https://t.me/UPSC_PDF

• BOD और COD के मध्य मुख्य अंतर: BOD िैषिक पदाथों को षिघरटत करने के षलए िायिीय िीिों द्वारा आिश्यक
ऑक्सीिन की मात्रा का मापन करती है और COD रासायषनक अषभकक्रया द्वारा अपषिष्ट िल में षिद्यमान काबषषनक और
अकाबषषनक घटकों को षिघरटत करने के षलए आिश्यक ऑक्सीिन का मापन करती है। इसषलए COD का मान BOD से
अषधक होता है।

Q 22.A
• षनिेि और षिकास पर मंषत्रमंडल सषमषत की अध्यक्षता प्रधानमंत्री द्वारा की िाती है और इसमें ितषमान में चार अन्य सदथय-
अषमत िाह (कें र्द्ीय गृह मंत्री); षनषतन गडकरी (कें र्द्ीय सडक पररिहन एिं रािमागष मंत्री/सूक्ष्म, लघु एिं मध्यम उद्यम मंत्री);
षनमषला सीतारमर् (कें र्द्ीय षित्त मंत्री) तथा षपयूि गोयल (कें र्द्ीय िाषर्ज्य मंत्री/रेल मंत्री) िाषमल हैं। इसषलए कथन 1 सही है।
इसे षनम्नषलषखत कायष सौंपे गए हैं -
o 1000 करोड रुपये या उससे अषधक के षनिेि िाली समयबद्ध आधार पर कायाषषन्ित की िाने िाली प्रमुख पररयोिनाओं
या अन्य महत्िपूर्ष पररयोिनाओं को षचषननत करना। इसषलए कथन 2 सही है।
o चयषनत क्षेत्रकों की पररयोिनाओं के संबंध में संबंषधत मंत्रालयों/षिभागों द्वारा अपेषक्षत अनुमोदन और मंिूररयााँ िारी
करने के षलए समय सीमा षनधाषररत करना;
o प्रत्येक अनुमोदन/मंिूरी प्राप्त करने हेतु षनधाषररत/ उसमें लगे समय और षिलंब (यकद हो तो) सषहत षचषननत की गई
पररयोिनाओं की प्रगषत की षनगरानी करना;
o अनुमोदन/मंिरू ी प्रदान करने में षिलंब हेतु उत्तरदायी मुद्दों सषहत षनधाषररत समय सीमा से अषधक षिलंषबत पररयोिनाओं
के कायाषन्ियन की समीक्षा करना;
o अनुमोदन और मंिरू ी प्रदान करने/प्रदान करने से इनकार करने के षलए मंत्रालयों/षिभागों द्वारा अपनाई िाने िाली
प्रकक्रयाओं की समीक्षा करना।
• कायष आिंटन षनयमों के अनुसार, सरकार मंषत्रमंडल सषमषतयों का गठन करती है और उनके कायों को षनधाषररत करती है।
• प्रधानमंत्री के प्रत्यक्ष षनयंत्रर् में कायषरत मंषत्रमंडल सषचिालय मंषत्रमंडल और मंषत्रमंडल सषमषतयों के षलए सषचिालय के रूप
में कायष करता है। इसषलए कथन 3 सही नहीं है।
Q 23.B
• षनिादीप्त मेघ (Noctilucent Clouds)
o षनिादीप्त मेघ या रात में चमकने िाले मेघ पृथ्िी के ऊपरी िायुमड
ं ल में महीन बादलों िैसी पररघटनाएं होते हैं। षनिादीप्त
मेघ षहम के रिों से षनर्मषत होते हैं और के िल खगोलीय धुंधलके (astronomical twilight) के दौरान कदखाई देते हैं।
लैरटन में Noctilucent (नॉक्टीलूसेंट) का अथष सामान्य तौर पर "रात में चमकना" होता है।
o इन्हें प्राय: भूमध्य रे खा से 50° से 70° उत्तरी और दषक्षर्ी अक्षांिों के मध्य ग्रीष्मऋतु के महीनों के दौरान देखा िाता है।
इसषलए कथन 2 सही है।
o षनिादीप्त मेघ के िल थथानीय ग्रीष्मऋतु में और सूयष के प्रेक्षक के षक्षषति (तल) से नीचे होने (हालांकक मेघ अभी भी सूयष
की प्रकाि में होते हैं) की षथथषत में कदखाई देते हैं।
o हाल के अध्ययनों से ज्ञात हुआ है कक िायुमंडल में उत्सर्िषत अषतररक्त मीथेन मध्यमंडल (mesosphere) में पहुंचने पर
अषतररक्त िल-िाष्प का उत्पादन करती है, षिससे षनिादीप्त मेघों का सुदढ़ृ ीकरर् होता है या ऐसे निीन मेघों का षनमाषर्
होता है।
o षनिादीप्त मेघ पृथ्िी के िायुमंडल में सिाषषधक ऊंचाई पर पाए िाने िाले मेघ हैं।
o षनिादीप्त मेघ लगभग 76 से 85 कक.मी. (47 से 53 मील) की ऊंचाई पर मध्यमंडल में षथथत होते हैं। इसषलए कथन 1
सही नहीं है।
o षनिादीप्त मेघ इतने अषधक महीन होते हैं कक इन्हें कदन के प्रकाि में नहीं देखा िा सकता है। ये के िल तब कदखाई देते हैं
िब िायुमंडल की षनचली परतें पृथ्िी की छाया में होती हैं और ये षक्षषति के नीचे षथथत सूयष के प्रकाि से प्रकाषित होते
हैं।

10 www.visionias.in ©Vision IAS

Google it:- https://upscpdf.com


https://t.me/UPSC_PDF Download From > https://upscpdf.com https://t.me/UPSC_PDF

Q 24.B
• बायोषसषमलर एक िैषिक षचककत्सीय उत्पाद (बायोथेरेप्यूरटक) होता है िो पहले से ही अनुमोकदत िैषिक दिा ('संदभष दिा') के
अत्यषधक समान होता है। बायोषसषमलर को सभी िैषिक दिाओं पर लागू भेिि (pharmaceutical) गुर्ित्ता, सुरक्षा और
प्रभािकाररता मानकों के आधार पर अनुमोकदत ककया िाता है। बायोषसषमलर आषधकाररक रूप से मूल "निप्रिषतक" उत्पादों का
थिीकृ त संथकरर् होते हैं और मूल उत्पाद के पेटेंट की समय सीमा समाप्त होने पर षिषनर्मषत ककए िा सकते हैं।
• बायोथेरेप्यूरटक्स िैषिक और सिीि स्रोतों से प्राप्त दिा उत्पाद होते हैं। इनमें षचककत्सीय टीके , रक्त, रक्त घटक, कोषिकाएं,
िीन षचककत्सा, ऊतक और अन्य सामषग्रयां सषममषलत होती हैं।
• बायोषसषमलरों को प्राय: भ्रमिि िेनेररक दिाएाँ मान षलया िाता है- ककं तु ये पूर्त
ष ः समान नहीं होती हैं।
o सामान्य दिाएाँ रासायषनक रूप से संश्लेषित होती हैं िबकक बायोषसषमलर को िरटल िीषित प्रर्ाषलयों में उत्पाकदत ककया
िाता है। इसषलए कथन (b) सही नहीं है।
o िैषिक दिाएं बडी, िरटल अर्ु या अर्ुओं का षमिर् होती हैं िो िीषित सामग्री से षनर्मषत हो सकती हैं। इस प्रकार
बायोषसषमलरों के अपने संदभष उत्पाद की सटीक प्रषतयां होने की संभािना नहीं होती है।
• हाल ही में षिश्ि थिाथथ्य संगठन (WHO) द्वारा पहली बायोषसषमलर दिा ‘िैषथटिमाब’ (Trastuzumab) को प्रारं षभक
अनुमषत प्रदान की गई है, षिससे यह महंगा, िीिन रक्षक उपचार िैषश्वक थतर पर मषहलाओं के षलए अषधक सथता और अषधक
सुलभ हो सकता है। 2015 में िैषथटिमाब (मोनोक्लोनल एंटीबॉडी) को लगभग 20% थतन कैं सर मामलों में उपचार हेतु
आिश्यक दिाओं की सूची में सषममषलत ककया गया था। इसने प्रारं षभक चरर् के थतन कैं सर के उपचार में और कु छ मामलों में
इस रोग के अषधक उच्चतम थतरों पर उच्च प्रभािकाररता प्रदर्िषत की है। संश्लेषित रसायनों के बिाय कोषिकाओं िैसे िैषिक
स्रोतों से उत्पाकदत बायोथेरेप्यूरटक दिाएाँ कैं सर के कु छ रूपों और अन्य गैर-संचारी रोगों के षलए महत्िपूर्ष उपचार हैं। िेनेररक
दिाइयों की भांषत ही, बायोषसषमलर समान प्रभाि रखने िाले निप्रितषक बायोथेरेप्यूरटक्स के कम महंगे संथकरर् हो सकते हैं।
इन्हें अन्य कं पषनयों द्वारा सामान्यत: मूल उत्पाद पर पेटेंट समाप्त हो िाने के पिात् षिषनर्मषत ककया िाता है।
Q 25.A
• अलबीडो सौर षिककरर् की पृथ्िी की सतह से अंतररक्ष में परािर्तषत होने िाली मात्रा को कहा िाता है। यह पराितषन गुर्ांक
होता है और इसका मान एक से कम होता है।
• िब सौर षिककरर् िायुमंडल से होकर संचररत होता है, तो इसकी एक षनषित मात्रा प्रकीर्र्षत, परािर्तषत और अििोषित
होती है। षिककरर् का परािर्तषत योग पृथ्िी का अलबीडो कहलाता है।
• अलबीडो िलिायु षिज्ञान, खगोल षिज्ञान और पयाषिरर् प्रबंधन में एक महत्िपूर्ष अिधारर्ा है। यह पृथ्िी की सतह के ऊिाष
संतल
ु न में महत्िपूर्ष भूषमका का षनिषहन करता है, क्योंकक यह आपषतत सौर षिककरर् के अििोषित होनी की दर को षनधाषररत
करता है।
• पृथ्िी के औसत अलबीडो में अंतर का पृथ्िी के तापमान पर व्यापक प्रभाि पडता है। यकद औसत अलबीडो षिगत ििष के अलबीडो
की तुलना में कम होता है तो यह अििोषित षिककरर् की अषधक मात्रा को षनर्दषष्ट करता है। इससे पृथ्िी के तापमान में िृषद्ध
होती है। िैषश्वक तापन की षनगरानी िाले उपग्रहों का उपयोग करके षनरं तर पृथ्िी के अलबीडो का मापन ककया िाता है।
• षिषभन्न प्रकार की सतहों का अलबीडो
सतह परािर्तषत षिककरर् का प्रषतित
षहमाच्छाकदत सतह 70-90
रे त 20-30
घास 14-37
िुष्क मैदान 15-25
आर्द्ष मैदान 10
घासभूषम 10-25
िल 3-5
सघन मेघ 70-80
षिरल मेघ 25-50
काली षमट्टी 8-14

11 www.visionias.in ©Vision IAS

Google it:- https://upscpdf.com


https://t.me/UPSC_PDF Download From > https://upscpdf.com https://t.me/UPSC_PDF

Q 26.B
• प्लेट षिितषषनक षसद्धांत (PlateTectonics Theory) के अनुसार, एक षिितषषनक प्लेट (षिसे षलथोथफे ररक प्लेट भी कहा
िाता है) ठोस चट्टान की षििाल एिं अषनयषमत आकार का एक खंड होती है, िो सामान्यत: महाद्वीपीय और महासागरीय
थथलमंडलों से षमलकर बनी होती है। ये प्लेटें दुबल
ष तामंडल (asthenosphere) पर एक दृढ इकाई के रूप में क्षैषति रूप से
संचलन करती हैं। षनम्नषलषखत तीन प्रकार की प्लेट अंतःकक्रयाएं होती हैं, षिन्हें प्लेट सीमाएं भी कहा िाता है:
o अपसारी सीमाएं (Divergent Boundaries): िहां प्लेटों के एक-दूसरे से दूर हटने पर नई भूपपषटी का षनमाषर् होता है।
षिन थथानों पर प्लेटें एक-दूसरे से दूर हटती हैं, इन्हें प्रसारी थथान (spreading sites) कहा िाता है। अपसारी सीमाओं
का सबसे िेष्ठ उदाहरर् मध्य-अटलांरटक कटक है। अन्य उदाहरर् लाल सागर ररफ्ट,बैकाल ररफ्ट िोन, पूिी अफ्रीकी भ्रंि
घाटी हैं। इसषलए युग्म 1 सही सुमषे लत है।
o अषभसारी सीमाएं (Convergent boundaries): िहां षलथोथफे ररक प्लेटें एक-दूसरे की ओर गषत करती हैं। इन क्षेत्रों में
प्लेटों के परथपर टकराि से भूकंप, ज्िालामुखीय गषतषिषधयां और भूपपषटी षिरूपर् उत्पन्न हो सकता है। िापान,
एलयूषियन द्वीपसमूह, षहमालय आकद इसके कु छ उदाहरर् हैं। इसषलए युग्म 2 सही सुमषे लत है।
o रूपांतररत सीमाएं (Transform Boundaries): िहां न तो भूपपषटी षनर्मषत होती है और न ही नष्ट होती है, क्योंकक प्लेटें
परथपर क्षैषति रूप से गषत करती हैं। रूपांतररत भंि दो प्लेटों का पृथक्करर् तल होता है िो सामान्यत: मध्य-महासगरीय
कटकों के लंबित षथथत होते हैं। सैन एंषड्रयास इस प्रकार की अंतःकक्रया का एक सामान्य उदाहरर् है। इसषलए युग्म 3 सही
सुमषे लत है।
Q 27.A
• चीन के राष्ट्रपषत िी षिनसपंग और उनके रूसी समकक्ष व्लाकदमीर पुषतन ने "पािर ऑफ साइबेररया" गैस पाइपलाइन का
उद्घाटन ककया। यह एक िृहद् सीमा-पारीय पाइपलाइन पररयोिना है िो न के िल चीन की ऊिाष सुरक्षा के षलए महत्िपूर्ष है
बषलक बीसिंग और मॉथको के मध्य षििेि संबंधों को सुदढ़ृ करने के षलए भी महत्िपूर्ष है। इस पररयोिना के अंतगषत आगामी
30 ििों में 400 षबषलयन डॉलर के गैस समझौते का अनुबंध ककया गया है।
• यह पूिी साइबेररया में गज़प्रोम द्वारा संचाषलत पाइपलाइन है षिसके माध्यम से याकु रटया से षप्रमोथकी क्राय और चीन तक
प्राकृ षतक गैस का पररिहन ककया िाएगा। यह साइबेररया से चीन तक पूिी गैस मागष का भाग है।

Q 28.C
• निीन िन्यिीि कायष योिना 2017-2031 (Wildlife Action Plan: NWAP) को ऐसे समय में सरकारी हथतक्षेप के षलए
एक फ्रेमिकष के रूप में तैयार ककया गया है िब देि में पयाषिास हाषन और मानि-िन्यिीि संघिष में तेिी से िृषद्ध हो रही है।
NWAP िह नीषतगत ढांचा है षिस पर आगामी 15 ििों में संरषक्षत क्षेत्रों (protected areas: PA) के षलए प्रबंधन
योिनाएं षिकषसत की िाएंगी। ितषमान योिना िन्यिीि संरक्षर् के षलए भूदश्ृ य दृषष्टकोर् (landscape approach)
अपनाने पर बल देती है। उललेखनीय है कक यह षपछले संरषक्षत क्षेत्र-कें कर्द्त दृषष्टकोर् में एक महत्िपूर्ष सुधार को दिाषती है।
इसषलए दोनों कथन सही हैं।
• हाल ही में आरंभ की गई यह कायष योिना देि के षलए तीसरी योिना है। ज्ञातव्य है कक पहली और दूसरी कायष योिना को
क्रमिः 1983 से 2001 तक, 2002 से 2016 तक कायाषषन्ित ककया गया था।
• NWAP इं षगत करती है कक िन्यिीि संरक्षर् के षलए भूदश्ृ य दृषष्टकोर् अपनाने से मानि-िन्यिीि संघिों के मुद्दे के पयषिेक्षर्
में सहायता षमल सकती है, षिनमें व्यापक संदभष में, हाषलया ििों में िृषद्ध हुई है। िहााँ एक ओर िन्यिीि PA से बाहर संचरर्
करते हैं िहीं दूसरी ओर मानि द्वारा िन्यिीिों के पयाषिासों में षनरंतर घुसपैठ की िा रही है। इस प्रकार PA की सीमाओं के
भीतर संरक्षर् का पयषिेक्षर् करने का पारंपररक दृषष्टकोर् ितषमान में उतना प्रासंषगक नहीं है और इसषलए भूदश्ृ य को उसकी
संपूर्त
ष ा में देखने की आिश्यकता है ताकक षिकास और संरक्षर् को एक साथ प्राथषमकता प्रदान की िा सके ।
• साथ ही, यह पहली बार है कक यह योिना िन्यिीि प्रबंधन षनयोिन प्रकक्रया में इनके िमन और अंगीकरर् के षलए आिश्यक
कारष िाइयों को एकीकृ त करके , िन्यिीिों पर िलिायु पररितषन के प्रभाि से संबषं धत सचंताओं की पहचान करती है। िलिायु
पररितषन पर षिियगत ध्यान के षन्र्द्त करना (थीमेरटक फोकस) भी एक निीन दृषष्टकोर् है। िलिायु पररितषन को एक
िाथतषिकता के रूप में थिीकार करते हुए, NWAP ने िलिायु पररितषन के षलए षिषिष्ट अनुसध
ं ान को बढ़ािा देने की योिना
12 www.visionias.in ©Vision IAS

Google it:- https://upscpdf.com


https://t.me/UPSC_PDF Download From > https://upscpdf.com https://t.me/UPSC_PDF

बनाई है, षिसमें िनथपषत प्रकारों के षितरर् और पाररषथथषतकी तंत्रों में पररितषनों की दीघषकाषलक षनगरानी और आकलन
िाषमल है। साथ ही, अषि, महामारी, सूखा और पररिर्तषत िलिायु से उत्पन्न अन्य पयाषिरर्ीय दबािों के षलए सुभेद्यता
मानषचत्रर् पर भी बल कदया गया है।

Q 29.A
• कथन 1 सही नहीं है: राष्ट्रीय हररत अषधकरर् (National Green Tribunal: NGT) की थथापना 18.10.2010 को राष्ट्रीय
हररत अषधकरर् अषधषनयम, 2010 के तहत की गई थी। इसका कायष पयाषिरर् संरक्षर् और िन संरक्षर् और अन्य प्राकृ षतक
संसाधन सषहत पयाषिरर् से संबंषधत ककसी भी कानूनी अषधकार के प्रितषन और क्षषतग्रथत व्यषक्त अथिा संपषत्त के षलए अनुतोि
(relief) और क्षषतपूर्तष प्रदान करना और इससे िुडेे़ हुए मामलों का प्रभाििाली और तीव्र गषत से षनपटारा करने के षलए ककया
गया है।
• कथन 2 सही है: राष्ट्रीय हररत अषधकरर् अषधषनयम, 2010 की धारा 19 के अंतगषत अषधकरर् को अपनी प्रकक्रया को
षिषनयषमत करने की िषक्त प्रदान की गई है। इसके अषतररक्त, अषधकरर् षसषिल प्रकक्रया संषहता, 1908 अथिा भारतीय साक्ष्य
अषधषनयम, 1872 की प्रकक्रया के अनुरूप कायष करने के षलए बाध्य नहीं है तथा प्राकृ षतक न्याय के षसद्धांतों से षनदेषित होता
है। हालांकक, अषधकरर् में अपने कायों का षनिषहन करने के षलए षसषिल प्रकक्रया संषहता के तहत प्रदत्त षसषिल न्यायालय की
िषक्तयां षनषहत है।
• अषधकरर् द्वारा अपने थियं के षनयम तैयार ककए गए हैं क्योंकक अषधकरर् को ककए गए आिेदन षसषिल मुकदमे या ररट
याषचकाओं से थिाभाषिक रूप से पृथक हैं। अषधकरर् NGT अषधषनयम, 2010 की अनुसच
ू ी 1 में उषललषखत आिश्यक
संषिषधयों के अंतगषत यथा आिश्यक पक्षकारों की पहचान करता है तथा उनसे ईमेल द्वारा तुरंत प्रषतकक्रया व्यक्त करने की मांग
करता है षिससे समय और लागत की बचत होती है। आिेदन में प्रषतिादी (respondents) के रूप में नाषमत प्रत्येक पक्षकार
को यंत्रित रूप में (mechanically) नोरटस िारी नहीं ककया िाता है।
• कथन 3 सही है: अषधकरर्, आिेदन दाषखल के 6 माह के भीतर, आिेदनों या अपील के षनपटान करने हेतु अषधदेषित है। प्रारंभ
में, NGT की पीठों को पांच थथानों पर थथाषपत करने तथा थियं को अषधक सुलभ बनाने के षलए सर्कष ट प्रकक्रया का अनुकरर्
करने का प्रथताि रखा गया था। अषधकरर् की प्रमुख पीठ नई कदलली में षथथत है और भोपाल, पुर्े, कोलकाता और चेन्नई
अषधकरर् की अन्य चार पीठे हैं।
Q 30.C
• िलिायु पररितषन और भूषम (Climate Change and Land): यह IPCC द्वारा िारी िलिायु पररितषन, मरुथथलीकरर्,
भूषम षनम्नीकरर्, संधारर्ीय भूषम प्रबंधन, खाद्य सुरक्षा और थथलीय पाररषथथषतकी तंत्रों में ग्रीनहाउस गैसों के उत्सिषन
संबंषधत एक षििेि ररपोटष है। यह ररपोटष अगथत 2019 में प्रकाषित की गई थी।
• बदलते िलिायु पररदृश्य में महासागर और षहममंडल (क्रायोथफीयर) पर षििेि ररपोटष (Special Report on the Ocean
and Cryosphere in a Changing Climate: SROCC): छठे मूलयांकन चक्र (Sixth Assessment Cycle) के दौरान
तीन षििेि ररपोटष तैयार करने हेतु 2016 में IPCC पैनल के षनर्षय के अनुसरर् के क्रम में SROCC तैयार की गई थी।
SROCC ‘ग्लोबल िार्मिंग ऑफ 1.5 षडग्री सेषलसयस (SR1.5)’ तथा ‘क्लाइमेट चेंि एंड लैंड (SRCCL)’ संबंधी अन्य दो
षििेि ररपोटों का अनुसरर् करता है। SROCC ररपोटष षसतंबर 2019 में प्रकाषित की गई थी।
• 1.5ºC का िैषश्वक तापन (Global Warming of 1.5ºC), यह िलिायु पररितषन के खतरे के प्रषत िैषश्वक अनुकक्रया को सुदढ़ृ
बनाने, संधारर्ीय षिकास और षनधषनता का उन्मूलन करने के प्रयासों के संदभष में , िैषश्वक तापन के पूिष-औद्योषगक थतर से
1.5°C अषधक होने और संबंषधत िैषश्वक ग्रीनहाउस गैस उत्सिषन से उत्पन्न प्रभािों से संबंषधत IPCC की एक षििेि ररपोटष है।
यह ररपोटष अक्टूबर 2018 में प्रकाषित की गई थी।
Q 31.D
• खुले मैदान में प्लाषथटक अपषिष्ट का दहन िायु प्रदूिर् का एक प्रमुख स्रोत है। प्राय: नगरपाषलका ठोस अपषिष्ट (षिसमें
लगभग 12% प्लाषथटक होता है) का दहन कर कदया िाता है, षिससे िायुमड
ं ल में डाईऑषक्सन, फ्यूरॉन, पारा और
पॉलीक्लोरीनेटेड बाईकफनाइल िैसी षििाक्त गैसें उत्सर्िषत होती हैं।
13 www.visionias.in ©Vision IAS

Google it:- https://upscpdf.com


https://t.me/UPSC_PDF Download From > https://upscpdf.com https://t.me/UPSC_PDF

• डाईऑषक्सन एक सामान्य पद है, िो पयाषिरर् में षचरथथायी सैकडों रसायनों के समूह को संदर्भषत करता है। सिाषषधक षििाक्त
यौषगक 2,3,7,8-टेिाक्लोरोडाईबेंिो-पी-डाईऑषक्सन या TCDD होता है।
o डाईऑषक्सन अपषिष्ट भथमीकरर्, रासायषनक और कीटनािक षिषनमाषर् तथा लुगदी एिं कागि षिरंिन िैसी क्लोरीन से
संबद्ध औद्योषगक प्रकक्रयाओं के अषनषच्छत उपोत्पाद के रूप में षनर्मषत होता है। डाइऑषक्सन एिेंट ऑरेंि का प्राथषमक
षििाक्त घटक था {एिेंट ऑरेंि िाकनािी और षनष्पत्रर् (पषत्तयां षगराने िाला) रसायन है, िो "सामररक रूप से उपयोग
ककए िाने िाले" रे न्बो हबीसाइड्स में से एक है}। ज्ञातव्य है कक अमेररकी सेना द्वारा ििष 1961 से ििष 1971 तक
षियतनाम युद्ध के दौरान अपने रासायषनक युद्ध कायषक्रम, ऑपरे िन रैंच हैंड के एक भाग के रूप में एिेंट ऑरेंि का व्यापक
रूप से उपयोग ककया गया था।
• फ्यूरॉन षििमचक्रीय काबषषनक यौषगक है। यह रंगहीन, ज्िलनिील (षिसका िथनांक लगभग सामान्य तापमान के समान होता
है) अत्यषधक िाष्पिील र्द्ि है। यह ऐलकोहॉल, ईथर और एषसटोन सषहत कु छ सामान्य काबषषनक षिलायकों में घुलनिील है,
परन्तु यह िल में अलप घुलनिील है।
• सीसा: यह पेिोल, डीिल, लेड एषसड बैटररयों, पेंट, हेयर डाई आकद उत्पादों में षिद्यमान होता है। यह तंषत्रका तंत्र को क्षषत
पहुंचा सकता है और पाचन संबध
ं ी समथयाओं तथा कु छ मामलों में कैं सर का कारर् भी बन सकता है। सीसा षििेि रूप से बच्चों
को प्रभाषित करता है।
• पारा: तुलनात्मक रूप से दुलभ
ष तत्ि होते हुए भी पारा, पयाषिरर् में सिषव्यापी है। यह प्राकृ षतक भूिैज्ञाषनक गषतषिषधयों और
मानि षनर्मषत प्रदूिर् का पररर्ाम है। प्राकृ षतक स्रोतों से पारा अपक्षय, षिघटन और िैषिक प्रकक्रयाओं के माध्यम से िलीय
पयाषिरर् में प्रिेि कर सकता है। मनुष्य के षलए अत्यंत उपयोगी होते हुए भी पारा, षििेि रूप से षमथाइल मकष री के रूप में
मानि िीिन के षलए अत्यषधक षििाक्त होता है, क्योंकक यह मानि िरीर द्वारा उत्सर्िषत नहीं होता है और इस कारर् संषचत
षिि के रूप में कायष करता है।
• बेररयम: यह रित-श्वेत (षसलिरी-निाइट) रं ग की धातु है, िो िायु के संपकष में आने पर रित-पीत (षसलिरी-येलो) रं ग की हो
िाती है। बेररयम प्रकृ षत में अनेक षभन्न-षभन्न रूपों में पाया िाता है, षिन्हें यौषगक कहा िाता है। ये यौषगक ठोस होते हैं तथा
पाउडर या रिों के रूप में षिद्यमान होते हैं, और इनका भलीभांषत दहन नहीं होता है।

Q 32.A
• षबना ककसी रासायषनक पररितषन के खंषडत होने और संदषलत होने के कारर् मूल चट्टानों में यांषत्रकी व्यिधान एिं उनका
पुनसिंगरठत होना गषतिील कायांतरर् कहलाता है। ऊष्मीय कायांतरर् के कारर् चट्टानों के पदाथों में रासायषनक पररितषन एिं
पुनर्क्रषथटलीकरर् होता है।
• ऊष्मीय कायांतरर् दो प्रकार के होते हैं यथा- संपकष कायांतरर् और प्रादेषिक कायांतरर्। संपकष कायांतरर् में, चट्टानें गमष ि
षनथसृत मैग्मा और लािा के संपकष में आती हैं तथा उच्च तापमान में चट्टानों के पदाथों का पुनर्क्रषथटलीकरर् होता है। प्राय:
चट्टानों में मैग्मा अथिा लािा के योग से नि पदाथष षनर्मषत होते हैं।
• प्रादेषिक कायांतरर् में, उच्च तापमान या दबाि अथिा इन दोनों के संयोिन से षिितषषनक दबाि के कारर् होने िाले षिरूपर्
से चट्टानों में पुनर्क्रषथटलीकरर् होता है। कायांतरर् की प्रकक्रया में चट्टानों के कु छ कर् या खषनि परतों अथिा रे खाओं के रूप में
व्यिषथथत हो िाते हैं। अत: कायांतररत चट्टानों में खषनि अथिा कर्ों के इस व्यिथथापन को पत्रर् (Foliation) या रे खांकन
(lineation) कहते हैं।
• अपिलकन (Exfoliation), उत्खनन और अपक्षय के दौरान भौषतक एिं रासायषनक प्रकक्रयाओं की िृंखला के कारर् चट्टान की
सतह से कु छ षमलीमीटर से कु छ मीटर तक मोटी परतों का िलकन (परतों का षिथथापन) है।
• खषनिीभिन (Mineralization) (मृदा षिज्ञान में) रासायषनक यौषगकों का काबषषनक पदाथष में अपघटन है, षिससे इन यौषगकों
के पोिक तत्ि पौधों द्वारा ग्रहर् ककए िा सकने िाले घुलनिील अकाबषषनक रूपों में षिमुक्त होते हैं।
• षिलीभिन (Lithification) िह प्रकक्रया है, षिसमें अिसाद दबाि के कारर् सघन हो िाते हैं, उनमें षनक्षेषपत र्द्ि षनष्काषसत
हो िाता है और कालांतर में िे ठोस चट्टानों में रूपांतररत हो िाते हैं।

14 www.visionias.in ©Vision IAS

Google it:- https://upscpdf.com


https://t.me/UPSC_PDF Download From > https://upscpdf.com https://t.me/UPSC_PDF

Q 33.A
• डेलटा षनक्षेपर्ात्मक भूआकृ षतयां हैं। नकदयों द्वारा लाया गया अिसाद समुर्द् में षिसर्िषत और षिथताररत हो िाता है। समुर्द् में
दूर तक षिथताररत नहीं होने की षथथषत में यह अिसाद तट पर ही िंकु के रूप में षनक्षेषपत हो िाता है, षिसे डेलटा के रूप में
िर्र्षत ककया िाता है। डेलटा के षनमाषर् के षलए अनुकूल पररषथथषतयां षनमनानुसार हैं:
o नदी के ऊपरी मागष में सकक्रय ऊध्िाषधर और पाश्वष अपरदन षिसके द्वारा अंततः डेलटा के रूप में षनक्षेषपत होने िाला व्यापक
अिसाद प्राप्त होता है। इसषलए कथन 1 सही है।
o तट संरषक्षत (ज्िार रषहत) होना चाषहए।
o डेलटा से संलि समुर्द् उथला होना चाषहए अन्यथा नदी द्वारा लाए गए अिसाद गहरे िल में प्रिाषहत हो सकते हैं। इसषलए
कथन 2 सही नहीं है।
o नदी के मागष में कोई षििाल झील नहीं होनी चाषहए अन्यथा अिसाद झील तल में षनक्षेषपत हो िाता है। इसषलए कथन 3
सही नहीं है।
o नदी के मुहाने पर कोई प्रबल धारा प्रिाषहत नहीं होनी चाषहए अन्यथा अिसाद नदी से पृथक हो िाता है।

Q 34.C
• मेक इन इं षडया षमटलथटैंड (MIIM) भारत में िमषन लघु और मध्यम उद्योग (SME) को षनिेि हेतु प्रोत्साषहत करने के षलए
लॉन्च ककया गया था। उललेखनीय है कक इसने षद्वपक्षीय व्यापार संबंधों को और अषधक प्रोत्साषहत करते हुए िमषनी के उद्यषमयों
में गहन रुषच का सृिन ककया है। यह भारत में िमषन षमटलथटैं ड एंड फै षमली आउं ड् कं पषनयों द्वारा षनिेि को सुषिधािनक
बनाने हेतु प्रारंभ एक बािार प्रिेि सहायता कायषक्रम है।
• यह कायषक्रम न के िल भारत में कें र्द्ीय और राज्य सरकार के संबद्ध मंत्रालयों की सहायता से , बषलक उन प्रमुख उद्योग भागीदारों
को भी संलि करके कायाषषन्ित ककया िा रहा है, िो भारत में बािार प्रिेि के षिषभन्न पहलुओं में कं पषनयों की सहायता कर
सकते हैं।
• हाल ही में आयोषित भारत-िमषन सममेलन में, MIIM कायषक्रम की सफलता की सराहना की गई। ज्ञातव्य है कक इस कायषक्रम ने
1.25 षबषलयन यूरो से अषधक के घोषित षनिेि के साथ 135 से अषधक िमषन षमटलथटैंड कं पषनयों को सहायता प्रदान की है।

Q 35.A
• संयक्त
ु िन प्रबंधन (JFM) ििष 1988 की राष्ट्रीय िन नीषत के अनुसरर् में आरं भ ककया गया एक दृषष्टकोर् और कायषक्रम है।
इसके अंतगषत राज्यों के िन षिभाग, िन संरक्षर् एिं प्रबंधन के षलए थथानीय िनिासी और िन सीमा के षनकट आिाषसत
समुदायों की सहायता करते हैं तथा िनों पर होने िाली लागतों एिं िनों से प्राप्त होने िाले लाभों को उनके साथ साझा करते हैं।
समुदाय, षनकटिती िनों के संरक्षर् एिं प्रबंधनाथष थथानीय थतर पर षनर्मषत उप-षनयमों और सूक्ष्म योिनाओं से षनदेषित होकर
JFM सषमषतयों में संगरठत होते हैं। JFM की एक मुख्य षििेिता यह है कक समुदायों को सदथयों द्वारा िनों के उपयोग का
प्रबंधन करने और साथ ही गैर-सदथयों को षनष्काषसत करने की िषक्त प्राप्त होती है।
• संयक्त
ु िन प्रबंधन सषमषत (JFMC) िनिासी और िन सीमा के षनकट षनिाषसत समुदायों का लोकतांषत्रक, षिकें र्द्ीकृ त और
पारदिी थथानीय संथथान है, िो पूर्त
ष : या अंित: ग्राम सभा का एक भाग होता है। संयक्त
ु िन प्रबंधन सषमषत की थथापना
राज्य के प्रितषनीय JFM षनयमों/कदिा-षनदेिों के प्रािधानों के अनुसार की िाती है। सामान्यत: एक रािथि गांि में एक ही
JFMC गरठत की िाती है। पाररषथथषतकी षिकास सषमषत (EDC) कायाषत्मक थतर पर JFMC के समान ही होती है, परन्तु ये
संरषक्षत क्षेत्रों और उनके बफर क्षेत्रों में षथथत गााँिों से संबद्ध होती है। इनकी थथापना, कायषप्रर्ाली, भूषमका, उत्तरदाषयत्ि,
िषक्तयां, षित्त आकद राज्य द्वारा िारी आदेिों के अनुसार षनधाषररत होते हैं। इसषलए कथन 1 सही है और कथन 2 सही नहीं है।
• ितषमान में लगभग 20 षमषलयन से अषधक लोगों से षमलकर षनर्मषत 1,18,213 JFMCs 25 षमषलयन हेक्टेयर से अषधक िन
क्षेत्रों का प्रबंधन कर रही हैं। इस प्रकार का प्रबंधन अलप उत्पादकता और काबषन का षनम्नथतरीय भंडारर् करने िाले िनों को
समृद्ध िनों में पररिर्तषत करने के प्रषत प्रषतबद्धता को प्रदर्िषत करता है, षिससे काबषन भंडारर् में िृषद्ध होती है। JFM देि के
कु ल िन क्षेत्र के लगभग 29.8% भाग को किर करता है (REDD+ थिेटिी इं षडया, 2018)।

15 www.visionias.in ©Vision IAS

Google it:- https://upscpdf.com


https://t.me/UPSC_PDF Download From > https://upscpdf.com https://t.me/UPSC_PDF

Q 36.B
• पृथ्िी के चुंबकीय क्षेत्र को उत्तरी और दषक्षर्ी ध्रुिों द्वारा पररभाषित ककया गया है, िो सामान्यत: पृथ्िी के घूर्न
ष अक्ष के साथ
संरेषखत होते हैं। चुंबकीय बल रे खाएं उत्तरी गोलाधष में पृथ्िी के आंतररक भाग की ओर तथा दषक्षर्ी गोलाधष में पृथ्िी के
आंतररक भाग से बाहर की ओर गमन करती हैं। पृथ्िी के आंतररक भाग में, ऊष्मा ठोस आंतररक क्रोड से बाह्य तरल क्रोड की
ओर थथानांतररत होती हैं, षिसके पररर्ामथिरूप बाह्य क्रोड की र्द्षित लौह धातु में संिहन की कक्रया संपन्न होती है। चूाँकक
लोहा एक धातु है और षिद्युत का चालन (षपघली हुई अिथथा में होने पर भी) करता है, अत: इसकी संचलन गषत के कारर्
चुंबकीय क्षेत्र का षनमाषर् होता है। इसषलए, र्द्षित लौह के संिहनी संचलन के पररर्ामथिरूप बाह्य क्रोड (न कक आंतररक क्रोड)
के भीतर पृथ्िी का चुब
ं कीय क्षेत्र सृषित होता है। इसषलए कथन 1 सही नहीं है।
• भूगभीय समय पर पृथ्िी का चुब
ं कीय क्षेत्र षथथर नहीं रहा है। उललेखनीय है कक चुंबकीय क्षेत्र के आिषधक क्षय और इसके पुन:
थथाषपत होने संबंधी कारर्ों को अभी तक पूर्ष रूप से ज्ञात नहीं ककया िा सका है। िब यह पुनथथाष षपत होता है, तो यह क्षय
होने से पूिष की षथथषत की ओर उन्मुख हो सकता है या षिपरीत ध्रुिीयता की ओर अषभषिन्यथत हो सकता है। अत: पृथ्िी के
चुंबकीय क्षेत्र में हुए पररितषनों का भूगभीय इषतहास की घटनाओं का काल षनधाषररत करने में उपयोग ककया िा सकता है।
इसषलए कथन 2 सही है।
Q 37.B
• कथन 1 सही है: बषहरूष्र्करटबंधीय चक्रिातों (Extra tropical cyclones) का षनमाषर् ध्रुिीय िाताग्र के साथ-साथ होता है।
उत्तरी गोलाद्धष में, िाताग्र के दषक्षर् में गमष िायु और उत्तर कदिा से ठं डी िायु प्रिाषहत होती है। िब िाताग्र के अनुकदि
िायुदाब में षगरािट होती है, तो गमष िायु उत्तर कदिा की ओर प्रिाषहत होती है और ठं डी िायु िामाितष चक्रिातीय पररसंचरर्
का षनमाषर् करते हुए दषक्षर् कदिा की ओर प्रिाषहत होती है। यह चक्रिाती पररसंचरर् उष्र् िाताग्र और िीत िाताग्र के साथ
सुषिकषसत बषहरूष्र्करटबंधीय चक्रिात का षनमाषर् करता है।
• बषहरूष्र्करटबंधीय चक्रिात षनम्नषलषखत कारर्ों से उष्र्करटबंधीय चक्रिात से षभन्न होते हैं:
o बषहरूष्र्करटबंधीय चक्रिातों में थपष्ट िाताग्री प्रर्ाली दृषष्टगत होती है, िो उष्र्करटबंधीय चक्रिातों में उपषथथत नहीं
होती है।
o बषहरूष्र्करटबंधीय चक्रिात का प्रभाि एक षिथतृत क्षेत्र पर होता है और इसकी उत्पषत्त थथल और समुर्द् दोनों पर हो
सकती है। िहीं उष्र्करटबंधीय चक्रिात की उत्पषत्त के िल समुर्द्ों पर होती है और थथलीय भागों पर पहुंचने पर ये नष्ट हो
िाते हैं।
o ऊष्र्करटबंधीय चक्रिात की तुलना में बषहरूष्र्करटबंधीय चक्रिात िृहद् क्षेत्र को प्रभाषित करते हैं।
o उष्र्करटबंधीय चक्रिात में िायु का िेग अत्यषधक होता है और यह अषधक षिनािकारी होता है।
• कथन 2 सही नहीं है: बषहरूष्र्करटबंधीय चक्रिात पषिम से पूिष कदिा की ओर गषत करते हैं, िबकक उष्र्करटबंधीय चक्रिात,
पूिष से पषिम कदिा की ओर गषत करते हैं।
• उष्र्करटबंधीय चक्रिात एक प्रकार के प्रचंड तूफान होते हैं, षिनकी उत्पषत्त उष्र्करटबंधीय महासागरों पर होती है। ये तटीय
क्षेत्रों की ओर गषत करते हैं तथा इन क्षेत्रों में ये उग्र पिनों, अत्यषधक ििाष और तूफान महोर्मष के कारर् अत्यषधक षिनाि करते
हैं।
• कथन 3 सही है: उष्र्करटबंधीय तूफानों के षनमाषर् और तीव्रता के षलए अनुकूल पररषथथषतयां षनम्नषलषखत हैं:
o बृहत् समुर्द्ी सतह, िहां तापमान 27 षडग्री सेषलसयस से अषधक हो;
o कोररऑषलस बल की उपषथथषत (इसषलए कथन 3 सही है);
o ऊध्िाषधर पिनों की गषत में अलप षभन्नता;
o पहले से षिद्यमान दुबल
ष षनमन दाब क्षेत्र या षनम्न थतर के चक्रिाती पररसंचरर् का होना;
o समुर्द् तल तंत्र पर ऊपरी अपसरर् (Upper divergence)।

Q 38.B
• मानि षिकास ररपोटष (The Human Development Index: HDR), संयक्त
ु राष्ट्र षिकास कायषक्रम (UNDP) के मानि
षिकास ररपोटष कायाषलय द्वारा प्रकाषित एक िार्िषक ररपोटष है।

16 www.visionias.in ©Vision IAS

Google it:- https://upscpdf.com


https://t.me/UPSC_PDF Download From > https://upscpdf.com https://t.me/UPSC_PDF

• निीनतम मानि षिकास सूचकांक में भारत को 189 देिों में से 130िां थथान प्राप्त हुआ है। दषक्षर् एषिया में, भारत का HDI
मान इस क्षेत्र के औसत मान 0.638 से अषधक है।
• इस ररपोटष को पहली बार ििष 1990 में पाककथतानी अथषिास्त्री महबूब उल हक़ तथा भारतीय नोबेल पुरथकार षििेता अमत्यष
सेन के द्वारा िारी ककया गया था। इसका लक्ष्य आर्थषक षिचार-षिमिष, नीषत तथा पक्षसमथषन के संदभष में लोगों को षिकास
प्रकक्रया के कें र्द् में समाषिष्ट करना था। ििष 1990 में िारी प्रथम HDR ने आर्थषक षिकास-के षन्र्द्त प्रषतमान को एक ऐसे
प्रषतमान की ओर थथानांतररत ककया, षिसके अंतगषत लोगों को िीिनयापन हेतु िे अिसर तथा थितंत्रताएं प्रदान करना िाषमल
था, िो उनके षिकास में सहायक होंगे।
• ररपोटष में प्रकाषित HDI िीिन प्रत्यािा, षिक्षा तथा प्रषत व्यषक्त आय सूचकों का एक संयुक्त सांषख्यकीय सूचकांक है, षिसका
उपयोग देिों को मानि षिकास की चार िेषर्यों में रैं क प्रदान करने हेतु ककया िाता है।
• HDI-2010 में “असमानता समायोषित मानि षिकास सूचकांक (Inequality adjusted Human Development Index:
IHDI)” िारी ककया गया था।

Q 39.B
• हाल ही में, भारत सरकार ने आर्द्रभूषष मयों के पुनथथाषपन तथा उनका पुनरुद्धार करने के षलए संपर्
ू ष देि में 100 मुख्य आर्द्षभषू मयों
को षचषन्हत ककया है। साथ ही इन सभी आर्द्षभषू मयों के षलए समेककत प्रबंधन योिनाओं को तैयार करने का कायष भी आरमभ
ककया गया है।
• देि की अंतररक्ष एिेंसी इसरो (ISRO) ने ििष 2011 में उपग्रह से प्राप्त षचत्रों के आधार पर “नेिनल िेटलैंड्स एटलस” का
प्रकािन ककया था, षिसमें भारत के संपूर्ष भौगोषलक क्षेत्र के लगभग 4.63% क्षेत्र पर षिथतृत 2 लाख से अषधक आर्द्रभूषषमयों का
मानषचत्रर् ककया गया था। िथतुतः आर्द्षभूषमयां ऐसे भू-क्षेत्र हैं, िो या तो अथथायी रूप से/मौसम षििेि में या थथायी रूप से
िल से आिृत रहते हैं। इसके अंतगषत दलदल, पंकभूषम और पीटभूषम िाषमल हैं। इसषलए कथन 1 सही है।
• सरकार द्वारा भी यह अनुभि ककया गया है कक आर्द्षभूषमयों तथा अन्य िलाियों को संरषक्षत करने का सबसे बेहतर तरीका
समुदायों को इनसे संबद्ध करना है। इसषलए मंत्रालय ने ‘आर्द्षभषू म षमत्रों (Wetland Mitras)’ के गठन का षनर्षय ककया है।
ज्ञातव्य है कक “आर्द्षभषू म षमत्र” संपूर्ष देि में षचषननत आर्द्षभूषमयों की षनगरानी करने हेतु थि-प्रेररत व्यषक्तयों का एक समूह है।
इसषलए कथन 2 सही नहीं है।
• कथन 3 सही है: बहुषिध मापदंडों के आधार पर आर्द्षभूषमयों के थिाथथ्य की षनगरानी करने हेतु “आर्द्षभषू म थिाथथ्य काडष
(Wetland Health Card)” की अिधारर्ा प्रथतुत की गई है।
Q 40.D
• युग्म 1 सुमषे लत नहीं है: पषिम बंगाल तथा भूटान की सीमा के षनकट पषिमी असम में षथथत ररपु और षचरांग दो संलि
पयाषिास िाले आरषक्षत िन क्षेत्र हैं। ये मानस टाइगर ररज़िष के बफर क्षेत्र के भाग भी हैं। ररपु तथा षचरांग ऐसे महत्िपूर्ष
पयाषिास हैं, िो असम के मानस राष्ट्रीय उद्यान, पषिम बंगाल के बुक्सा टाइगर ररििष (या बक्सा) तथा भूटान के िनों {षििेि
रूप से एषियाई हाषथयों (एलफस मैषक्समस) हेतु महत्िपूर्ष िन} को परथपर िोडने हेतु एक गषलयारे के रूप में कायष करते हैं।
यह भारत का एक MIKE (मॉषनटररं ग द इललीगल ककसलंग ऑफ एलेफन्ट्स) थथल है।
• युग्म 2 सुमषे लत नहीं है: कदपोर षबल को दीपोर बील के रूप में भी उच्चाररत ककया िाता है। यह असम में गुिाहाटी िहर
(कामरूप षिले) के दषक्षर्-पषिम में षथथत है। यह एक तािे िल की थथायी झील है और ब्रह्मपुत्र नदी की मुख्य धारा के दषक्षर्
में ब्रह्मपुत्र की पूिषिती िाषहका में षथथत है। यह असम का एक िन्य िीि अभयारण्य है तथा ििष 2002 में इसे एक महत्िपूर्ष
रामसर थथल घोषित ककया गया था।
• युग्म 3 सही सुमषे लत है: काज़ीरंगा, असम में ब्रह्मपुत्र नदी के दषक्षर्ी तट के बाढ़कृ त मैदानों में अिषथथत है। ज्ञातव्य है कक ििष
1985 में यूनेथको (UNESCO) द्वारा इसे इसके षिषिष्ट प्राकृ षतक पयाषिरर् के षलए षिश्व षिरासत थथल घोषित ककया गया
था। ज्ञातव्य है कक ििष 1908 में इसे सिषप्रथम गैंडों की घटती संख्या की रोकथाम हेतु एक आरषक्षत िन क्षेत्र के रूप में थथाषपत
ककया गया था।

17 www.visionias.in ©Vision IAS

Google it:- https://upscpdf.com


https://t.me/UPSC_PDF Download From > https://upscpdf.com https://t.me/UPSC_PDF

Q 41.D
• लिर्ीय मृदा को ऊसर मृदा भी कहा िाता है। लिर्ीय मृदा में बडी मात्रा में सोषडयम, पोटेषियम और मैिीषियम पाया िाता
है। इसषलए यह अनुिरष होती है तथा ककसी भी िनथपषत के षिकास के षलए उपयुक्त नहीं होती है।
• िुष्क िलिायु और खराब अपिाह के कारर् इसमें लिर् की मात्रा अषधक होती है। यह मृदा िुष्क और अद्धष-िुष्क क्षेत्रों तथा
िलाक्रांत क्षेत्रों और अनूपों में पायी िाती है।
• इनकी संरचना बलुई से लेकर दोमट तक होती है तथा इनमें नाइिोिन एिं कै षलसयम की कमी होती है। लिर्ीय मृदा का
अषधकतर प्रसार गुिरात, पूिी तट के डेलटा क्षेत्रों और पषिम बंगाल के सुंदरबन क्षेत्रों में पाया िाता है।
• कच्छ के रर् में, दषक्षर्-पषिम मानसून लिर्ीय कर्ों को पररिषहत करता है, िो िहां एक पपषटी के रूप में ऊपरी सतह पर
षनक्षेषपत हो िाते हैं। इसषलए षिकलप 1 सही है।
• डेलटा क्षेत्रों में समुर्द्ी िल का प्रिेि भी लिर्ीय मृदा के षनमाषर् को प्रोत्साषहत करता है। इसषलए षिकलप 2 सही है।
• अत्यषधक ससंचाई िाले गहन कृ षि क्षेत्रों, षििेितः हररत क्रांषत िाले क्षेत्रों में उिषर िलोढ़ मृदा भी लिर्ीय हो रही है। िुष्क
िलिायु दिाओं में अत्यषधक ससंचाई के कारर् के षिका कक्रया (capillary action) को बढ़ािा षमलता है षिसके पररर्ामथिरूप
मृदा की ऊपरी परत पर लिर् का िमाि होने लगता है। ऐसे क्षेत्रों में, षििेितः पंिाब और हररयार्ा में, ककसानों को मृदा की
लिर्ता की समथया का समाधान करने के षलए षिप्सम के प्रयोग का परामिष कदया िाता है। इसषलए षिकलप 3 सही है।

Q 42.B
• भारत षिश्व के उन चुसनंदा प्रथम देिों में से एक है िहां षिषभन्न क्षेत्रकों में िीतलन की मांग को पूरा करने हेतु दीघषकाषलक
दृषष्टकोर् के साथ एक व्यापक िीतलन कारष िाई योिना (कू सलंग एक्िन प्लान) का षिकास ककया गया है। इसमें उन कायों की
सूची तैयार की गई है षिससे िीतलन संबंधी मांग को कम करने में सहायता प्राप्त हो सकती है। भारतीय िीतलन कारष िाई में
षनम्नषलषखत लक्ष्य षनधाषररत ककए गए हैं:
o ििष 2037-38 तक षिषभन्न क्षेत्रों में िीतलक मांग (Cooling Demand ) को 20% से 25% तक कम करना।
o ििष 2037-38 तक प्रिीतक मांग (Refrigerant Demand) को 25% से 30% तक कम करना। इसषलए कथन 1 सही
नहीं है।
o ििष 2037-38 तक िीतलन हेतु ऊिाष आिश्यकता को 25% से 40% तक कम करना।
o राष्ट्रीय S&T कायषक्रम के अंतगषत “िीतलन तथा संबंषधत क्षेत्रों” को महत्िपूर्ष क्षेत्रों (thrust area) के रूप में पहचान
प्रदान करना।
o ििष 2022-23 तक कौिल भारत षमिन के साथ समन्िय थथाषपत करके सेिा क्षेत्रक के 100,000 तकनीषियनों को
प्रषिक्षर् और प्रमार्-पत्र उपलब्ध कराना। इसषलए कथन 2 सही है।
• भारतीय िीतलन कारष िाई योिना (India Cooling Action Plan: ICAP) का मुख्य बल पयाषिरर्ीय और सामाषिक-
आर्थषक लाभ प्राप्त करने हेतु प्रारंभ प्रयासों में समन्िय थथाषपत करना है। ICAP का महत्िपूर्ष लक्ष्य समाि के षलए
पयाषिरर्ीय और सामाषिक-आर्थषक लाभ सुषनषित करते हुए सभी के षलए संधारर्ीय िीतलन और उष्मीय राहत प्रदान
करना है। यह प्रत्यक्ष तथा अप्रत्यक्ष उत्सिषनों को कम करने में सहायक होगा।
• इन कदमों से िलिायु संबंधी महत्िपूर्ष लाभ प्राप्त होंगे। पयाषिरर् संबंधी लाभों के अषतररक्त समाि षनम्न प्रकार से भी
लाभाषन्ित होगा:
o सभी को उष्मीय राहत (Thermal comfort )– आर्थषक रूप से कमिोर िगष (EWS) और षनम्न-आय समूह (LIG) के
आिासों के िीतलन का प्रािधान।
o संधारर्ीय िीतलन – िीतलन से उत्पन्न GHG उत्सिषन में कमी।
o ककसानों की आय को दोगुना करना– बेहतर कोलड चेन इं फ्राथिक्चर, ककसानों को उनके उत्पाद के बेहतर मूलय, कृ षि
उत्पादों की क्षषत में कमी के माध्यम से।
o बेहतर आिीषिका और पयाषिरर्ीय सुरक्षा के षलए कु िल िमिषक्त।
o मेक-इन-इं षडया– एयर-कं डीिसनंग और संबंषधत िीतलन उपकरर्ों का देि में ही षिषनमाषर्।
o िैकषलपक िीतलन प्रौद्योषगकी हेतु अनुसंधान एिं षिकास को बढ़ािा देना– इसका लक्ष्य िीतलन क्षेत्रक में निोन्मेि में
तीव्रता लाना है।
18 www.visionias.in ©Vision IAS

Google it:- https://upscpdf.com


https://t.me/UPSC_PDF Download From > https://upscpdf.com https://t.me/UPSC_PDF

Q 43.C
• कथन 1 सही है: राष्ट्रीय िलिायु पररितषन अनुकूलन षनषध (National Adaptation Fund for Climate Change:
NAFCC) कें र्द्ीय क्षेत्रक योिना है। इसे ििष 2015-16 में थथाषपत ककया गया था। NAFCC का समग्र उद्देश्य िलिायु
पररितषन के प्रषतकू ल प्रभािों को कम करने िाली ठोस अनुकूलन गषतषिषधयों को सहायता प्रदान करना है। इस योिना के
अंतगषत गषतषिषधयों को एक पररयोिना मोड में कायाषषन्ित ककया िाता है। कृ षि, पिुपालन, िल, िाषनकी, पयषटन आकद क्षेत्रों
में अनुकूलन से संबंषधत पररयोिनाएं NAFCC के अंतगषत षित्त पोिर् की पात्र हैं।
• कथन 2 सही है: NABARD को, क्योटो प्रोटोकॉल के अंतगषत अनुकूलन कोि (Adaptation Fund: AF) के षलए राष्ट्रीय
कक्रयान्ियन इकाई (NIE) के रूप में थथाषपत होने और देि भर में इसकी उपषथथषत के कारर्, NAFCC के अंतगषत अनुकूलन
पररयोिनाओं के कायाषन्ियन के षलए राष्ट्रीय कायाषन्ियन इकाई (NIE) के रूप में नाषमत ककया गया है। इस व्यिथथा के अंतगषत,
NABARD िलिायु पररितषन के षलए राज्य कायष योिना (SAPCC) से पररयोिना षिचारों/अिधारर्ाओं की पहचान करने
की सुषिधा प्रदान करने, पररयोिना षनमाषर्, मूलयांकन, अनुमोदन, षनषध के संषितरर्, षनगरानी एिं मूलयांकन तथा राज्य
सरकारों सषहत षहतधारकों की क्षमता षनमाषर् में सहायता प्रदान करता है।

Q 44.B
• िनिरी 2020 में षबहार के भागलपुर िन प्रभाग में अलिर्ीय िल के कछु ओं के षलए अपनी तरह के प्रथम पुनिाषस कें र्द् का
उद्घाटन ककया िाएगा। आधे हेक्टेयर में षिथतृत, यह पुनिाषस कें र्द् एक बार में 500 कछु ओं को आिय देने में सक्षम होगा। िैकफक
इं षडया के हाषलया अध्ययन के अनुसार, भारत में प्रषत ििष लगभग 11,000 कछु ओं की तथकरी हो रही है।
• कथन 1 सही है: पयाषिरर्षिदों के अनुसार, कछु ए मृत काबषषनक पदाथों और रोगग्रथत मछषलयों का भक्षर् करके , षिकाररयों के
रूप में मत्थय आबादी को षनयंषत्रत करके और िलीय पौधों एिं खरपतिारों को षनयंषत्रत करके नकदयों में महत्िपूर्ष भूषमका
षनभाते हैं। इन्हें थिथथ िलीय पाररषथथषतकी प्रर्ाषलयों के संकेतक के रूप में भी िाना िाता है। हालांकक ितषमान में इन
प्रिाषतयों के समक्ष बांधों और बैरािों के षनमाषर्, प्रदूिर्, अिैध षिकार, मछली पकडने के िाल से दुघषटनािि फं सने और अपने
नेसथटंग पयाषिास के ह्रास के कारर् गंभीर खतरा उत्पन्न हो गया है।
• कथन 3 सही है: कछु ओं के समक्ष मुख्य रूप से दो कारर्ों - भोिन और पालतू िंतओं
ु के रूप में उपयोग हेतु बढ़ते व्यापार - से
गंभीर खतरा उत्पन्न हो गया है। इस प्रचषलत षिश्वास के कारर् कछु ओं को प्राय: मांस के षलए लषक्षत ककया िाता है कक यह
अत्यषधक ऊिाष और षिषभन्न रोगों से सुरक्षा प्रदान करता है। सामान्यत: मुलायम-किच िाले कछु ए इस धारर्ा का षिकार हैं।
िहीं दूसरी ओर, कठोर-किच िाले कछु ओं, षििेि रूप से थपॉटेड, का अिैध षिकार पालतू िंतुओं के रूप में उपयोग हेतु व्यापार
के षलए ककया िा रहा है। ऐसे कछु ओं की दषक्षर्-पूिष एषिया, चीन और िापान में अत्यषधक मांग है।
• कथन 2 सही नहीं है: भारत अलिर्ीय िल के कछु ओं की 24 प्रिाषतयों और कच्छपों की 4 प्रिाषतयों का आियथथल है।
पूिोत्तर और उत्तर भारत के क्षेत्रों को षिश्ि में िीिष तीन कच्छप िैि षिषिधता आकिषर् थथलों में थथान प्रदान ककया गया है।
भारतीय तटीय िल और द्वीपों पर समुर्द्ी कछु ओं की के िल पााँच प्रिाषतयां पायी िाती हैं यथा- ऑषलि ररडले
(लेषपडोके लाईसोषलिैषसया), ग्रीन (के लोषनयामाइडस), हॉक्सषबल (एररटमोके लाइषसषमब्रकाटा), लॉगरहेड (के यरट्टैकरे ट्टा) और
लेदरबैक (डमोके लाईथकोररषसया) कछु ए हैं।
Q 45.D
• मरुथथल एक अनुिरष क्षेत्र है, िहां अत्यंत कम ििाष होती है और इसके पररर्ामथिरूप, पादप और िंतओं
ु के उद्भि एिं षिकास
के षलए यहां पररषथथषतयां प्रषतकू ल होती हैं। दूसरे िब्दों में, यह अत्यंत िुष्क क्षेत्र होता है िहााँ िनथपषत अत्यंत षिरल होती हैं।
• महाद्वीपों के पषिमी भागों में गमष और उष्र्करटबंधीय मरुथथलों की उपषथथषत के कारर् षनम्नषलषखत हैं:
• ऊष्र्करटबंधीय पूिी पिनों की उपषथथषत
o षिश्व के अषधकांि मरुथथल उपोष्र् करटबंधीय क्षेत्रों में महाद्वीपों के पषिमी भाग में षिद्यमान हैं , क्योंकक उष्र्करटबंधीय
क्षेत्रों में प्रचषलत पिनें उष्र्करटबंधीय पूिी पिनें हैं।
o िब आर्द्षतायुक्त व्यापाररक पिनें पूिष से पषिम की ओर प्रिाषहत होती हैं, तो पूिी भाग में ििषर् होने से उनकी आर्द्षता में
कमी हो िाती है और महाद्वीप के पषिमी भागों में पहुाँचने तक िुष्क हो िाती है इसषलए िहां ििषर् नहीं होता है। इस
प्रकार, िह क्षेत्र आर्द्षता से िंषचत हो िाता है, षिससे सूखे की षथथषत उत्पन्न होती है, िो मरुथथल षनमाषर् का कारर् है।

19 www.visionias.in ©Vision IAS

Google it:- https://upscpdf.com


https://t.me/UPSC_PDF Download From > https://upscpdf.com https://t.me/UPSC_PDF

• प्रषतचक्रिातीय पररषथथषतयां (अश्व अक्षांि का क्षेत्र) :


o महाद्वीपों के पषिमी ककनारे पर 20°-30° अक्षांि के मध्य का क्षेत्र िायु के अितलन का क्षेत्र होता है। इसके कारर्, िायु
अितलन से गमष और संपीषडत हो िाती है और इसमें आर्द्षता की कमी होती है।
• िृषष्ट-छाया प्रदेि का षनमाषर्:
o पिषतों का पिनषिमुख क्षेत्र, िहां पिनषभमुख पिषतीय क्षेत्रों की तुलना में कम ििाष होती है, िृषष्ट-छाया प्रदेि कहलाता है।
उदाहरर् के षलए, थार मरुथथल भारत में िृषष्ट-छाया प्रदेि के कारर् षनर्मषत हुआ है क्योंकक यह अरािली पिषतीय क्षेत्र के
समानांतर षिथताररत है। इसषलए यहां पिषतों द्वारा अिरोध के रूप में कायष नहीं करने के कारर् आर्द्षतायुक्त पिनें इस क्षेत्र
में षबना ििाष ककए आगे प्रिाषहत हो िाती हैं।
• ठं डी महासागरीय धाराओं की उपषथथषत
o महाद्वीपों के पषिमी तटों से संलि ठं डी महासागरीय धाराओं में तट पर प्रिाषहत िायु को षथथर करने की प्रिृषत्त होती है।
यह मेघ षनमाषर् और ििषर् को प्रषतबंषधत करती हैं। इसषलए, तटीय क्षेत्रों के षनकट िुष्क क्षेत्र या समुर्द्ी मरुथथल का
षनमाषर् हो िाता है।
o इसके उदाहरर् हैं- उत्तरी अमेररका का सोनोरन मरुथथल, दषक्षर् अमेररका के पेरू और अटाकामा मरुथथल और अफ्रीका के
सहारा (मोरक्को का भाग) और नामीब मरुथथल।
Q 46.D
• हाल ही में मषहला एिं बाल षिकास मंत्रालय ने षबल एंड मेसलंडा गेट्स फाउं डेिन (BMGF) के सहयोग से भारतीय पोिर्
कृ षि कोि (BPKK) की थथापना की घोिर्ा की गई है। यह बेहतर पोिर् पररर्ामों हेतु भारत के 127 कृ षि-िलिायु क्षेत्र में
उगाई िाने िाली षिषिध फसलों का एक संग्रह होगा। भारतीय पोिर् कृ षि कोि पररयोिना के षनम्नषलषखत दो घटक हैं यथा-:
• एक खाद्य एटलस का षिकास
o कृ षि-खाद्य एटलस देि के 127 कृ षि-िलिायु क्षेत्रों की षिषभन्न फसलों के एक कोि के रूप में कायष करे गा। इस एटलस के
तीन भाग होंगे यथा- ितषमान में उगाई िा रही फसलें,कृ षि-पाररषथथषतकीय दिाएं (मृदा, िैषिक काबषन सामग्री, भूिल की
उपलब्धता आकद) तथा पोिर् एिं आहार संबंधी षिषिधता को प्रोत्साषहत करने हेतु षिले या प्रखंड षििेि में फसलों की
व्यापक षिषिधता में िृषद्ध करने संबधी कदिा-षनदेि।
• पोिर् (POSHAN) अषभयान हेतु िन-आंदोलन के षलए बेहतर प्रथाओं का अषभलेखन
o इसमें ऐसी सामाषिक, व्यािहाररक और सांथकृ षतक प्रथाओं को अषभलेषखत ककया िाएगा, िो थिाथथ्यप्रद आहार संबंधी
व्यिहारों को प्रोत्साषहत एिं सुदढ़ृ करें गी। षििेितया संबद्ध क्षेत्रों में षनिाषसत िनसंख्या समूह हेतु सामाषिक और
व्यिहारिादी पररितषन संचार के षलए उपयुक्त सिषिेष्ठ रर्नीषतयों के प्रचार-प्रसार हेतु बहु-षिियक षििेिज्ञों के समूह
तथा एक साधन तंत्र के षिकास की सहायता से सिोत्तम प्रथाओं की पहचान करना भी इस घटक का एक भाग है।
• इसषलए षिकलप (d) सही उत्तर है।
Q 47.A
• अरािली पिषत िृंखला षिश्व की प्रमुख पिषत िृंखलाओं और प्राचीनतम िषलत पिषत िृंखलाओं में से एक है। यह उत्तर-पूिष से
दषक्षर्-पषिम की ओर लगभग 300 मील तक षिथतृत है।
• अरािली पिषत िृंखला दषक्षर्-पषिम में गुिरात से उत्तर-पूिष में कदलली तक लगभग 700 ककमी तक षिथताररत है। इसके साथ
ही यह पूिी रािथथान ि हररयार्ा एिं कदलली के संलि क्षेत्रों तक षिषभन्न थथलाकृ षतयों का षनमाषर् करती है और समृद्ध
बषथतयों के षलए रमर्ीय पृष्ठभूषम का सृिन करती है। रािथथान के माउं ट आबू में षथथत गुरुषिखर इस िृख ं ला का सिोच्च
षिखर है। इसषलए कथन 1 और 2 सही नहीं हैं।
• अरािली पिषतमाला की षििेिताएं:
o यह िुष्क पर्षपाती िनों से आच्छाकदत है।
o यह पादप षिषिधता में समृद्ध है तथा यहां कराया (औिधीय गोंद प्रदान करता है), गुग्गल, आंिला, मूसली, खैर, सलई,
मोदाद, धािडा, खाखरा, रटमरू आकद िैसे दुलभ
ष औिधीय पादप पाए िाते हैं।
o अरािली िृंखला प्राकृ षतक संसाधनों की दृषष्ट से अत्यंत समृद्ध है और यहां अनेक प्रायद्वीपीय नकदयों िैसे बनास, लूनी, सखी
और साबरमती का उद्गम होता है।
o मालिा का पठार अरािली और सिंध्याचल पिषत िृख
ं लाओं के मध्य अिषथथत है। इसषलए कथन 3 सही है।

20 www.visionias.in ©Vision IAS

Google it:- https://upscpdf.com


https://t.me/UPSC_PDF Download From > https://upscpdf.com https://t.me/UPSC_PDF

Q 48.C
• कथन 1 सही है: ग्रीष्मषनषष्क्रयता अथिा ऐषथटिेिन (Aestivation), िीतषनषष्क्रयता (hibernation) के ही समान िंतओं
ु की
प्रसुप्तािथथा में िाने की षथथषत है, हालांकक यह कक्रया सर्दषयों की बिाय गर्मषयों में संपन्न होती है। ग्रीष्मषनषष्क्रयता की प्रमुख
षििेिताएं षनषष्क्रय हो िाना और उपापचय दर का घट िाना हैं। अत: िंतु उच्च तापमान तथा िुष्क पररषथथषतयों के कारर् इस
अनुकूलन षिषध को अपनाते हैं।
• कथन 2 सही नहीं है: व्यािहाररक अनुकूलन पादपों और िंतुओं, दोनों में पररलषक्षत होता है। पादप अनुकूलन संरचनागत,
व्यिहाररक या कक्रयात्मक हो सकते हैं। पादपों के व्यािहाररक अनुकूलन उन्हें लाभ पहुंचाने िाले व्यिहार होते हैं। सभी पादपों
के प्ररोह (shoots) प्रकाि संश्लि
े र् की प्रकक्रया को तीव्र करने हेतु प्रकाि की उपषथथषत में तेिी से िृषद्ध करते हैं। प्रकाि की
षिद्यमानता में षिकास और अन्य अनुितषन यह सुषनषित करते हैं कक पादप अपने पयाषिरर् में होने िाले पररितषनों के प्रषत
अनुकक्रया कर सकते हैं। पादप मूल संभितया गुरुत्िाकिषर् के कारर् नीचे की ओर बढ़ते हैं या प्रकाि संश्लि
े र् की कक्रया को
अषधक करने हेतु प्रत्यक्षत: िल की कदिा में िृषद्ध करती हैं। अन्य पादप, िैसे िीनस फ्लाईिैप ने कीटों के भक्षर् हेतु संरचनात्मक
और व्यािहाररक अनुकूलन षिकषसत ककया है। फ्लाईिैप एक संरचनात्मक अनुकूलन िबकक कीट पकडने के षलए िैप का बंद
होना एक व्यािहाररक अनुकूलन है।
• कथन 3 सही है: िलीय पादपों में भी अनुकूलन होता है। उदाहरर् के षलए, समुर्द्ी खरपतिार (seaweed) एक िलीय पादप
है। यह िलमि अिथथा में षिकषसत होने हेतु अनुकूषलत है। इस पादप के प्रत्येक पर्ष में उसके थियं के िात युक्त बुलबुले होते हैं,
िो िल से पादप में ऑक्सीिन के षिषनमय हेतु आिश्यक माध्यम प्रदान करते हैं। इससे समुर्द्ी खरपतिार लंबित अिथथा में
बने रहते हैं। िलमि िलीय पादपों की पषत्तयां भूषम पर पललषित होने िाले पादपों की पषत्तयों की तुलना में अषधक मुलायम
होती हैं। इस गुर् से िलीय पादप लहर के अनुरूप टू टे षबना मुडने हेतु अनुकूषलत हो िाते हैं।

Q 49.B
• करे ि प्रर्ाली (Karez system)
o बीदर की एक सिाषषधक अषद्वतीय षििेिता ऐषतहाषसक ‘करे ि’ प्रर्ाली है (षिसे कनत भी कहा िाता है)। यह एक िल
दोहन प्रर्ाली है। इसकी उत्पषत्त ईरान (तत्कालीन पर्सषया या फारस) में हुई थी। इसे कनाषटक के बीदर, गुलबगाष और
बीिापुर में 15िीं िताब्दी ई. में बहमनी सुलतानों द्वारा षनर्मषत ककया गया था।
o ये भौमिल धाराओं हेतु षनर्मषत भूषमगत नहरें हैं, िो अषधिाषसत क्षेत्रों और बीदर के ककले के भीतर पेय िल की आपूर्तष
करती थीं। यह प्रर्ाली बीदर िैसे नगरों के षलए आिश्यक थी, क्योंकक यहां मृदा के चट्टानी होने के कारर् पेय िल संचय
हेतु कू प षनमाषर् करठन था। इसषलए षिकलप (b) सही उत्तर है।
o हाल ही में, बीिापुर, कनाषटक की सुरंगा बािडी (िो प्राचीन करे ि प्रर्ाली का एक अषभन्न भाग है) को षिश्व थमारक कोि
के “दक्कन पठार की प्राचीन िल प्रर्ाली” िगष के तहत षिश्व थमारक षनगरानी सूची में सूचीबद्ध ककया गया है।
• अहार पायने
o यह दषक्षर् षबहार की एक देिि पारं पररक बाढ़ िल संग्रहर् प्रर्ाली है तथा इस क्षेत्र में ससंचाई का सबसे महत्िपूर्ष स्रोत
रही है। अहार ऐसे िलािय होते हैं षिनके तीन पाश्वों पर तटबंध षनर्मषत होते हैं और इनका षनमाषर् छोटी नकदयों या
पायने िैसी कृ षत्रम धाराओं के अंषतम छोर पर ककया िाता है।
o पायने ससंचाई के प्रयोिनाथष और अहार में िल को संगृहीत करने हतु नदी से षनकाली गई व्यपितषन िाषहकाएं होती हैं।
उललेखनीय है कक दषक्षर् षबहार में धान की कृ षि इन्हीं िल संग्रह प्रर्ाषलयों के कारर् ही संभि हुई है , क्योंकक यहां उत्तरी
षबहार की तुलना में अलप ििाष होती है। इस प्रर्ाली का सिाषषधक षिकास गया षिले में हुआ है।
• खडीन
o इसे ‘धोरा’ भी कहते हैं, यह कृ षि हेतु धरातलीय िलप्रिाह के संग्रहर् के षलए अषभकषलपत एक देिि संरचना है। इसकी
मुख्य षििेिता इसके पथरीली उच्च भूषमयों के षनम्निती क्षेत्रों में षनचली पहाषडयों की ढलानों पर षमट्टी से षनर्मषत
दीघाषकार (100-300 मीटर) तटबंध हैं। अषतररक्त िल संकरे िलषनकासी मागों और अषधप्लि मागों से षनष्काषसत हो
िाता है। यह कृ षि भूषम पर ही ििाष िल संग्रहर् तथा फसल उत्पादन हेतु इस िल-संतप्त
ृ भूषम के उत्तरिती उपयोग के
षसद्धांत पर आधाररत है।

21 www.visionias.in ©Vision IAS

Google it:- https://upscpdf.com


https://t.me/UPSC_PDF Download From > https://upscpdf.com https://t.me/UPSC_PDF

o इसे प्रथमतः 15िीं िताब्दी ई. में पषिमी रािथथान के िैसलमेर में पालीिाल ब्राह्मर्ों द्वारा षनर्मषत ककया गया था। यह
प्रर्ाली अषधकांितः 4500 ईथिी पूिष में उर (ितषमान ईराक में) के षनिाषसयों की ससंचाई प्रर्ाली और पिात्िती मध्य-
पूिष के नेबेषतयनों की ससंचाई प्रर्ाली के समान है। ऐसी ही एक प्रर्ाली के 4000 ििष पूिष नेगि
े मरुथथल और 500 ििष पूिष
दषक्षर्-पषिमी कोलोरॅ डो में भी प्रचषलत होने के साक्ष्य प्राप्त हुए हैं।
• एरी
o तषमलनाडु राज्य की यह प्रर्ाली भारत की सिाषषधक प्राचीन िल प्रबंधन प्रर्ाषलयों में से एक है। ितषमान में भी राज्य में
इसका व्यापक रूप से उपयोग ककया िाता है। यह अत्यषधक ििाष के दौरान बाढ़ षनयंत्रर् प्रर्ाली के रूप में कायष करती है,
मृदा अपरदन और ििाष िल के अपव्यय को रोकती है तथा भूषमगत िल का पुनभषरर् करती है।

Q 50.C
• कनारी धारा (ठं डी धारा)
o कनारी धारा उत्तरी अटलांरटक महासागर में दषक्षर्िती कदिा की ओर प्रिाषहत होने िाली महासागरीय धारा प्रर्ाली का
एक भाग है।
o यह उत्तरी अटलांरटक धारा से दषक्षर् की ओर षिभाषित होकर अफ्रीका के उत्तर-पषिमी तट के साथ-साथ दषक्षर्-पषिम
की ओर प्रिाषहत होती है। अंततः अटलांरटक की उत्तरी षििुितरे खीय धारा में समायोषित होने हेतु पषिम की ओर
षिक्षेषपत होने से पूिष दषक्षर् में सेनेगल तक प्रिाषहत होती है।
o महाद्वीप की अपतटीय पिनों के कारर् महासागरीय िल के उद्वेलन के प्रभािाधीन िल का तापमान ठं डा हो िाता है।
चूंकक यह धारा कनारी द्वीप के चतुर्दषक प्रिाषहत होती है, अत: यह पूिष की ओर सहारा के ऊष्मीय प्रभाि को कम करने में
सहायक होती है। तापीय षमिर् इस क्षेत्र को उत्कृ ष्ट मत्थयन क्षेत्र के रूप में षिकषसत करता है।
• फॉकलैंड धारा (ठं डी धारा)
o अंटाकष रटक सागर का ठं डा िल फॉकलैंड धारा के रूप में दषक्षर् अमेररका के पूिी तट के साथ-साथ दषक्षर् से उत्तर की ओर
अिेंटीना तक प्रिाषहत होता है।
o यह धारा 30° दषक्षर्ी अक्षांि के षनकट सिाषषधक षिथताररत और षिकषसत हो िाती है।
o यह धारा अंटाकष रटक क्षेत्र से दषक्षर् अमेररकी तट तक अनेक षहमिैलखंड लाती है।
• क्यूरोषििो धारा (गमष धारा)
o इसे कु रुषसिो या िापानी धारा भी कहा िाता है। इसका अषधकांि भाग उपोष्र्करटबंधीय उच्च दाब पट्टी में और पछु आ
पिनों के प्रभािाधीन प्रिाषहत होता है। षििुितरे खीय िल धारर् करने के कारर् यह एक गमष िलधारा है।
• ओयाषियो धारा (ठं डी धारा)
o इसे ओयाषसिो, ओखोटथक या क्यूराइल धारा भी कहा िाता है। यह ठं डी उप-आकष रटक धारा है, िो बेररं ग िलसषन्ध से
प्रिाषहत होती है। यह िामाितष कदिा में दषक्षर् की ओर प्रिाषहत होती है और िापान के होकै डो द्वीप के समीप क्यूरोषििो
धारा से षमल िाती है।
Q 51.D
• गोदािरी नदी भारत की एक महत्िपूर्ष नदी है और यह बंगाल की खाडी में अपिाषहत होने से पूिष पषिमी भारत से दषक्षर्ी
भारत तक प्रिाषहत होती है। इस नदी के िलग्रहर् क्षेत्र को देि के सबसे बडे िलग्रहर् क्षेत्रों में से एक माना िाता है।
• इस नदी का उद्गम महाराष्ट्र राज्य के नाषसक षिले में पषिमी घाट में षथथत त्र्यब ं क (त्र्यब ं के श्वर) नामक थथान से होता है।
इसषलए कथन 1 सही है।
• इसकी आयु, आकार और लंबाई के कारर् इसे दषक्षर् गंगा या िृद्ध गंगा (बूढ़ी गंगा) कहा िाता है। इसके डेलटाई क्षेत्र में ही
नौसंचालन संभि है।
• इस नदी की लमबाई 1,465 ककमी है और यह गंगा के बाद देि की दूसरी सबसे लंबी नदी है। नदी का अपिाह बेषसन भारत के
सात राज्यों अथाषत छत्तीसगढ़, महाराष्ट्र, आंध्र प्रदेि, तेलग
ं ाना, मध्य प्रदेि, कनाषटक और उडीसा तथा एक कें र्द् िाषसत प्रदेि
पुडुचरे ी में षिथतृत है। इसषलए कथन 2 सही है।
• गोदािरी नदी की प्रमुख सहायक नकदयां इं र्द्ािती, प्रार्षहता, पैनगंगा, पूर्ाष, दुहना, मंिरा, िधाष, िैनगंगा, सबरी आकद हैं।
• पोलािरम पररयोिना आंध्र प्रदेि के पषिमी गोदािरी षिले और पूिी गोदािरी षिले में गोदािरी नदी पर षथथत एक
षनमाषर्ाधीन बहुउद्देश्यीय राष्ट्रीय पररयोिना है। इसषलए कथन 3 सही है।

22 www.visionias.in ©Vision IAS

Google it:- https://upscpdf.com


https://t.me/UPSC_PDF Download From > https://upscpdf.com https://t.me/UPSC_PDF

• ििष 2014 में सरकार ने पोलािरम पररयोिना को एक राष्ट्रीय पररयोिना घोषित कर कदया और मंत्रालय ने षनमाषर् कायों को
अनुमषत देने के षलए ‘काम रोकने के आदेि (Stop Work Order)’ [िब तक अनुबंषधत पक्षों के मध्य कोई समझौता नहीं हो
िाता है या कोई षनर्षय नहीं ले षलया िाता, तब तक कायष को थथषगत रखने की षलए षनमाषर् उद्योग में उपयोग ककया िाने
िाला कानूनी उपकरर्] को प्राथथषगत बनाए रखा। इसे देि में िल के अभाि को दूर करने के षलए षडज़ाइन ककया गया है। यह
पररयोिना पूर्त
ष ः कें र्द् द्वारा षित्त पोषित है और राष्ट्रीय नदी-िोडो पररयोिना का एक भाग है।
• िून 2019 में, कें र्द्ीय पयाषिरर् मंत्रालय ने पोलािरम बहुउद्देिीय पररयोिना से संबंषधत षनमाषर् कायष की अिषध दो ििष बढ़ा
दी और इस पररयोिना से संबंषधत षनमाषर् कायों को अनुमषत प्रदान की।

Q 52.B
• ज्िालामुखीय थथलाकृ षतयों को भू-पपषटी के भीतर या भू-पपषटी पर लािा के ठं डे होने के थथान के आधार पर बषहिेधी या िाह्य
थथलाकृ षतयों (Extrusive landforms) और अन्तिेधी या आंतररक थथलाकृ षतयों (intrusive landforms) में षिभाषित
ककया िाता है।
• ज्िालामुखी उद्गार के दौरान षनथसृत होने िाले पदाथों से बषहिेधी थथलाकृ षतयों का षनमाषर् होता है। ज्िालामुखी उद्गार के
दौरान षनथसृत पदाथों में लािा प्रिाह, पाइरोक्लाषथटक मलबा, ज्िालामुखीय बम, राख एिं धूलकर्, नाइिोिन यौषगक,
सलफर यौषगक और कु छ मात्रा में क्लोरीन, हाइड्रोिन और आगषन िैसी गैसें िाषमल हैं।
• बषहिेधी थथलाकृ षतयों के उदाहरर् षनम्नषलषखत हैं:
o िंिाकार और दरारी उद्गार नषलकाएं
o मध्य-महासागरीय कटक
o षमषित प्रकार की ज्िालामुखीय थथलाकृ षतयां
o िीलड प्रकार की ज्िालामुखीय थथलाकृ षतयां
o दरारी उद्गार प्रकार की बेसालट प्रिाह थथलाकृ षतयां
• कालडेरा
• ससंडर िंकु
o असंगरठत पाइरोक्लाषथटक टुकडों से षनर्मषत ककसी तीव्र िंिाकार ढाल िाली थथलाकृ षत को ससंडर िंकु कहा िाता है।
ज्िालामुखी उद्गार के दौरान षनकास नषलका के आस-पास षनथसृत ज्िालामुखीय सक्लंकर, ससंडर, ज्िालामुखीय राख
(थकोररया) आकद असंगरठत पाइरोक्लाषथटक पदाथों का िमाि हो िाता है।
o ससंडर िंकु ज्िालामुखी पूर्त ष ः असंगरठत सूक्ष्म कर्ों से षनर्मषत होते हैं तथा इनके षनमाषर् में लािा का योगदान नहीं होता
है। ससंडर िंकु का ढाल सामान्यतः अत्यषधक तीव्र होता है और इसके िीिष पर एक छोटा क्रेटर षथथत होता है। ये छोटे-छोटे
ज्िालामुखी होते हैं।
o िब लािा सतह पर न पहुंचकर भू-पपषटी के भीतर ही ठं डा हो िाता है तब अन्तिेधी आिेय िैलों या प्लूटोषनक िैलों का
षनमाषर् होता है। ज्िालामुखी की अन्तिेधी गषतषिषधयों के कारर् अन्तिेधी आिेय िैलों की षिषभन्न आकृ षतयों का षनमाषर्
होता है।
• अन्तिेधी थथलाकृ षतयों के उदाहरर् षनम्नषलषखत हैं:
o बैथोषलथ
o लैकोषलथ
o लैपोषलथ
o फै कोषलथ
o षसल
• डाइक
o अन्तिेधी आिेय चट्टानों के भू-पपषटी के अंदर क्षैषति लािा परतों के षनकट ठं डे होने से षसल का षनमाषर् होता है। इन
चट्टानों के कम मोटाई िाले िमाि को िीट (sheets) कहते हैं, िबकक सघन मोटाई िाले िमाि को षसल (sills) कहते हैं।
o िब दरार के माध्यम से लािा का प्रिाह ऊपर की ओर होता है और धरातल के लगभग लंबित रूप में िमा हो िाता है तो
इसके पररर्ामथिरूप एक दीिार की भांषत संरचना का षनमाषर् होता है, षिसे डाइक कहा िाता है। पषिमी महाराष्ट्र और
दक्कन िैप के अन्य भागों में अन्तिेधी आिेय चट्टानों में डाइक बहुतायत में पाई िाती हैं।

23 www.visionias.in ©Vision IAS

Google it:- https://upscpdf.com


https://t.me/UPSC_PDF Download From > https://upscpdf.com https://t.me/UPSC_PDF

Q 53.C
• षिकलप (c) सही उत्तर है।
• सरसों (Mustard):
o इसे मुख्य रूप से भारत के उत्तरी क्षेत्रों में उगाया िाता है। षिगत दिक में इसके उत्पादन में तीव्र िृषद्ध हुई थी। यह रबी की
एक फसल है।
o इसका सबसे बडा लाभ यह है कक इसे षिषिध कृ षि-िलिायु पररषथथषतयों में उगाया िा सकता है। भारत में अलसी और
सरसों की कृ षि एक साथ की िाती है। यह भारत में उपिाई िाने िाली नौ प्रमुख षतलहन फसलों में से एक है और इसका
थथान मूगं फली के पिात् दूसरा है। इस फसल की कटाई माचष या अप्रैल में की िाती है।
o अलसी और सरसों में तेल की मात्रा 36% से 42% के मध्य होती है। भारतीय सरसों का थिाद तीखा (pungent
flavour) होता है और प्रायः इसका उपयोग षिषभन्न भारतीय व्यंिनों में मसाले के रूप में ककया िाता है।
• अलसी (Flaxseed):
o अलसी एक िार्िषक िाकीय पौधा (herbaceous annual) है।
o इसका पौधा अनेक प्रकार की मृदाओं और िलिायु के साथ अनुकूषलत हो सकता है, ककन्तु सुप्रिाषहत रेतीली दोमट मृदा
और समिीतोष्र् िलिायु में ही उषचत रूप से िृषद्ध करता है।
o कई क्षेत्रों में एक ही भूषम पर अलसी की कृ षि छह ििष में एक बार की िाती है, क्योंकक षनरंतर एक ही भूषम पर इसकी
कृ षि करने से मृदा की उिषरता का ह्रास होता है।
o सबसे िांछनीय रे िों (फाइबर) का उत्पादन िीत आर्द्ष मौसम में होता है।
o अलसी के तेल में 57% और 71% के बीच पॉलीअनसेचरु े टेड िसा (अलफा-षलनोलेषनक एषसड, षलनोषलक एषसड) की
मात्रा षिद्यमान होती है।
• षतल (Sesame) - षतल षिश्व भर के उष्र्करटबंधीय क्षेत्रों में प्राकृ षतक रूप से उगाया िाने िाला एक पुष्पीय पौधा
(flowering plant) है और इसके खाद्य बीिों (edible seeds) के षलए इसकी कृ षि की िाती है।
o षतल की ककथमें अनेक प्रकार की मृदा के अनुकूल होती हैं, ककन्तु सुप्रिाषहत, मध्यम आकार िाली उिषर और उदासीन pH
िाली मृदा में उषचत रूप से िृषद्ध करती हैं। हालााँकक, इसकी फसल उच्च लिर्ीय और िल भराि िाली मृदा के षलए कम
सहनिील होती है।
o िाषर्षज्यक षतल की फसलों को 90 से 120 पालारषहत कदनों की आिश्यकता होती है। इसकी िृषद्ध और पैदािार के षलए
23 षडग्री सेषलसयस से अषधक उष्र् पररषथथषतयां आिश्यक होती हैं।
• सोयाबीन (Soybean)
o यह पूिी-एषिया की एक लेग्यूषमनस थथानीय फसल है, षिसकी कृ षि व्यापक रूप से खाद्य फषलयों (edible bean) के
षलए की िाती है। इसके अंकुरर् के षलए लगभग 15 से 32°C तापमान की आिश्यकता होती है, ककन्तु तीव्र िृषद्ध के षलए
फसल को उच्च तापमान की आिश्यकता होती है।
o फसल को पुष्पन के समय या पुष्पन से ठीक पूिष 60-65 सेमी िार्िषक ििाष की आिश्यकता होती है षिससे पुष्पन और
फषलयों का षनमाषर् सुगम हो िाता है, िबकक पररपिता के दौरान ििाष होने से सोयाबीन के बीिों की गुर्ित्ता खराब हो
िाती है। इसकी िृषद्ध के षलए बेहतर काबषषनक पदाथों िाली रेतीली दोमट मृदा आिश्यक होती है।
o सोयाबीन की कृ षि खरीफ और िसंत दो फसलीय मौसमों में की िाती है।
o खरीफ के मौसम में बुिाई का सबसे सामान्य समय मानसून की िुरुआत या िून के अंषतम सप्ताह से िुलाई के प्रथम सप्ताह
तक होता है िबकक िसंत की बुिाई 15 फरिरी से 15 माचष के मध्य की िाती है।
Q 54.A
• कभी प्रिासी पषक्षयों के षलए एक प्रमुख आिय थथल के रूप में ज्ञात षबहार में षथथत कािर झील एषिया की सबसे बडी मीठे
िल की गोखुर झील है। यह झील ितषमान में लुप्त होता िा रहा एक आर्द्षभूषम पाररषथथषतकी तंत्र है। थथानीय रूप से कािर नाम
से षिख्यात यह झील बेगूसराय िहर से 22 ककमी की दूरी पर उत्तर-पषिम में षथथत है। यह एक अिषिष्ट गोखुर झील है,
षिसका षनमाषर् भूगभीय काल में गंडक नदी (गंगा की एक सहायक नदी) में षिसपषर् के कारर् हुआ है। उललेखनीय है कक कािर
झील को िन्यिीि (संरक्षर्) अषधषनयम, 1972 के तहत एक अषधसूषचत क्षेत्र घोषित ककया गया था। पषक्षयों के अिैध षिकार
की रोकथाम हेतु इसे ििष 1986 में षबहार राज्य सरकार द्वारा संरषक्षत क्षेत्र घोषित ककया गया था। ज्ञातव्य है कक ििष 1989 में
24 www.visionias.in ©Vision IAS

Google it:- https://upscpdf.com


https://t.me/UPSC_PDF Download From > https://upscpdf.com https://t.me/UPSC_PDF

भारत सरकार द्वारा इसे एक पक्षी अभयारण्य घोषित ककया गया था। प्राषधकरर्ों ने 15,000 एकड (एक एकड 0.4 हेक्टेयर के
समतुलय होता है) क्षेत्र को एक आर्द्षभषू म के रूप में अषधसूषचत ककया था, िो रािथथान के भरतपुर में षथथत के िलादेि राष्ट्रीय
उद्यान के कु ल क्षेत्रफल से छह गुना अषधक है। परन्तु इस झील का क्षेत्र तीव्र गषत से संकुषचत हो रहा है। ििष 1984 में झील
6,786 हेक्टेयर क्षेत्र में षिथतृत थी, िो 2004 में घटकर 6,043.825 हेक्टेयर रह गया। ििष 2012 तक झील का क्षेत्र घटकर
मात्र 2,032 हेक्टेयर रह गया था। झील में िल थतर कम हो गया है, षिसका कारर् अत्यषधक गाद और सुपोिर् (िब अत्यषधक
िैिालों एिं पादपों की िृषद्ध तथा उनके अपघटन से िल में उपलब्ध ऑक्सीिन की मात्रा में कमी होती है , षिससे अन्य िीिों
की मृत्यु हो िाती है) पररलषक्षत होता है।
• बरुआ सागर ताल एक षििाल झील है, िो उत्तर प्रदेि में झााँसी के षनकट बरुआ सागर में षथथत है।
• उधिा झील पक्षी अभयारण्य समपूर्ष झारखंड राज्य में अिषथथत एकमात्र पक्षी अभयारण्य है। यह अभयारण्य िीत ऋतु में
यूरोप और साइबेररया से व्यापक संख्या में आने िाले प्रिासी पषक्षयों के षलए एक अथथायी आिय थथल (stopping point) के
रूप में प्रषसद्ध है।
• के िलादेि राष्ट्रीय उद्यान या के िलादेि घाना राष्ट्रीय उद्यान रािथथान के भरतपुर में षथथत है। ज्ञातव्य है कक इसे पूिष में भरतपुर
पक्षी अभयारण्य के रूप में िाना िाता था। यह एक प्रषसद्ध पक्षी अभयारण्य है। के िलादेि घाना राष्ट्रीय उद्यान एक मानि
षनर्मषत और मानि-प्रबंषधत आर्द्षभूषम है तथा भारत के प्रमुख राष्ट्रीय उद्यानों में से एक है। यह एक षिश्व षिरासत थथल भी है।

Q 55.A
• समथत ऑथिेषलया में बुिफायर (Bushfires) और ग्रासफायर (grassfires) सामान्य घटनाएं हैं। ग्रासफायर पांच से दस सेकंड
में ही तीव्र गषत से प्रसाररत हो िाती है और कई षमनटों तक िलती या सुलगती रहती है। बुिफायर सामान्यतः मंद गषत से
प्रसाररत होती है, ककन्तु अषधक ऊष्मा उत्पन्न करती है। इसका तात्पयष यह है कक यह दो से पांच षमनट में प्रसाररत होती है ककन्तु
कई कदनों तक िलती या सुलगती रह सकती है। ऑथिेषलया के पयाषिरर् में बुिफायर एक सामान्य घटना है। ऑथिेषलया के
अनेक देिि पादप अषि-प्रिर् और अत्यषधक ज्िलनिील होते हैं, िबकक अनेक प्रिाषतयााँ ऐसी हैं िो भूषम पर पुन: उत्पन्न होने
के षलए िनाषि पर षनभषर करती हैं। देिि ऑथिेषलयाई लोगों द्वारा दीघषकाल से भूषम प्रबंधन उपकरर् के रूप में अषि का
उपयोग ककया िाता रहा है तथा इसका उपयोग कृ षि उद्देश्यों हेतु भूषम को साफ करने और तीव्र, अषनयंषत्रत अषि से संपषत्तयों
के संरक्षर् हेतु अभी भी ककया िा रहा है।
• मूलभूत कारकों िैसे ईंधन, ऑक्सीिन और ककसी प्रज्िलन स्रोत की उपषथथषत यह षनधाषररत करती है कक बुिफायर की घटना
घरटत होगी अथिा नहीं। बुिफायर का प्रसार अषि की तीव्रता और गषत पर आधाररत होता है, िो पररिेि के तापमान, फ्यूल
लोड (ज्िलनिील पदाथष की मात्रा), फ्यूल मॉइथचर (ज्िलनिील पदाथष की नमी), िायु प्रिाह और ढाल के कोर् पर षनभषर
करता है।
o फ्यूल लोड (Fuel load): फ्यूल लोड को भूषम पर संषचत िृक्षों से षगरी हुई सूखी छाल, पषत्तयों के ढेर और छोटी िाखाओं
की मात्रा के रूप में िर्र्षत ककया िाता है।
o फ्यूल मॉइथचर (Fuel moisture): िुष्क िनीय सामषग्रयााँ (िैसे िुष्क लकडी, पषत्तयााँ घास-फू स आकद) त्िररत रूप से िल
िाती हैं, ककन्तु नम या आर्द्ष सामषग्रयााँ सरलता से िल नहीं पाती हैं। इसके पररर्ामथिरूप, ििाष के बाद का समय और
प्राप्त ििाष की मात्रा बुिफायर के खतरे का आकलन करने हेतु एक महत्िपूर्ष कारक है।
o िायु प्रिाह (Wind speed): यह ताज़े ईंधन (िनीय सामग्री) को प्रज्ज्िलन सबंद ु पर लाकर अषि प्रज्िषलत करने का कायष
करती है और ऑक्सीिन की षनरं तर आपूर्तष करती है। अषि-पुि
ं ों के प्रसार (spotting) के माध्यम से िायु अषि के तीव्र
प्रसार को बढ़ाती है, अथाषत् िायु के माध्यम से आग की लपटों के अन्यत्र िगहों पर पहुाँचने से अषि दूसरी िगहों पर भी
प्रज्िषलत होती रहती है।
o पररिेि का तापमान (Ambient temperature): तापमान षितना अषधक होगा, आग लगने या उसके षनरं तर प्रसाररत
होने की संभािना उतनी ही अषधक होगी। ऐसा इसषलए है क्योंकक उच्च तापमान पर िनीय सामग्री प्रज्िलन सबंदु के षनकट
होती है और पहले से ही उष्र् फ्यूल लोड िीघ्रता से िलने लगता है।
o सापेषक्षक आर्द्षता (Relative humidity): िुष्क िायु, आर्द्ष िायु की तुलना में अषधक तीव्रता िाली अषि को बढ़ािा देती
है। पादप षनम्न आर्द्षता पर अषधक ज्िलनिील हो िाते हैं क्योंकक िे अपनी नमी को अषधक सुगमता से षनष्काषसत कर पाते
हैं।
25 www.visionias.in ©Vision IAS

Google it:- https://upscpdf.com


https://t.me/UPSC_PDF Download From > https://upscpdf.com https://t.me/UPSC_PDF

o ढाल का कोर् (Slope angle): अषि षिककरर् और संिहन के माध्यम से अपने ईंधन स्रोत को पहले से ही गमष कर देती है।
पररर्ामतः, िैसे-िैसे अषि का आरोही प्रिाह होता है िैसे- िैसे इसकी तीव्रता में िृषद्ध होती िाती है और िैस-े िैसे यह
अधोगामी गषत करती है िैस-े िैसे उसकी तीव्रता में कमी आती िाती है।
o प्रज्िलन स्रोत (Ignition Source): बुिफायर मानिीय गषतषिषधयों और प्राकृ षतक कारर्ों (तषडत िैसे प्रमुख प्राकृ षतक
स्रोत), दोनों से उत्पन्न हो सकती है। ऑथिेषलया में लगभग 50% बुिफायर के षलए प्राकृ षतक कारक उत्तरदायी होते हैं।
o इसषलए सभी षिकलप सही हैं।

Q 56.C
• हमारे सौर मंडल में आठ ग्रह हैं। आठ ग्रहों में से बुध, िुक्र, पृथ्िी और मंगल को आंतररक ग्रह कहा िाता है क्योंकक िे सूयष और
क्षुर्द्ग्रहों की पट्टी के बीच में षथथत हैं। अन्य चार ग्रहों को बाह्य ग्रह कहा िाता है। िैकषलपक रूप से , पहले चार ग्रहों को पार्थषि
ग्रह कहा िाता है, षिसका अथष है कक ये ग्रह पृथ्िी की भांषत ही िैलों और धातुओं से बने हैं तथा इनका घनत्ि अपेक्षाकृ त उच्च है।
अन्य चार ग्रह िोषियन या षििाल गैसीय ग्रह कहलाते हैं। िोषियन का अथष है बृहथपषत की भांषत। इनमें से अषधकांि पार्थषि
ग्रहों की तुलना में षििाल हैं और उनमें अषधकांितः हीषलयम और हाइड्रोिन से षनर्मष त सघन िायुमंडल हैं। इसके षनम्न कारर्
हैं:
o पार्थषि ग्रह मूल/िनक तारे के षनकट बने िहााँ अत्यषधक तापमान के कारर् गैसें संघषनत नहीं हो पाईं तथा घनीभूत भी
नहीं हो सकीं। िोषियन ग्रहों का षनमाषर् अपेक्षाकृ त अषधक दूरी पर हुआ।
o सौर पिनें सूयष के समीप अषधक िषक्तिाली थीं, अतः पार्थषि ग्रहों से षििाल मात्रा में गैस और धूलकर् उडा ले गईं। सूयष से
दूरी बढ़ने पर सौर पिनें इतनी िषक्तिाली नहीं रह गईं षिसके कारर् िे िोषियन ग्रहों से गैसों को नहीं हटा पाईं। इसषलए
कथन 1 सही है।
o पार्थषि ग्रहों के छोटे होने के कारर् इनकी कमज़ोर गुरुत्िाकिषर् िषक्त बाहर िाती हुई गैसों को बांध कर नहीं रख सकी।
इसषलए कथन 2 सही है।
Q 57.A
• ििष 2019 भारत में षततली िोधकताषओं के षलए एक महत्िपूर्ष ििष रहा है।
• िनिरी 2019 में 120 ििों की लंबी अिषध के बाद गंगटोक में षसकक्कम षिश्वषिद्यालय के िोधकताषओं ने कं चनिंगा नेिनल
पाकष से थमॉल िुडब्राउन षततली प्रिाषतयों की पुनः खोि की है।
• तषमलनाडु में षततषलयों की कु ल 32 प्रिाषतयााँ हैं िो राज्य में थथाषनक हैं। निीनतम नीषतगत पहल के साथ, महाराष्ट्र के बाद
यह पांचिा भारतीय राज्य बन गया है षिसने अपने इस रं गीन कीट (षततली) को राज्य प्रतीक का दिाष कदया है।
• भारत में पाई िाने िाली महत्िपूर्ष षततषलयााँ:
o महाराष्ट्र- ब्लू मॉरमॉन, िो भारत (दषक्षर् भारत) और िीलंका में पायी िाती है,
o उत्तराखंड- कॉमन पीकॉक िो एषिया और ऑथिेषलया की मूल प्रिाषत है और इसे चाइनीज़ पीकॉक बटरफ्लाई भी कहा
िाता है,
o कनाषटक- सदनष (दषक्षर्ी) बडषसिंग, िो दषक्षर् भारत में थथाषनक है और भारत की सबसे बडी षततली है) और
o के रल द्वारा अपने राज्य की एक षततली (मालाबार बैंडड
े पीकॉक) को राज्य प्रतीक का दिाष कदया गया है।
• राज्य में 32 षततली हॉटथपॉट हैं, षिन्हें ककसी ऐसे थथल के रूप में षनर्दषष्ट ककया िाता है िहां राज्य में उपलब्ध सभी प्रिाषतयों
में से कम से कम 25% पाई िाती हों। षििेिज्ञ ऐसे और अषधक क्षेत्रों की पहचान करने हेतु प्रयासरत हैं।
• इसषलए के िल षिकलप 1 सही है।

Q 58.D
• षििाल बाढ़ एिं डेलटा मैदानों में नकदयां कदाषचत ही सीधे मागों में प्रिाषहत होती हैं। बाढ़ और डेलटा मैदानों में लूप सदृि
(िक्राकार) िाषहका (चैनल) प्रारूप षिकषसत होते हैं, षिन्हें षिसपष (Meanders) कहा िाता है। इनके षनर्मषत होने के षलए
अषनिायष पररषथथषतयां षनम्नषलषखत हैं-
o मंद ढाल पर प्रिाषहत िल की तटों पर क्षैषति या पाश्वष अपरदन करने की प्रिृषत्त। इसषलए कथन 1 सही है।

26 www.visionias.in ©Vision IAS

Google it:- https://upscpdf.com


https://t.me/UPSC_PDF Download From > https://upscpdf.com https://t.me/UPSC_PDF

o तटों पर िलोढ़ षनक्षेपों की अषनयषमत और असंगरठत प्रकृ षत होती है, षिनके माध्यम से िल पार्श्वषक दबाि उत्पन्न करता
है। इसषलए षिकलप 2 सही है।
o कोररऑषलस बल से प्रिाषहत िल का षिक्षेपर् (िायु के षिक्षेपर् के समान)। इसषलए षिकलप 3 सही है।
o िब चैनल की ढाल प्रिर्ता अत्यषधक मंद हो िाती है, तो िल का प्रिाह भी धीमा हो िाता है और यह पाश्वों का अपरदन
करना आरंभ कर देता है। तटों पर षिद्यमान अलप अषनयषमतताएं भी धीरे -धीरे एक लघु िक्र या मोड में पररिर्तषत हो
िाती हैं।
o ये मोड नदी के आंतररक भाग में िलोढ़ षनक्षेप और बाह्य तट के अपरदन के कारर् तीव्र हो िाते हैं। यकद अपरदन, षनक्षेपर्
तथा षनम्न कटाि न हो तो षिसपष षनमाषर् की प्रिृषत्त कम हो िाती है।
• षिसपों के गहरे छलले के आकार के रूप में षिकषसत हो िाने पर ये आंतररक भागों में अपरदन के कारर् नदी चैनल से पृथक हो
िाते हैं, षिनसे गोखुर झील (Ox-bow lake) का षनमाषर् होता है।

Q 59.A
• पृथ्िी का अक्षीय झुकाि मौसमी घटनाओं का कारर् बनता है। ििषपयषन्त, पृथ्िी के षिषभन्न भागों पर सूयष की अषधकांि ककरर्ें
प्रत्यक्ष रूप से आपषतत होती हैं। इसषलए, िब उत्तरी ध्रुि का सूयष की ओर झुकाि होता है, तब उत्तरी गोलाद्धष में ग्रीष्मकाल
आरमभ होता है तथा िब दषक्षर्ी ध्रुि का सूयष की ओर झुकाि होता है, तब उत्तरी गोलाधष में िीतऋतु का आगमन होता है।
इसषलए कथन 1 सही है।
• पृथ्िी का पररक्रमर् न के िल तापमान पररघटनाओं को प्रभाषित करता है, बषलक िथतुतः उनके घरटत होने का कारर् भी बनता
है, षिससे िसंत, ग्रीष्म, पतझड और िीतकाल िैसी ऋतुएाँ घरटत होती हैं। पृथ्िी द्वारा सूयष की पररक्रमा के कारर् ककसी
गोलाद्धष के सूयष की ओर झुकाि की षथथषत पर षिषिष्ट मौसम की षिद्यमानता षनभषर करती है। अत: प्रत्येक गोलाद्धष में मौसम
सदैि षिपरीत होते हैं। इसषलए कथन 2 सही है।
• पृथ्िी की धुरी एक कालपषनक ध्रुि है, िो पृथ्िी के कें र्द् के माध्यम से "िीिष" से "आधार" की ओर होता है। पृथ्िी इस ध्रुि के
चारों ओर घूर्न
ष करती है, षिससे प्रत्येक पूर्ष घूर्न
ष एक कदन में पररिर्तषत होता है। यही कारर् है कक कदन और रात घरटत होते
हैं तथा पृथ्िी के घूर्न
ष से मौसम प्रभाषित नहीं होता है। इसषलए कथन 3 सही नहीं है।
Q 60.B
• िांषत पैगोडा, षिन्हें िांषत थतूपों के रूप में भी िाना िाता है, िांषत को प्रेररत करने हेतु षनर्मषत बौद्ध थमारक हैं। ये थथान
प्रत्येक नथल एिं पृष्ठभूषम के व्यषक्त के षलए षनर्मषत ककए गए थे। थतूपों का षनमाषर् िापानी षभक्षु षनककडत्सु फू िी के मागषदिषन में
ककया गया था, िो महात्मा गांधी की असहंसा के प्रषत प्रषतबद्धता से अत्यषधक प्रभाषित थे। इसषलए कथन 1 सही नहीं है।
• षिश्व में षििेि रूप से षहरोषिमा, मयूषनख और नागासाकी िैसे युद्ध-ग्रथत (षद्वतीय षिश्व युद्ध) िहरों में षनर्मषत ये थतूप एकता
और घषनष्ठता के प्रतीक हैं। इन महान संरचनाओं का षनमाषर् फू िी द्वारा थथाषपत षनप्पोनज़ू मायोिी बौद्ध संघ के षभक्षुओं और
षभक्षुषर्यों की सहायता से थथानीय लोगों द्वारा ककया गया था।
• इन प्रमुख िांषत थतूपों में से एक षबहार के रािगीर में षथथत है। षनककडत्सु फू िी द्वारा िांषत पैगोडाओं की थथापना की िृंखला
में, रािगीर में भी एक िांषत पैगोडा थथाषपत ककया गया था, षिसे षिश्व िांषत थतूप कहा िाता है। यह श्वेत संगमरमर से
षनर्मषत है और षिश्व में सिाषषधक उं चाई (400 मी. की ऊंचाई पर एक पहाडी पर) पर षथथत पैगोडा है। थतूप के चारों ओर
भगिान बुद्ध की चार प्रषतमाएं षनर्मषत की गई हैं। इसषलए कथन 2 सही है।
• हाल ही में, षबहार के रािगीर में षिश्व िांषत थतूप (षिश्व िांषत पैगोडा) की 50िीं ििषगांठ मनाई गई षिसमें भारत के राष्ट्रपषत
ने भी भाग षलया।

Q 61.A
• प्रिाषतयों की सुभद्य
े ता, दो कारकों यथा उनकी थथाषनकता और दुलभ
ष ता के संयोिन के कारर् होती है। िेरों, भेषडयों और
िंगली कु त्तों के मामले में, यहां तक कक िब िंतु संरषक्षत क्षेत्रों में रह रहे हैं तब भी के िल आबादी का आकार थथानीय थतर पर
उनकी षिलुषप्त के पूिाषनुमान हेतु पयाषप्त कारक नहीं है।

27 www.visionias.in ©Vision IAS

Google it:- https://upscpdf.com


• सामान्यतः एक िंतु के िरीर का आकार षितना बडा होता है, उतने ही अषधक समय तक िह िीषित रहता है तथा उसके द्वारा
प्रषतििष उत्पन्न संतषतयों की संख्या भी उतनी ही कम होती है। अपेक्षाकृ त षििाल आकार के िानिरों में तुलनात्मक रूप से षनम्न
आबादी घनत्ि की प्रिृषत्त होती है। इस प्रकार, कहा िा सकता है कक एक षिकासक्षम आबादी िैसे- हाषथयों की आबादी, उतनी
ही संख्या में खरगोिों की आबादी की तुलना में अत्यषधक थथान अषधगृहीत करती है।
• कु छ प्रिाषतयां समूहन व्यिहार प्रदर्िषत करती हैं, षिससे िे ककसी प्रकार के षिक्षोभ या षिकार के प्रषत सुभद्य
े बन िाती हैं।
उदाहरर् के षलए, समूहन के इस व्यिहार के कारर् चमगादड अपनी आबादी के एक बडे भाग को तब अत्यषधक सुभेद्य या
संिेदनिील बना सकते हैं िब िे अपनी संतषतयों के पालन के षलए बडी संख्या में ककसी षििेि गुफा में समूहन करते हैं। ऐसे में
उनकी एक बडी आबादी या पयाषिास मानि के आिागमन से अथिा गुफा के व्यिसायीकरर् या बाढ़ से षिक्षुब्ध, अव्यिषथथत
या क्षषतग्रथत हो सकते हैं।
• ऐसी प्रिाषतयां षिनमें षनम्न प्रकीर्षन (dispersal) दर या तेज़ी से षततर-षबतर होने की अलप क्षमता होती है िे प्रषतकू ल षथथषत
में होती हैं, क्योंकक िहााँ पर यह संभािना समाप्त हो िाती है कक एक आबादी के द्वारा दूसरी आबादी को बचाया िा सके ।
क्योंकक छोटी आबादी, िृहद् आबादी की तुलना में अत्यषधक िोषखम में होती है, एकल प्रिाषतयां िो तत्परता से प्रकीर्र्षत हो
सकती हैं िे षिलुषप्त की कगार पर षथथत थथानीय आबादी को बचा सकती हैं।

Q 62.C
• एन्हासन्संग कोथटल एंड ओिन ररसोसष एकफषिएंसी (ENCORE) प्रोिेक्ट को दो अषतव्यापी चरर्ों (प्रत्येक पांच ििष का) के
साथ आठ ििीय मलटीफे ज़ प्रोग्रामेरटक एप्रोच (MPA) ऑपरे िन के रूप में प्रथताषित ककया गया है, िो भारत के राष्ट्रीय तटीय
षमिन (India’s National Coastal) अन्तःथथाषपत है। ENCORE पररयोिना का उद्देश्य षनम्नषलषखत घटकों के माध्यम से
भारत के सभी तटीय राज्यों और संघ िाषसत प्रदेिों में एकीकृ त तटीय क्षेत्र प्रबंधन को सुदढ़ृ करना है।
o घटक 1: षिके न्र्द्ीकृ त तटीय प्रबंधन के षलए बेहतर क्षमता।
o घटक 2: तटीय क्षेत्रों में बेहतर संरक्षर् और प्रदूिर् उपिमन।
o घटक 3: पररयोिना का प्रबंधन, षनगरानी एिं मूलयांकन।
• यह कायषक्रम, एकीकृ त तटीय क्षेत्र प्रबंधन (ICZM) दृषष्टकोर् को अपनाने और कायाषषन्ित करने के षलए सामूषहक क्षमता
(समुदायों एिं षिके न्र्द्ीकृ त िासन सषहत) का षनमाषर् करके तटीय संसाधन दक्षता और प्रत्याथथता को बढ़ाने में भारत सरकार
की सहायता करने का प्रयास करता है।
• चूकं क ENCORE कायषक्रम, षिश्व बैंक के इन्िेथटमेंट प्रोिेक्ट फाईनेंससंग (IPF) इं थूमेंट के तहत षित्त पोषित है, इसषलए इस
इं थूमेंट की पररचालन नीषतयां (Operational Policies: OPs) और बैंक प्रकक्रयाएं (Bank Procedures: BPs)
पररयोिना पर भी लागू होती हैं।
• इसषलए दोनों कथन सही हैं।

Q 63.B
• षहमानी धौत (Outwash plain)
o एक षहमानी धौत, षहमनद के अंत में षपघले हुए िल के षहमानी द्वारा षनक्षेषपत षहमानी-िलोढ़ षनक्षेपों (िैसे बिरी, रेत
आकद) से षनर्मषत एक मैदान होता है। इसे सैंडर (sander) भी कहा िाता है।
o प्रिाषहत अिथथा में होने के कारर् षहमनद द्वारा अंतर्नषषहत चट्टानी सतह का घिषर् ककया िाता है और इससे प्राप्त मलबे
को षहमनद अपने साथ िहन करता है। षहमनद के मुहाने पर षपघले हुए िल के द्वारा षहमानी धौत पर अपने साथ प्रिाषहत
मलबे का षनक्षेपर् कर कदया िाता है। उललेखनीय है कक बडे गोलाश्म षहमोढ़ के अंषतम छोर पर षनक्षेषपत होते रहते है
तथा छोटे कर् षनक्षेषपत होने से पूिष आगे प्रिाषहत होते रहते हैं।
o सैंडसष, आइसलैंड में सामान्य रूप से पाए िाते हैं िहां भूतापीय प्रकक्रया, बफष को षपघलाने और षपघले हुए िल से अिसाद
के षनक्षेप की गषत को तीव्र कर देती है।
• सकष (cirques)
o षहमनदीय पिषतों में सकष सिाषषधक सामान्य थथलाकृ षत है। सकष प्राय: षहमनद घारटयों के िीिष पर पाए िाते हैं। पिषत की
चोरटयों से नीचे की ओर प्रिाषहत होती षहम इन सको का कटाि करती है।
28 www.visionias.in ©Vision IAS
o सकष गहरे , लंबे एिं चौडे गतष होते हैं षिनकी दीिारें तीव्र ढाल िाली सीधी या अितल होती हैं। प्रायः षहमनद के षपघलने
पर इन सकों में िलीय झील का षनमाषर् भी हो िाता है। ऐसी झीलों को सकष झील या टानष झीलें कहा िाता है। दो या दो
से अषधक सकष एक सीढ़ीनुमा क्रम में दृषष्टगत होते हैं।
• प्लाया (Playas)
o मरुथथलों में सिाषषधक प्रमुख थथलाकृ षत मैदान होते हैं। चारों ओर और साथ में पिषतों और पहाषडयों से षघरे बेषसनों में िल
का अपिाह मुख्य बेषसन के मध्य में होता है तथा बेषसन के ककनारों से षनरं तर लाए गए अिसादों के षनक्षेप के कारर्,
बेषसन के मध्य में लगभग एक समतल मैदान की रचना हो िाती है।
o पयाषप्त िल उपलब्ध होने पर, यह मैदान एक उथले िल षनकाय से किर हो िाता है। इस प्रकार की उथली िल झीलें ही
प्लाया कहलाती हैं। इन झीलों में िल िाष्पीकरर् के कारर् के िल अलप अिषध के षलए ही षिद्यमान रह पाता है। प्राय:
प्लाया में लिर्ों के समृद्ध षनक्षेप पाए िाते हैं। ऐसे प्लाया मैदान िो लिर्ों से आच्छकदत होते हैं कललर भूषम या क्षारीय
क्षेत्र कहलाते हैं।

Q 64.B
• डी िैक (DTrack) हैकरों द्वारा भारत के षित्तीय और अनुसंधान कें र्द्ों पर हमला करने के षलए प्रयुक्त एक मैलिेयर है। इसकी
खोि साइबर षसक्योररटी फमष कै थपरथकाई (Kaspersky) द्वारा की गई थी। भारत में ATMs को हैक करने के षलए षडज़ाइन
ककया गया ATMDtrack, इसका एक पूिि
ष ती संथकरर् था। न्यूषक्लयर पािर कॉरपोरे िन ऑफ इं षडया षलषमटेड (NPCIL) ने
यह थिीकार ककया है कक उसके एक कं प्यूटर पर मैलिेयर का हमला हुआ है।
• मैलिेयर को पीषडत व्यषक्त के ATM में प्रिेि कराने हेतु षडज़ाइन ककया गया था, यह उन ATMs में प्रिेि कराए गए काडष डेटा
को पढ़ एिं संगह
ृ ीत कर सकता है। कु छ समय पिात् इसका एक संथकरर् दषक्षर् कोररया में बैंककं ग प्रर्ाली पर हमले के साथ-
साथ षिश्व भर में िानाक्राई (WannaCry) रैनसम िमष हमलों के षलए प्रयोग ककया गया था।
Q 65.C
• कथन 1 सही नहीं है: क्षोभमंडल िायुमंडल का सिाषषधक षनचला संथतर है। इसकी औसत ऊाँचाई 13 ककमी है तथा इसका
षिथतार ध्रुिों के पास लगभग 8 ककमी की ऊाँचाई और भूमध्य रे खा पर लगभग 18 ककमी तक होता है। इस संथतर में धूल के कर्
और िलिाष्प पाए िाते हैं। िलिायु और मौसम में होने िाले सभी पररितषन इस संथतर में घरटत होते हैं। इस परत में प्रत्येक
165 मीटर की ऊंचाई पर 1°C की दर से तापमान में कमी होती है।
• समताप मंडल से क्षोभमंडल को पृथक करने िाले क्षेत्र को क्षोभसीमा के रूप में िाना िाता है। क्षोभसीमा में िायु का तापमान
भूमध्य रे खा पर लगभग -80°C तथा ध्रुिों पर -45 °C होता है। यहााँ का तापमान लगभग षथथर होता है।

29 www.visionias.in ©Vision IAS


• समतापमंडल, क्षोभसीमा के ऊपर अिषथथत है तथा इसका प्रसार 50 ककमी की ऊाँचाई तक है। समताप मंडल की एक महत्िपूर्ष
षििेिता इसमें ओिोन परत की अिषथथषत है। यह परत पराबैगनी षिककरर् को अििोषित करती है तथा ऊिाष के तीव्र,
हाषनकारक प्रभािों से पृथ्िी पर िीिन के संरक्षर् का कायष करती है। इस परत में, ऊाँचाई के साथ तापमान में िृषद्ध होने लगती
है।
• मध्यमंडल, समताप मंडल के ऊपर षथथत है, षिसका षिथतार 80 ककमी की ऊंचाई तक है। इस परत में एक बार पुनः ऊंचाई में
िृषद्ध के साथ तापमान में कमी होती है और 80 ककमी की ऊाँचाई पर तापमान -100°C तक पहुाँच िाता है। मध्यमंडल की
ऊपरी सीमा को मध्यसीमा के रूप में िाना िाता है।
• कथन 3 सही है: आयनमंडल, मध्यसीमा के ऊपर 80 से 400 ककमी के मध्य षथथत है। इसमें षिद्युत आिेषित कर् पाए िाते हैं
षिन्हें आयन के रूप में िाना िाता है। इनकी अिषथथषत के कारर् इसे आयनमंडल के रूप में िाना िाता है। पृथ्िी से भेिी गयी
रे षडयो तरं गें इस परत द्वारा पृथ्िी पर पुनः परािर्तषत हो िाती हैं। इस परत में पुनः ऊंचाई के साथ तापमान में िृषद्ध होने लगती
है।
• कथन 2 सही नहीं है: समताप मंडल में ऊंचाई में िृषद्ध के साथ तापमान में षगरािट होती है। इस तापमान थतरीकरर् के कारर्,
समताप मंडल में अलप संिहन और षमिर् होता है, इसषलए यहां िायु की परतें अत्यषधक षथथर होती हैं। िाषर्षज्यक िेट
षिमान षिक्षोभों (िो क्षोभमंडल में एक सामान्य पररघटना है) से बचने के षलए षनचले समताप मंडल में उडते हैं।

Q 66.A
• दषक्षर् प्रिांत षथथत बोगनषिले द्वीप में संपन्न ऐषतहाषसक मतदान (िनमत संग्रह) में पापुआ न्यू षगनी से थितंत्र होने के पक्ष में
भारी मतदान ककया गया है। इसने अलगाििाकदयों को उत्साषहत ककया है क्योंकक उनके द्वारा ही संप्रभुता की मांग प्रारंभ की
गई थी।
• इस िनमत संग्रह (referendum) को 23 निंबर से 7 कदसंबर 2019 के बीच संपन्न ककया गया था, षिसका पररर्ाम 11
कदसंबर को घोषित ककया गया था। यह िनमत संग्रह मतदाताओं द्वारा पापुआ न्यू षगनी के भीतर अषधक थिायत्तता प्राप्त करने
या पूर्ष थितंत्रता प्राप्त करने के षिकलपों में से ककसी एक षिकलप का चयन करने हेतु करिाया गया था। िैध मतों में से 98.31%
पूर्ष थितंत्रता के पक्ष में डाले गए थे। इस मतदान का पररर्ाम बाध्यकारी नहीं है, बोगनषिले के दिे के संबंध में अंषतम षनर्षय
पापुआ न्यू षगनी की सरकार का ही होगा।

30 www.visionias.in ©Vision IAS


Q67. B
• उच्च ज्िार और षनम्न ज्िार के मध्य ऊंचाई के अंतर को ज्िारीय क्रम या ज्िारीय परास (tidal range) कहा िाता है।
• महीने में एक बार, िब चंर्द्मा की कक्षा पृथ्िी के सबसे षनकट होती है (उपभू), तो असामान्य रूप से उच्च और षनम्न ज्िार उत्पन्न
होते हैं। अतः इस अिषध के दौरान ज्िारीय क्रम सामान्य से अषधक होते हैं। दो सप्ताह बाद, िब चंर्द्मा पृथ्िी से अषधकतम दूरी
(अपभू) पर षथथत होता है, तब चंर्द्मा का गुरुत्िाकिषर् बल सीषमत हो िाता है और ज्िार-भाटा का क्रम उसकी औसत ऊंचाई
से कम हो िाता है।
• िब प्रत्येक ििष 3 िनिरी के आस-पास पृथ्िी सूयष के सबसे षनकट षथथत होती है (उपसौर), तो असामान्य रूप से उच्च और षनम्न
ज्िारों के साथ ज्िारीय क्रम भी काफी अषधक होते हैं। िब प्रत्येक ििष 4 िुलाई के आस-पास पृथ्िी सूयष से अषधकतम दूरी पर
षथथत होती है (अपसौर), तो ज्िारीय क्रम औसत की अपेक्षा बहुत कम होते हैं।
• इसषलए उपभू (Perigee) और उपसौर (Perihelion) की षथथषत में ज्िारीय क्रम सामान्य से अषधक होते हैं।

Q 68.A
• बांधिगढ़ राष्ट्रीय उद्यान भारत के राष्ट्रीय उद्यानों में से एक है, िो मध्य प्रदेि के उमररया षिले में षथथत है। बांधिगढ़ (105 िगष
ककलोमीटर क्षेत्र में षिथतृत) को 1968 में राष्ट्रीय उद्यान और 1993 में बाघ आरषक्षत क्षेत्र (टाइगर ररििष) घोषित ककया गया
था। इस उद्यान में सिाषषधक िैि-षिषिधता पाई िाती है। बांधिगढ़ भारत के सिाषषधक बाघ आबादी घनत्ि (8 बाघ प्रषत िगष
ककमी) िाले थथानों में से एक है। इस उद्यान में तेंदओं
ु और षहरर्ों की षिषभन्न प्रिाषतयों की बडी प्रिनन आबादी (breeding
population) पाई िाती है।
• निंबर 2018 में, प्रत्येक ििष की भांषत 38 हाषथयों के एक झुंड द्वारा भोिन और िल की खोि में बांधिगढ़ के िंगलों में प्रिेि
ककया गया था। ये पहली बार हुआ कक िे कई ऋतुओं तक मध्य प्रदेि में ही अषधिाषसत रहे एिं दो नए बच्चों को िन्म भी कदया
तथा एक ििष के पिात् भी उत्तरी छत्तीसगढ़ षथथत अपक्षषयत िंगलों में पुन: नहीं लौटे।
• "अषखल भारतीय बाघ अनुमान 2018" के अनुसार भारत षिश्व में बाघों के षलए सबसे बडे और सुरषक्षत पयाषिासों में से एक के
रूप में उभरा है। इसके तहत िर्र्षत ककया गया है कक 526 बाघों की संख्या के साथ मध्य प्रदेि देि का "बाघ राज्य (tiger
state)" है।
• बांदीपुर राष्ट्रीय उद्यान कनाषटक षथथत एक िन आरषक्षत क्षेत्र है, िहााँ बाघों की कु छ आबादी पाई िाती है।
• षिम कॉबेट राष्ट्रीय उद्यान उत्तराखंड षथथत एक िनाच्छाकदत िन्यिीि अभयारण्य है। यह िनथपषत-िात और प्रार्ी-िात में
समृद्ध है और यह "बंगाल टाइगर" के षलए प्रषसद्ध है।
• नागरहोल राष्ट्रीय उद्यान कनाषटक षथथत एक िन्यिीि आरषक्षत क्षेत्र है। यह नीलषगरर बायोथफीयर ररििष का एक भाग है।

Q 69. C
• कथन 1 और 3 सही नहीं हैं: यूरोपीय ग्रीन डील को कदसमबर 2019 में मैषड्रड में आयोषित अंतरराष्ट्रीय िलिायु पररितषन
सममेलन अथाषत् 25िीं कॉन्फ्रेंस ऑफ पाटीि (COP) की समाषप्त पर िारी ककया गया था। यह डील COP25 दथतािेिों का
भाग नहीं है। COP25 सममलेन के दौरान, यूरोपीय संघ (EU) ने िलिायु पररितषन के संबंध में उठाए िाने िाले अषतररक्त
उपायों के रूप में यूरोपीय ग्रीन डील की घोिर्ा की। यूरोपीय संघ की इस घोिर्ा को एक बडे कदम के रूप में थिीकार ककया
गया है, भले ही इसकी सफलता हेतु अन्य देिों के पूरक प्रयासों की आिश्यकता हो। चीन और संयक्त
ु राज्य अमेररका के पिात्
यूरोपीय संघ (इसके 28 सदथयों सषहत) षिश्व में ग्रीनहाउस गैसों का तीसरा सबसे बडा उत्सिषक है। भारत इस डील का एक
पक्षकार नहीं है क्योंकक यह पहल यूरोपीय देिों तक ही सीषमत है।
• कथन 2 सही है: ग्रीन डील का अषत महत्िपूर्ष उद्देश्य यूरोप की अथषव्यिथथा के रूपांतरर् हेतु एक रोडमैप प्रदान करना और
2050 तक यूरोप को प्रथम काबषन तटथथ (carbon neutral) महाद्वीप के रूप में थथाषपत करना है।
• ग्रीन डील में कई अन्य षिषिष्ट प्रथतािों को भी िाषमल ककया गया है, षिनमें से प्रमुख हैं:
o 2050 तक िून्य काबषन उत्सिषन करना तथा 1990 के थतर की तुलना में काबषन उत्सिषन में 50% (यकद 55% नहीं) की
कटौती करना।
31 www.visionias.in ©Vision IAS
o 2020 में एक नई रसायन रर्नीषत अपनाए िाने की संभािना है, िो िायु और िल प्रदूिर् षनिारर् पर कें कर्द्त होगी।
o 2020 में 'फामष टू फोकष (farm to fork)' नामक एक रर्नीषत िुरू की िाएगी, िो खाद्यान्न उत्पादन और षितरर् की
षथथरता में सुधार करेगी।

Q 70.B
• पयाषिरर् षिक्षा, िागरुकता और प्रषिक्षर् (EEAT) योिना पयाषिरर्, िन और िलिायु पररितषन मंत्रालय (MoEF&CC)
की 1983-84 से िारी एक थथाषपत कें र्द्ीय क्षेत्र की योिना है। इसका उद्देश्य पयाषिरर् िागरुकता को प्रोत्साषहत करना और
पयाषिरर् संरक्षर् में थकू ली छात्रों की भागीदारी को बढ़ाना है। इस योिना के तहत 2001-2002 में राष्ट्रीय हररत िाषहनी
(National Green Corps: NGC) 'इको-क्लब’ कायषक्रम को प्रारंभ ककया गया था, षिसका उद्देश्य थकू ली छात्रों को अपने
समीपिती पयाषिरर्, उसकी अंतर्क्रषयाओं और सामना की िाने िाली समथयाओं के बारे में प्रत्यक्ष अनुभि के माध्यम से ज्ञान
प्रदान करना था। इस कायषक्रम का उद्देश्य पयाषिरर् के प्रषत उषचत दृषष्टकोर् षिकषसत करना तथा पयाषिरर् और षिकास से
संबंषधत मुद्दों के संबंध बच्चों को संिेदनिील बनाना है।
• हाल ही में MoEF&CC के राष्ट्रीय हररत िाषहनी 'इको-क्लब’ कायषक्रम को लागू करने िाली राज्य नोडल एिेंषसयों की प्रथम
िार्िषक बैठक का आयोिन मंत्रालय के पयाषिरर् षिक्षा प्रभाग द्वारा ककया गया था। इसके अषतररक्त राष्ट्रीय हररत िाषहनी
योिना (िो एक दिक से अषधक समय से बंद थी) को िममू और कश्मीर (J&K) में पुनप्राषरंभ ककया गया है।
• MoEFCC द्वारा के िषडया (गुिरात) में आयोषित राज्य नोडल एिेंसी (SNA) की बैठक में िममू और कश्मीर प्रदूिर् षनयंत्रर्
बोडष (PCB) द्वारा इस योिना के पुनरुद्धार हेतु प्रयास ककए गए थे। तदनुसार, MoEF द्वारा J&K PCB को 2.60 करोड रुपये
का अनुदान िारी करने की थिीकृ षत प्रदान की है।
• इस योिना के तहत PCB कें र्द् िाषसत प्रदेि के दोनों क्षेत्रों में पंिीकृ त थकू लों के 3,742 इको-क्लबों के साथ-साथ पंिीकृ त
कॉलेिों के 96 इको-क्लबों को षित्तीय सहायता प्रदान करेगा, षिसके षलए बोडष द्वारा िीघ्र ही प्रत्यक्ष बैंक अंतरर् के माध्यम से
धनराषि िारी की िाएगी।

Q 71.D
• नगरीय ऊष्मा द्वीप (Urban Heat Island: UHI) एक नगरीय क्षेत्र या महानगरीय क्षेत्र होता है, िो मानिीय गषतषिषधयों के
कारर् अपने षनकटिती ग्रामीर् क्षेत्रों की तुलना में अषधक गमष होता है।
• समपूर्ष षिश्व में, नगर प्रायः अपने षनकटिती ग्रामीर् क्षेत्रों की तुलना में कु छ षडग्री अषधक गमष होते हैं। एक हाषलया अध्ययन ने
यह सत्याषपत ककया है कक षिश्व भर में, इस तथाकषथत UHI प्रभाि की तीव्रता एक िहर की िनसंख्या और िार्िषक ििषर् पर
षनभषर करती है।
• प्रषसद्ध िोधकताष गैषब्रएल मैनोली के अनुसार, इसका एक प्रमुख कारक - ककसी िहर द्वारा उसके चतुर्दषक भू-क्षेत्र की तुलना में
िायुमंडल के साथ आदान-प्रदान की िाने िाली िल की मात्रा है। यूषनिर्सषटी कॉलेि लंदन की पयाषिरर्ीय अषभयंता मैनोली के
अनुसार, "िाष्पन-िाष्पोत्सिषन (Evapotranspiration) िल को मृदा से िायुमंडल में थथानांतररत करने की एक प्रकक्रया होती
है तथा यह िनथपषत द्वारा षिषनयषमत होती है।" "इस प्रकक्रया के कारर् पृथ्िी की सतह पर िीतलन प्रभाि उत्पन्न होता है।"
• मैनोली की टीम को यह ज्ञात हुआ कक नगरीय ऊष्मा को कम करने के षलए िहर के हररत क्षेत्र में िृषद्ध करना एक सामान्य
रर्नीषत है। यकद ककसी िहर में उसके चतुर्दषक क्षेत्र की तुलना में अषधक िनीय क्षेत्र षिद्यमान होता है , तो UHI की तीव्रता
अपेक्षाकृ त कम होगी। परन्तु अध्ययन से यह ज्ञात होता है कक यकद ककसी िहर में हरा-भरा क्षेत्र (lush and verdant region)
षिद्यमान होता है, तब भी "िनथपषत ऊष्मा को कम करने में सहायता करे गी, परंतु उपयुषक्त के समान प्रभाि प्राप्त करने हेतु और
अषधक िनीकरर् की आिश्यकता होगी"।
• मैनोली ने थपष्ट ककया है कक क्षेत्रीय रूप से, "दो सामान्य प्रिृषत्तयां ज्ञात हैं यथा: नगरीय ऊष्मा द्वीप की तीव्रता िनसंख्या और
ििषर् में िृषद्ध के साथ बढ़ती है"। "िनसंख्या एक िहर के आकार, उसके थिरूप, गषतषिषधयों और अिसंरचनाओं का
प्रषतषनषधत्ि करती है, िबकक ििषर् थथानीय िलिायु तथा िानथपषतक षििेिताओं को षनरुषपत करता है।
• "कु छ िोधों के अनुसार, िहर के आकार और उसकी अषत सघन प्रकृ षत के साथ ऊष्मा द्वीप की तीव्रता बढ़ िाती है।

32 www.visionias.in ©Vision IAS


Q 72.B
• षिकलप में दी गई सभी पहाषडयााँ पूिािंचल पहाषडयों अथिा पूिी पहाषडयों का भाग हैं। दषक्षर्-पषिम से उत्तर-पूिष की ओर
इनकी कदिा षनम्नानुसार है यथा:
o डफला पहाषडयााँ
o षमरी पहाषडयााँ
o अबोर पहाषडयााँ
o षमश्मी पहाषडयााँ
• डफला पहाषडयााँ: ये तेिपुर और उत्तरी लखीमपुर के उत्तर में षथथत हैं तथा पषिम में एका पहाषडयााँ (Aka Hills) और पूिष में
अबोर िेर्ी से षघरी हुई हैं।
• अबोर पहाषडयााँ: यह भारत के सुदरू उत्तर-पूिष में चीन सीमा के षनकट अरुर्ाचल प्रदेि के एक क्षेत्र में षथथत हैं। यह पूिष में
षमश्मी और पषिम में षमरी पहाषडयों से षघरी हुई हैं। इस क्षेत्र से ब्रह्मपुत्र की सहायक नदी, कदबांग भी प्रिाषहत होती है।
• षमश्मी पहाषडयााँ: ये पहाषडयााँ महान षहमालय पिषतमाला के दषक्षर्ी षिथतार क्षेत्र में अिषथथत हैं तथा इसके उत्तरी और पूिी
भाग चीन से संलि हैं।

Q 73.D
• पषिमी घाट दक्कन पठार के भ्रंषित और अपरकदत ककनारों के पिषतीय भाग हैं। भूगभीय साक्ष्य इंषगत करते हैं कक पषिमी घाट
का षनमाषर् लगभग 150 षमषलयन ििष पूिष गोंडिाना के षििाल महाद्वीप के षिखंडन के दौरान हुआ था। इसषलए कथन 1 सही
है।

33 www.visionias.in ©Vision IAS


• पषिमी घाट पिषत िृंखला षहमालय के पिात् भारत की दूसरी सबसे बडी पिषत िृंखला है। इसकी लंबाई लगभग 1500
ककलोमीटर है। यह तापी नदी घाटी से नीलषगरर पहाषडयों तक षिथतृत है।
• यह भारतीय प्रायद्वीप के पषिमी तट के समानांतर षिथताररत है, िो के रल, तषमलनाडु , कनाषटक, गोिा, महाराष्ट्र और गुिरात
राज्यों में षिथतृत है।
• पषिमी घाट ब्लॉक पिषत के रूप में षिद्यमान हैं। भारत के अफ्रीका से पृथक होने की प्रकक्रया में, एक भ्रंि घाटी अरब सागर के
रूप में षिकषसत हुई तथा पषिमी घाट इस भ्रंि घाटी के कगार के रूप में िेि रह गए। प्रायद्वीपीय भारत के पषिमी घाट की
तीव्र ढलान उस भ्रंि को दिाषती है।
• अनाईमुडी (2,695 मीटर), पषिमी घाट की अन्नामलाई पहाषडयों पर अिषथथत प्रायद्वीपीय पठार का उच्चतम षिखर है, उसके
पिात् नीलषगरर पहाषडयों का डोडाबेट्टा (2,637 मीटर) दूसरा सबसे ऊंचा षिखर है। इसषलए कथन 2 सही है।
• अषधकांि प्रायद्वीपीय नकदयों का उद्गम पषिमी घाट से ही होता है।
• दक्कन का पठार पषिम में पषिमी घाट, पूिष में पूिी घाट तथा उत्तर में सतपुडा, मैकाल िेर्ी और महादेि पहाषडयों से षघरा है।
• पषिमी घाट को थथानीय रूप से षिषभन्न नामों से िाना िाता है, िैसे महाराष्ट्र में सह्याकर्द्, कनाषटक एिं तषमलनाडु में नीलषगरर
पहाषडयााँ तथा के रल में अन्नामलाई पहाषडयााँ और काडषमम (इलायची) पहाषडयााँ। इसषलए कथन 3 सही है।

Q 74.C
• हाल ही में सरकार ने षडषिटल संचार अिसंरचना के त्िररत षिकास, षडषिटल अंतराल को समाप्त करने, षडषिटल सिषक्तकरर्
और समािेिन को सुषिधािनक बनाने तथा सभी को िहनीय ि सािषभौषमक ब्रॉडबैंड सेिा उपलब्ध कराने हेतु राष्ट्रीय ब्रॉडबैंड
षमिन का िुभारंभ ककया है। षमिन के कु छ प्रमुख उद्देश्य षनम्नषलषखत हैं:
o ििष 2022 तक सभी गााँि को ब्रॉडबैंड सेिा उपलब्ध करिाना। इसषलए कथन 1 सही है।
o संपूर्ष देि षििेितया ग्रामीर् एिं सुदरू क्षेत्रों में ब्रॉडबैंड सेिा की सािषभौषमक ि न्यायोषचत पहुंच को सुषिधािनक बनाना।
o 30 लाख ककलोमीटर रूट तक िृषद्धमान (इं क्रीमेंटल) ऑषप्टकल फाइबर के बल षबछाई िाएगी तथा टािर घनत्ि को ििष
2024 तक प्रषत हिार िनसंख्या पर 0.42 से बढ़ाकर 1.0 ककया िाएगा।
o मोबाइल एिं इं टरनेट हेतु सेिाओं की गुर्ित्ता में महत्िपूर्ष रूप से सुधार करना।
o राज्यों/के न्र्द् िाषसत प्रदेिों के साथ षमलकर कायष करने के षलए राइट ऑफ िे (ककसी अन्य की अचल संपषत्त से मागष प्राप्त
करने का षिषधक अषधकार) मॉडल षिकषसत ककया िाएगा। यह ऑषप्टकल फाइबर के बल षबछाने हेतु आिश्यक राइट ऑफ
िे अनुमोदन सषहत षडषिटल अिसंरचना के षिथतार से संबंधी नीषतयों के अंगीकरर् के षलए सहायक होगा।
o राज्यों/के न्र्द् िाषसत प्रदेिों में षडषिटल संचार अिसंरचना और अनुकूल नीषतगत ईको-षसथटम की उपलब्धता का मापन
करने के षलए ब्रॉडबैंड रे डीनेस इंडेक्स (BRI) का सृिन करना।
• राष्ट्रीय षडषिटल संचार नीषत-2018 (NDCP-18) षडषिटल संचार अिसंरचना और सेिाओं को देि की उन्नषत एिं संपन्नता के
प्रमुख सामथ्यष तथा षनधाषरक के रूप में मान्यता प्रदान करती है। NDCP-18 के मुख्य उद्देश्यों में से एक “सभी के षलए ब्रॉडबैंड
सेिा” का प्रािधान करना है ताकक षिथतृत-व्यापक, न्यायोषचत और समािेिी षिकास के पररर्ामी लाभ देि के सभी लोगों को
प्राप्त हो सकें । “सभी के षलए ब्रॉडबैंड” के उद्देश्य को कायाषषन्ित करने तथा सािषभौषमक ब्रॉडबैंड पहुंच सुषनषित करने के षलए
“राष्ट्रीय ब्रॉडबैंड षमिन” का िुभारं भ ककया गया है। इसषलए कथन 2 सही है।

Q 75.B
• संथाली, ऑथिो-एषियारटक भािाओं के मुंडा उप-पररिार से संबद्ध सिाषषधक बोली िाने िाली भािा है। यह मुख्यत: असम,
षबहार, झारखंड, षमिोरम, ओषडिा, षत्रपुरा और पषिम बंगाल में बोली िाती है। इसके अषतररक्त, यह बांग्लादेि और नेपाल
में भी प्रचषलत है। इसे 92िें संषिधान संिोधन अषधषनयम द्वारा भारतीय संषिधान की आठिीं अनुसच
ू ी में समाषिष्ट ककया गया।
संथाली भािा की षलषप ओल-षचकक है, षिसे पंषडत रघुनाथ मुमूष द्वारा षिकषसत ककया गया। हाल ही में ओषडिा के एक
राज्यसभा सांसद द्वारा आिश्यक लोक महत्ि के मामले को उठाने के षलए संथाली भािा में सदन (राज्यसभा) को संबोषधत
ककया गया। इसषलए षिकलप (b) सही उत्तर है।

34 www.visionias.in ©Vision IAS


Q 76.A
• िायुमंडलीय दाब का धरातल पर षितरर् एक समान नहीं होता है। इसमें ऊध्िाषधर और क्षैषति दोनों प्रकार के षितरर् में षभन्न
होता है।
• िायुमंडलीय दाब के अक्षांिीय षितरर् को िैषश्वक क्षैषति षितरर् कहते हैं। इस षितरर् की षििेिता समदाब व्यिथथा या 'दाब
पेरटयों' के षिषिष्ट ज्ञात क्षेत्रों की उपषथथषत है।
• पृथ्िी के धरातल पर सात दाब पेरटयां पाई िाती हैं। ये सात दाब पेरटयां इस प्रकार हैं: षििुितीय षनम्न िायुदाब पेटी, उपोष्र्
उच्च िायुदाब पेटी, उप-ध्रुिीय षनम्न िायुदाब पेटी और ध्रुिीय उच्च िायुदाब पेटी। षििुितीय षनम्न िायुदाब पेटी को छोडकर,
अन्य सभी पेरटयां उत्तरी और दषक्षर्ी गोलाद्धों में सदृश्य युग्म में पाई िाती हैं।
• उत्पषत्त की प्रकक्रया के आधार पर िायुदाब की पेरटयों को दो िृहद् समूहों में षिभाषित ककया गया है:
o तापिन्य िायुदाब पेरटयां (षिसमें षििुितीय षनम्न िायुदाब पेटी और ध्रुिीय उच्च िायुदाब पेटी िाषमल है)। इसषलए कथन
1 सही नहीं है।
o गषतिन्य िायुदाब पेरटयां (षिसमें उपोष्र् उच्च िायुदाब पेटी और उप-ध्रुिीय षनम्न िायुदाब पेटी िाषमल है)
• षििुितीय षनम्न िायुदाब पेटी या 'डोलड्रम':
o यह पेटी 5°N और 5°S अक्षांिों के मध्य षथथत है, हालााँकक यह 5°N और 5°S से 20°N और 20°S के मध्य षिथतृत हो
सकती है। षििुित रे खीय क्षेत्र में अत्यषधक तापमान के कारर् िायु गमष हो िाती है और ऊपर उठती है। षिससे षििुितीय
षनम्न िायुदाब पेटी का षनमाषर् होता है। अत्यषधक षनम्न िायुदाब के साथ िांत षथथषतयां इस पेटी की षििेिताएं हैं।
o धरातलीय पिनें सामान्यतः अनुपषथथत होती हैं क्योंकक इस पेटी के षनकट आने िाली पिनें इसके सीमांत क्षेत्र के षनकट
ऊपर उठाने लगती हैं। इस प्रकार, इस पेटी में के िल ऊध्िाषधर पिनें (संिहन धाराएं) ही पाई िाती हैं। यह पेटी दोनों
गोलाद्धों में उपोष्र् उच्च िायुदाब पेटी से आने िाली व्यापाररक पिनों (trade winds) का अषभसरर् क्षेत्र है। अत्यषधक
िांत िायु प्रिाह के कारर् इस पेटी को डोलड्रम (िांत पेटी) भी कहते हैं। इसषलए कथन 2 सही है।
• उपोष्र् उच्च िायुदाब पेटी या 'अश्व अक्षांि (Horse Latitudes)':
o यह पेटी उत्तरी और दषक्षर्ी दोनों गोलाद्धों में 30°-35° अक्षांिों के मध्य पायी िाती है। इसषलए कथन 3 सही नहीं है।
o इस पेटी में उच्च दाब की उपषथथषत का मुख्य कारर् षििुित रे खीय क्षेत्र से आने िाली िायु के भारी होने से अितलन होना
है।
o इस पेटी में उच्च दाब कोररऑषलस बल के कारर् ऊपरी थतरों पर िायु के अिरुद्ध प्रभाि से भी होता है। अितषलत िायु गमष
और िुष्क होती है, इसषलए अषधकांि मरुथथल इस पेटी के साथ दोनों गोलाद्धों में पाए िाते हैं।
o अितषलत िायु धाराएं षनम्न दाब पेटी से संबद्ध पिन के साथ प्रिाषहत होने लगती हैं। इस क्षेत्र में पररितषनिील एिं मंद
पिनों के साथ िांत दिाएं पायी िाती हैं िो उच्च दाब पेटी का षनमाषर् करती हैं, षिसे अश्व अक्षांि (horse latitudes)
कहते हैं। प्राचीन काल में, घोडों से भरे पाल िाले िहाि िब इस पेटी से गुिरते थे तो यहां िायु की िांत दिाओं के कारर्
िहाि का आगे बढ़ना करठन हो िाता था। अतः िहाि को हलका करने के षलए कु छ घोडों को समुर्द् में फें क कदया िाता
था। इस पेटी में प्रायः उष्र्करटबंधीय और बषहरुष्र् षिक्षोभ का अषभसरर् होता है।
• उप-ध्रुिीय षनम्न िायुदाब पेटी: यह पेटी 45°N और S अक्षांिों से आकष रटक और अंटाकष रटक िृत्त के मध्य पायी िाती है। पछु आ
पिनों और ध्रुिीय पूिी पिनों के अषभसरर् के पररर्ामथिरूप ऊध्िाषधर प्रिाषहत िायु के कारर् इस पेटी में षनम्न दाब पाया
िाता है।
• िीत ऋतु के दौरान, थथल और सागर के मध्य अत्यषधक षभन्नता होने के कारर्, यह पेटी दो अलग-अलग षनम्न दाब कें र्द्ों में
षिभाषित हो िाती है - पहली एलयूषियन द्वीप समूह के षनकटिती क्षेत्र और दूसरी आइसलैंड और ग्रीनलैंड के मध्य।
• ग्रीष्म ऋतु के दौरान, थथल और सागर के मध्य कम षििमता होने के कारर् यह पेटी अषधक षिकषसत और षनयषमत होती है।
इसके अषतररक्त, इस पेटी में ध्रुिों और उपोष्र् िायुदाब क्षेत्रों से पिनें आकर षमलती हैं और ऊपर उठती हैं, इन पिनों के
तापमान और आर्द्षता में अत्यषधक षभन्नता होती है षिसके कारर्, इस क्षेत्र में चक्रिाती तूफान या षनम्न िायुदाब की दिाएं
उत्पन्न होती हैं।
• ध्रुिीय उच्च िायुदाब पेटी: ध्रुिों पर सबसे षनम्न तापमान पाया िाता है, िो िायु के अितलन का कारर् बनता है इसषलए यह
ध्रुिीय उच्च दाब का षनमाषर् करता है। ध्रुिीय उच्च दाब छोटे क्षेत्र में और ध्रुिों के चारों ओर षिथतृत होते हैं।

35 www.visionias.in ©Vision IAS


Q 77.D
• पूिी भारत में षथथत कृ ष्र्ा-गोदािरी बेषसन देि का सबसे बडा िेल गैस भंडार है।
• गुिरात में षथथत कै मबे बेषसन देि का सबसे बडा बेषसन है।

Q 78.D
• भूिल की कमी का मुख्य कारर् षनरंतर भूिल पंसपंग है। भूिल में कमी के कु छ नकारात्मक प्रभाि षनम्नषलषखत हैं:
o िल ताषलका में षगरािट: अत्यषधक पंसपंग के कारर् भूिल ताषलका में षगरािट हो सकती है तथा अब कु ओं से भूिल की
प्राषप्त नहीं हो रही है।
o िर्धषत लागत: िल ताषलका में षगरािट होने के कारर् अषधक ऊिाष का उपयोग करके िल को सतह तक लाने के षलए
अषधक पंप ककया िाता है। चरम पररषथथषतयों में, इस प्रकार के कु एं का उपयोग लागत-षनिेधात्मक हो सकता है।
o सतही िल आपूर्तष में कमी: भूिल और सतही िल आपस में संबद्ध होते हैं। िब भूिल का अत्यषधक उपयोग ककया िाता है
तब भूिल से संबद्ध झीलों, धाराओं और नकदयों द्वारा िल आपूर्तष कम हो िाती है। इसषलए कथन 1 सही है।
o भूषम अितलन: भूषम अितलन तब होता है िब धरातल के नीचे का आधार नष्ट हो िाता है। यह प्रायः मानिीय
गषतषिषधयों के कारर् होता है षिसमें मुख्य रूप से भूिल का अत्यषधक दोहन िाषमल है, षिससे मृदा संहत (compacts)
होती है और ढह िाती है। इसषलए कथन 2 सही है।
• िल की गुर्ित्ता संबषं धत समथयाएं: तटीय क्षेत्रों में अत्यषधक पसमपंग से लिर्ीय िल अन्तः क्षेत्रों में और ऊपर की ओर आ
सकता है, षिसके पररर्ामथिरूप िल आपूर्तष लिर्ीय िल से संदषू ित हो िाती है। िाथतषिक रूप में , रािथथान और महाराष्ट्र
िैसे कु छ राज्यों में भू-िल में फ्लोराइड की सांर्द्ता में िृषद्ध हुई है और इस पद्धषत के कारर् पषिम बंगाल और षबहार के भागों
में आसेषनक की सांर्द्ता में िृषद्ध हुई है। इसषलए कथन 3 सही है।

Q 79.A
• िैि आिधषन (Biomagnification) को बायोएषमप्लकफके िन या िैषिक आिधषन भी कहते हैं। यह ककसी खाद्य िृंखला में क्रषमक
रूप से उच्च पोिर् थतर पर सहनिील िीिों के ऊतकों में ककसी भी प्रकार के षििाक्त पदाथष िैसे कक कीटनािकों का सांर्द्र् है।
िैि आिधषन खाद्य िृंखला में बढ़ते पोिर् थतरों में एक षििाक्त पदाथष (िैसे- कीटनािक, धातुए,ं आकद) की सांर्द्ता में संचयी
िृषद्ध को संदर्भषत करता है। यह तब घरटत होता है िब कृ षि, औद्योषगक अथिा मानि अपषिष्ट प्रत्यक्ष रूप से समुर्द् में अथिा
नकदयों, सीिेि आकद के माध्यम से समुर्द् में प्रिाषहत कर कदए िाते हैं। यह िृषद्ध षििाक्त पदाथों के थथाषयत्ि के पररर्ामथिरूप

36 www.visionias.in ©Vision IAS


हो सकती है - षिसमें पयाषिरर्ीय प्रकक्रयाओं और िल-अघुलनिीलता द्वारा पदाथष को षिखंषडत नहीं ककया िा सकता है।
इसषलए कथन 1 सही है।
• िैि संचायक (Bioaccumulants) ऐसे पदाथष होते हैं षिनका सिीिों में सांर्द्र् बढ़ता है। इन पदाथों को िीषित िीि संदषू ित
िायु, िल या भोिन में अंतगृषहीत करते हैं। इसका कारर् यह है कक इन पदाथों का अत्यंत मंद गषत से उपापचय या उत्सिषन
होते हैं।
• िैि आिद्धषन होने के षलए प्रदूिक को थथायी रूप से बने रहना चाषहए, उसे गषतिील और िसा में घुलनिील होना चाषहए।
यकद प्रदूिक अलपकाषलक है, तब हाषनकारक होने से पूिष ही यह षिखंषडत हो िाएगा। इसषलए कथन 2 सही नहीं है।

Q 80.C
• कथन 1 सही है: पृथ्िी पर चादरों के रूप में षहम प्रिाह या पिषतीय ढालों से घारटयों में रै षखक प्रिाह के रूप में प्रिाषहत षहम
संहषत को षहमनद कहते हैं। महाद्वीपीय षहमनद या षगररपद षहमनद िे षहमनद हैं िो िृहद् षगररपदीय मैदान पर षहम परत के
रूप में षिथतृत हों तथा पिषतीय या घाटी षहमनद िे षहमनद हैं िो पिषतीय ढालों में प्रिाषहत होते हैं।
• कथन 2 सही है: प्रिाषहत िल के षिपरीत षहमनद प्रिाह मंद गषत से होता है। षहमनद प्रषत कदन कु छ सेंटीमीटर से कु छ मीटर
या इससे कम या अषधक तक प्रिाषहत हो सकते हैं। षहमनद मुख्यतः गुरुत्िाकिषर् बल और ऊपरी षहम के अत्यषधक भार से
उत्पन्न दाब के कारर् गषतमान होते हैं। षहमनदों में प्रबल अपरदन होता है षिसका कारर् इसके अपने भार से उत्पन्न घिषर् है।
षहमनद अपक्षय रषहत चट्टानों का भी प्रभाििाली रूप से अपरदन करते हैं षिससे ऊाँचे पिषत छोटी पहाषडयों और मैदानों में
पररिर्तषत हो िाते हैं।

Q 81.D
• एक आक्रामक पादप प्रिाषत पयाषिास को व्यापक रूप से पररिर्तषत कर देती है। यह अत्यषधक मात्रा में पोिक तत्िों एिं आर्द्षता
का उपयोग कर देिि प्रिाषतयों के षिकास को बाषधत करती है तथा सूक्ष्मिीिों, कीटों और रोगिनकों के पुनः आक्रमर् हेतु
मागष प्रिथत करती है। इन आक्रामक षिदेिी प्रिाषतयों (IAS) के प्रसार से थथाषनक िैि षिषिधता के समक्ष खतरा उत्पन्न होता
है, प्राकृ षतक पादप अनुक्रमर् षिकृ त हो िाता है, पयाषिास की संरचना और संघटन में पररितषन होता है तथा पाररषथथषतकी तंत्र
सेिाओं को गंभीर रूप से क्षषत पहुंचती है।
• आक्रामक षिदेिी प्रिाषतयों के रूप में पुष्पीय प्रिाषतयों में से कु छ भारत में सचंता का कारर् बनी हुई हैं:
o प्रोसोषपस िूलीफ्लोरा (Prospis julflora): षिलायती कीकर (प्रोसोषपस िूलीफ्लोरा) देिि िनथपषतयों के षनम्नीकरर्
और मृदा के पोिर् के ह्रास का कारर् बनता है। यह िुष्क पररषथथषतयों में बेहतर रूप से षिकषसत होता है और इसमें
लिर्ता को सहन करने की पयाषप्त क्षमता होती है। िृक्ष के सघन षितान (canopy) अन्य िृक्ष प्रिाषतयों की िृषद्ध और
पुनरुत्पादन को अिरुद्ध करते हैं, षिससे क्षेत्र की प्रिातीय समृषद्ध (species richness) में कमी आती है। यह अपनी
कं टीली िाखाओं के कारर् पिुओं को चारे की प्राषप्त हेतु षिचरर् करने में करठनाई उत्पन्न करता है। इसका एक सुदढ़ृ बीि
पररक्षेपर् तंत्र (dispersal mechanism) है और यह सूयष के प्रकाि को अिरुद्ध करने के साथ रसायनों को स्राषित करते
हुए देिि पौधे के षिकास को प्रषतकू ल रूप से प्रभाषित करता है।
o पाथेषनयम षहथटेरोफोरस (Parthenium hysterophorus): इसे बोल-चाल की सामान्य भािा में कांग्रेस घास या गािर
घास के रूप में िाना िाता है। यह माना िाता है कक इसे भारत और ऑथिेषलया में उत्तरी अमेररका से लाया गया। यह
अफ्रीका, एषिया और ऑथिेषलया में सातिें सिाषषधक षिनािकारी खरपतिार के रूप में उभरा है, यह िैि षिषिधता के
षलए एक गंभीर समथया होने के अषतररक्त, मानि और िंतओं
ु के थिाथथ्य संबंधी गंभीर समथयाओं, िैसे त्िचािोथ
(डमेटाइरटस), अथथमा और ब्रोंकाइरटस तथा कृ षि क्षषत के षलए उत्तरदायी है।
o लैंटाना कै मारा (Lantana camara): भारत में प्रषिष्ट एक अन्य आक्रामक षिदेिी प्रिाषत है। इसका षिषिधतापूर्ष एिं
व्यापक भौगोषलक षितरर् इसकी व्यापक पाररषथथषतक सहनिीलता (ecological tolerance) को प्रषतसबंषबत करता है।
यह षिषभन्न पयाषिासों में और षिषभन्न प्रकार की मृदाओं पर उगने में सक्षम होती है। लैंटाना अगमय सघन झाडी के रूप में
षिकषसत होती है, िो देिि प्रिाषतयों के षिकास को अिरुद्ध कर सकती है। लैंटाना प्राकृ षतक पयाषिासों और देिि
िनथपषत एिं िंतओं
ु के समक्ष खतरा उत्पन्न करती है (FAO)।

37 www.visionias.in ©Vision IAS


o िाटर हायाससंथ (इकार्नषया क्रासाइप्स: Eichhornia crassipes) को िलकुं भी के रूप में भी िाना िाता है। यह अपने
अत्यषधक प्रसार के कारर् खुले िल की सतह को आच्छाकदत कर लेती है। इसका िृहत् थतर पर प्रसार सूयष के प्रकाि के
प्रिेि को षनषिद्ध करता है और िायु-िल के इं टरफे स को महत्िपूर्ष रूप से पररिर्तषत करता है तथा पाररषथथषतकी तंत्र की
उत्पादकता को प्रषतकू ल रूप से प्रभाषित करता है।

Q 82.A
• लाल मृदाओं का षिकास दक्कन पठार के पूिी और दषक्षर्ी भाग में षनम्न ििाष िाले क्षेत्रों में रिेदार आिेय चट्टानों पर होता है।
उच्च तापमान और उच्च ििाष िाले क्षेत्रों में लैटेराइट मृदा का षिकास होता है। इसषलए कथन 1 सही नहीं है।
• उत्तरी मैदानों और नदी घारटयों में िलोढ़ मृदा का व्यापक प्रसार हुआ है। ये मृदाएं देि के कु ल क्षेत्रफल के लगभग 40 प्रषतित
भाग में षिथताररत हैं। इस प्रकार िलोढ़ मृदाएं भारत में मृदा के सिाषषधक िृहत् समूह का षनमाषर् करती है , न कक लाल एिं
पीली मृदाएं। इसषलए कथन 2 सही नहीं है।
• इन मृदाओं में लाल रं ग चट्टानों में लौह तत्ि के व्यापक षिसरर् के कारर् षिकषसत होता है। इनका पीला रं ग िलयोिन के
कारर् होता है। इसषलए कथन 3 सही है।
• महीन कर्ों िाली लाल एिं पीली मृदा सामान्य रूप से उपिाऊ होती है, िबकक िुष्क उच्च भूषम क्षेत्रों में पाई िाने िाली थथूल
कर्ों िाली मृदा में उिषरता षनम्न होती है। इनमें सामान्यतः नाइिोिन, फॉथफोरस और नयूमस की कमी होती है।

Q 83.D
• टटषल और टॉरटॉइि (कच्छपों की प्रिाषतयां) के व्यािसाषयक दोहन के साथ-साथ िीषित प्रिाषतयों के अिैध व्यापार में संलि
अिैध िन्यिीि व्यापाररयों की बढ़ती गषतषिषधयों को ध्यान में रखते हुए िन्यिीि अपराध षनयंत्रर् ब्यूरो (WCCB) ने
“ऑपरे िन सेि कू मष” नामक एक प्रिाषत षिषिष्ट कारष िाई की पररकलपना की। इसका उद्देश्य षििेितया उन प्रमुख राज्यों पर
ध्यान कें कर्द्त करना है, िहां िीषित कच्छप प्रिाषतयों का अत्यषधक षिकार, पररिहन ि अिैध व्यापार होता है। अतः युग्म 1
सही सुमषे लत नहीं है।
• देि में षतब्बती ऐन्टलोप (िाहतूि) के अिैध व्यापार की ओर प्रितषन एिेंषसयों का ध्यान आकर्िषत करने हेतु WCCB द्वारा 1
अक्टूबर 2018 से 31 माचष 2019 तक “ऑपरे िन सॉफ्ट गोलड” का संचालन ककया गया था। ध्यातव्य है कक प्रितषन कायषिाषहयों
के दौरान इस मृग के अिैध व्यापार को रोकने हेतु सुदढ़ृ प्रयास नहीं ककए िाते थे।
• ‘ऑपरे िन थंडरबडष’ इं टरपोल (INTERPOL) की बहु-राष्ट्रीय और बहु-प्रिातीय प्रितषन कारष िाई का कू ट नाम है। ऑपरे िन
थंडरबडष की सफलताओं को आगे बढ़ाने के क्रम में "ऑपरेिन थंडरथटॉमष" नामक इं टरपोल समषन्ित प्रितषन ऑपरे िन आरमभ
ककया गया था, षिसमें WCCB ने इसी अिषध के दौरान राष्ट्रव्यापी “मलटी-थपीिीि िाइलडलाइफ ऑपरे िन” का समन्ियन
ककया था। इसषलए युग्म 2 सही सुमषे लत नहीं है।
• देि में ई-कॉमसष प्लेटफॉमष के माध्यम से अिैध व्यापार के खतरे का सामना करने हेतु, WCCB ने प्रितषन एिेंषसयों का ध्यान
आकर्िषत करने के षलए “ऑपरे िन िाइलडनेट (WILDNET)” नामक एक प्रितषन अषभयान का आयोिन एिं समन्ियन ककया
था। इसका उद्देश्य इं टरनेट पर षनरंतर बढ़ते अिैध िन्यिीि व्यापार पर प्रितषन एिेंषसयों का ध्यान कें कर्द्त कराना था। इसषलए
युग्म 3 सही सुमषे लत नहीं है।
• ऑपरे िन षबरषबल (BIRBIL) का उद्देश्य सभी पषक्षयों और षबलली प्रिाषतयों के अिैध व्यापार से संबद्ध आसूचना एकत्र करना
था।
• WCCB द्वारा एक िन्यिीि प्रितषन कायषिाही "ऑपरे िन लेथकनो” (LESKNOW) में सहयोग ककया गया था। इस ऑपरे िन
का उद्देश्य देि में अलपज्ञात प्रिाषतयों के अिैध व्यापार की ओर प्रितषन एिेंषसयों का ध्यान आकर्िषत कराना था।
• ब्यूरो ने “ऑपरे िन लेथकनो-2 (LESKNOW-II)” कू ट नाम से एक अन्य िन्यिीि प्रितषन ऑपरे िन को भी पररकषलपत एिं
समन्िषयत ककया था। ऑपरे िन का उद्देश्य देि में मृग, िंगली सूकर, षसयार, नेिले, मॉषनटर षलिाडष, सी कु कु मबर (समुर्द्ी
खीरा) और पैंगोषलन िैसी अलपज्ञात प्रिाषतयों के अिैध व्यापार की ओर प्रितषन एिेंषसयों का ध्यान कें कर्द्त कराना था।
38 www.visionias.in ©Vision IAS
Q 84.B
• चंबल नदी का उद्गम मध्य प्रदेि के मालिा पठार षथथत महू के षनकट से होता है तथा यह उत्तरमुखी होकर एक महाखड्ड से
प्रिाषहत होते हुए रािथथान के कोटा में प्रिेि करती है। चंबल और उसकी सहायक नकदयााँ उत्तर-पषिमी मध्य प्रदेि के मालिा
क्षेत्र से अपिाषहत होती हैं तथा साथ ही रािथथान के दषक्षर्-पूिी भाग से इसकी सहायक नदी, बनास (षिसका उद्गम अरािली
िेर्ी से होता है) अपिाषहत होती है। चंबल अपनी उत्खात भूषम थथलाकृ षत (badland topography) के षलए प्रषसद्ध है, षिसे
चंबल बीहड (ravines) कहा िाता है।
• चंबल नदी पर कई बांधों का षनमाषर् ककया गया है। नदी के िल का उपयोग िलषिद्युत उत्पादन के षलए ककया िाता है। इन
बांधों में गांधी सागर बांध, ििाहर सागर बांध और रार्ा प्रताप सागर बांध िाषमल हैं।
• कोटेश्वर बांध भारत के उत्तराखंड के रटहरी षिले में रटहरी बांध के अनुप्रिाह क्षेत्र (downstream) में षथथत भागीरथी नदी पर
षनर्मषत एक गुरुत्िाकिषर् बांध है।
• भीमगढ़ बांध अथिा भीमगढ़ (षिसे आषधकाररक रूप से ऊपरी िैनगंगा बांध भी कहा िाता है), मध्य प्रदेि के षसिनी षिले में
िैनगंगा नदी पर षनर्मषत ककया गया है।

Q 85.C
• प्रिासन हेतु के षलए उत्तरदायी कारर्ों को दो व्यापक िेषर्यों में िाषमल ककया िा सकता है: (i) प्रषतकिष कारक (push
factors), षिसके कारर् लोग अपने षनिास थथान या मूल थथल को छोडते हैं; और (ii) अपकिष कारक (pull factors), िो
षिषभन्न थथानों से लोगों को आकर्िषत करते हैं।
• भारत में लोग ग्रामीर् क्षेत्रों से नगरीय क्षेत्रों में मुख्यतः षनधषनता, कृ षि भूषम पर उच्च िनसंख्या दबाि, थिाथथ्य देखभाल
सेिाओं, षिक्षा िैसी मूलभूत अिसंरचना सुषिधाओं का अभाि आकद के कारर् प्रिास करते हैं। इन कारकों के अषतररक्त बाढ़,
सूखा, चक्रिाती तूफान, भूकंप, सुनामी िैसी प्राकृ षतक आपदाएं, युद्ध और थथानीय संघिष भी प्रिास को बढ़ाती हैं।
• दूसरी ओर, अपकिष कारक, लोगों को ग्रामीर् क्षेत्रों से नगरों की ओर आकर्िषत करते हैं। नगरीय क्षेत्रों की ओर प्रिास के षलए
अषधकांि ग्रामीर् प्रिाषसयों हेतु सिाषषधक महत्िपूर्ष अपकिष कारक बेहतर अिसर, कायष की षनयषमत उपलब्धता और सापेषक्षक
रूप से अषधक मिदूरी है। षिक्षा के बेहतर अिसर, बेहतर थिाथथ्य सुषिधाएाँ और मनोरं िन के साधन आकद भी अत्यंत महत्िपूर्ष
अपकिष कारक हैं।
• इसषलए षिकलप (c) सही उत्तर है।

Q 86.A
• पयाषिरर्, िन और िलिायु पररितषन मंत्रालय द्वारा ई-अपषिष्ट (प्रबंधन और षनपटान) षनयम, 2011 को प्रषतथथाषपत करते
हुए, 23 माचष 2016 को ई-अपषिष्ट (प्रबंधन) षनयम, 2016 को अषधसूषचत ककया गया।
• ई-अपषिष्ट (प्रबंधन) षनयम, 2016
o षनयमों में षनमाषता, षिक्रेता, ररफर्बषिर और षनमाषता षिममेदारी संगठन (PRO) को अषतररक्त षहतधारकों के रूप में प्रथतुत
ककया गया है। अनुसच
ू ी I में सूचीबद्ध उपकरर्ों के अषतररक्त इलेषक्िकल और इलेक्िॉषनक उपकरर्ों (EEE) के घटकों,
उपभोग्य सामषग्रयों, कलपुिों एिं षहथसों के संदभष में षनयमों की प्रयोज्यता को षिथताररत ककया गया है।
o कॉमपैक्ट फ्लोरोसेंट लैंप (CFL) और अन्य पारा युक्त लैंपों को षनयमों के दायरे अंतगषत लाया गया है।
o षिथताररत षनमाषता षिममेदारी (Extended Producer Responsibility: EPR) के तहत उत्पादकों द्वारा ई-अपषिष्ट के
एकत्रीकरर् हेतु संग्रहर् कें र्द्, संग्रहर् सबंद,ु टेक बैक षसथटम (पुनिाषपसी प्रर्ाली) आकद को सषममषलत करने के षलए संग्रहर्
तंत्र आधाररत दृषष्टकोर् को अपनाया गया है।
o षडपॉषिट ररफं ड थकीम को एक अषतररक्त आर्थषक साधन के रूप में प्रथतुत ककया गया है, षिसमें षनमाषता इलेषक्िकल और
इलेक्िॉषनक उपकरर्ों की षबक्री के समय िमा राषि के रूप में एक अषतररक्त राषि िसूल करता है और इन उपकरर्ों को
इनकी पररचालन अिषध के अंत में िापस करने पर उपभोक्ता को ब्याि सषहत लौटाता है।

39 www.visionias.in ©Vision IAS


o षनयमों के अंतगषत एक षिकलप के रूप में ई-िेथट एक्सचेंि का प्रािधान ककया गया है। यह एक्सचेंि इन षनयमों के अंतगषत
अषधकृ त एिेंषसयों या संगठनों के मध्य एंड-ऑफ-लाइफ इलेषक्िकल और इलेक्िॉषनक उपकरर्ों से उत्पन्न ई-अपषिष्ट की
षबक्री और खरीद के षलए सहायता प्रदान करने िाले थितंत्र बािार उपकरर् या सेिाएं प्रदान करने िाली थितंत्र
इलेक्िॉषनक प्रर्ाषलयों के रूप में थथाषपत ककया िाएगा।
o षनयमों में षिघटन और पुनचषक्रर् कायों में संलि िषमकों की सुरक्षा, उनके थिाथथ्य और कौिल षिकास को सुषनषित करने
के षलए राज्य सरकार की भूषमका भी षनधाषररत की गई है।
o षनयमों के प्रािधानों के उललंघन के षलए षित्तीय िुमाषने के आरोपर् संबंधी प्रािधानों सषहत ई-अपषिष्ट के अनुपयुक्त
प्रबंधन के कारर् पयाषिरर् या तीसरे पक्ष को होने िाली क्षषत संबध
ं ी दाषयत्िों को भी िाषमल ककया गया हैं।
o िहरी थथानीय षनकायों (नगरपाषलका सषमषत / पररिद / षनगम) को ऑफष न प्रोडक्ट को एकत्र करने और इन्हें अषधकृ त
षिघटनकताष (षडथमेंटलर) या पुनचषक्रर्कताष (ररसाइक्लर) को सुपुदष करने का (चैनलाइि) का दाषयत्ि सौंपा गया
है।

Q 87.D
• भारत का उत्तरी मैदान ससंधु, गंगा और ब्रह्मपुत्र नकदयों द्वारा बहाकर लाए गए िलोढ़ षनक्षेप द्वारा षनर्मषत है। इस मैदान की
पूिष से पषिम तक लंबाई लगभग 3,200 ककमी है। इन मैदानों की औसत चौडाई 150-300 ककमी के मध्य है। िलोढ़ षनक्षेप की
अषधकतम मोटाई 1,000-2,000 मीटर के मध्य है। उत्तर से दषक्षर् तक, इस मैदान को तीन प्रमुख भागों में षिभाषित ककया िा
सकता है: भाभर, तराई और िलोढ़ मैदान। िलोढ़ मैदानों को खादर और बांगर में षिभाषित ककया िा सकता है। इसषलए
षिकलप (d) सही है।
• भाभर क्षेत्र 8 से 10 ककमी चौडाई की पतली पट्टी में षिथतृत है िो षििाषलक षगरीपद के समानांतर षिथताररत है। इसके
पररर्ामथिरूप षहमालय पिषत िेषर्यों से बाहर षनकलती नकदयां यहां पर भारी िलीय-भार िैसे बडे िैल और गोलाश्म को
षनक्षेषपत कर देती है और कभी-कभी थियं इसी क्षेत्र में षिलुप्त भी हो िाती है। भाभर के दषक्षर् में तराई क्षेत्र षथथत है षिसकी
चौडाई 10 से 20 ककमी है। भाभर क्षेत्र में लुप्त नकदयां इस क्षेत्र में धरातल पर पुन: प्रकट हो िाती हैं और चूंकक इनकी कोई
षनषित िाषहकाएं नहीं होती इसषलए ये क्षेत्र अनूप बन िाते हैं षिसे तराई कहा िाता है। यह क्षेत्र षिषभन्न प्रकार की
िनथपषतयों से आच्छाकदत रहता है और षिषभन्न प्रकार के िन्यिीिों का पयाषिास क्षेत्र है। तराई के दषक्षर् में पुराने िलोढ़
(बांगर) और नए िलोढ़ (खादर) षनक्षेपों से षनर्मषत एक पट्टी षथथत है। इस मैदान में नकदयों की प्रौढ़ अिथथा में बनने िाली
अपरदनात्मक एिं षनक्षेपात्मक थथलाकृ षतयां िैसे बालू-रोषधका, षिसपष, गोखुर झीलें और गुषमफत नकदयां पायी िाती हैं।
ब्रह्मपुत्र घाटी का मैदान अपने नदीय द्वीपों और बालू-रोषधकाओं के षलए िाना िाता है। यहााँ अषधकांि क्षेत्र में समय-समय पर
बाढ़ आती है और नकदयां अपना मागष पररिर्तषत कर गुंकफत िाषहकाओं का षनमाषर् करती हैं।

Q 88.A
• बार-हेडड
े गूज़ (Anser indicus) एक हंस (goose) है। ये मध्य एषिया में पिषतीय झीलों के षनकट तथा दषक्षर्-एषिया
(प्रायद्वीपीय भारत तक) में िीतकाल के दौरान हिारों के समूह (colony) में प्रिनन करते हैं। यह धरातल पर षनर्मषत घोंसले में
एक समय में तीन से आठ अंडे देते हैं।
• बार-हेडड
े गूज़ प्रत्येक ििष मंगोषलया से भारत में प्रिास करने के क्रम में षहमालय की सबसे ऊंची चोरटयों को पार करते हैं। िब
षिरल िायु में ऑक्सीिन का थतर षगरकर 7 प्रषतित पर पहुाँच िाता है, तो इसके साथ अनुकूषलत होने के षलए पषक्षयों की
40 www.visionias.in ©Vision IAS
उपापचय कक्रया (metabolism) में भी षगरािट हो िाती है, कफर भी ये पूिष की भांषत उडते रहते हैं। बार-हेडेड गूज़ 26,000
फीट से अषधक ऊाँचाई पर उडते हैं। उललेखनीय है कक इनके द्वारा ककए िा रहे ऑक्सीिन के कु िल उपयोग के संबंध में बेहतर
समझ षिकषसत करने से मानि षचककत्सा उपचार क्षेत्र में सकारात्मक पररर्ाम प्राप्त हो सकते हैं।
• िैज्ञाषनकों को लंबे समय से यह ज्ञात है कक बार-हेडेड गूज़ में हीमोग्लोषबन के साथ ऑक्सीिन को आबद्ध करने की उन्नत क्षमता
होती है। अध्ययनों से ज्ञात हुआ है कक बानेकल गीज़ और अन्य संबंषधत प्रिाषतयों (िो इनकी भांषत अषधक ऊाँचाई पर नहीं उड
सकते हैं) की तुलना में बार-हेडेड गूज़ में पेक्टोरल मांसपेषियों में षथथत एकल कोषिकाओं के चतुर्दषक अषधक के षिकाएाँ
(capillaries) षिद्यमान होती हैं। ज्ञातव्य है कक इन कोषिकाओं में माइटोकॉषन्ड्रया की संख्या भी अषधक होती है।
माइटोकॉषन्ड्रया कोषिका को ऊिाष की आपूर्तष हेतु ऑक्सीिन का उपयोग करता है। इसके अषतररक्त, षहमालय के ऊपर से उडते
समय बार-हेडेड गूज़ घारटयों के ऊपर अपेक्षाकृ त अषधक ऑक्सीिन युक्त िायु का लाभ उठाते हुए अपनी उडान को भूषम के
षनकट बनाए रखते हैं।
• इन्हे सामान्यतः ‘पषक्षयों की दुषनया के एथिोनॉट’ के रूप में िाना िाता है।

Q 89.A
• भारत में कु छ ऐषतहाषसक पयाषिरर्ीय षिधानों में षनम्नषलषखत सषममषलत हैं:
• भारत सरकार द्वारा ििष 1973 में 'प्रोिेक्ट टाइगर' की िुरुआत के साथ ही राष्ट्रीय पिु बाघ के संरक्षर् की कदिा में एक अग्रर्ी
पहल की गयी थी। उललेखनीय है कक प्रोिेक्ट टाइगर की िुरुआत के समय देि में नौ बाघ आरषक्षत क्षेत्र थे , िहीं ितषमान में
इनकी संख्या बढ़कर पचास हो गई है, षिनका षिथतार बाघ की रें ि िाले 18 राज्यों में है। बाघ आरषक्षत क्षेत्र का कु ल क्षेत्रफल
देि के कु ल भौगोषलक क्षेत्रफल का लगभग 2.21% है। प्रोिेक्ट टाइगर की िुरुआत ििष 1973 में भारतीय िन्यिीि बोडष के
एक षििेि कायष बल की अनुिंसाओं के आधार पर की गई थी। इसका प्रमुख उद्देश्य:
o भारत में बाघों की उपलब्ध आबादी के अनुरक्षर् को सुषनषित करना है।
o इस प्रकार के िैषिक महत्ि के क्षेत्रों को लोगों के लाभ, षिक्षा और मनोरं िन के षलए राष्ट्रीय धरोहर के रूप में संरषक्षत
करना है।
• राज्य के िन्यिीि संपदा के संरक्षर् हेतु मूलय एिं आिश्यकता को थिीकार करते हुए, तषमलनाडु सरकार ने “िन्यिीि (संरक्षर्)
अषधषनयम, 1972” के अषधषनयषमत होने से पूिष ही राज्य के बहुमूलय िन्यिीिों के संरक्षर् हेतु एक पृथक कानून (तषमलनाडु
िाइलड एलीफे ट षप्रििेिन एक्ट, 1873 और िंगली पक्षी और पिु (संरक्षर्) अषधषनयम, 1912) को अषधषनयषमत ककया गया
था। ििष 1936 में घोषित “िेदांथग
ं ल पक्षी अभयारण्य” भारत का "प्रथम" पक्षी अभयारण्य था। देि में ििष 1973 में प्रोिेक्ट
टाइगर की िुरुआत होने से लगभग ग्यारह ििष पूिष ििष 1962 में मुंडनथुराई में ‘बाघों के षलए अभयारण्य’ की घोिर्ा की गयी
थी।
• पषिमी घाट (भारत) का एक षनरुषपत क्षेत्र नीलषगरर िीिमंडल आरषक्षत क्षेत्र (Nilgiri Biosphere Reserve: NBR) 5670
िगष ककमी क्षेत्र में षिथतृत है। इसे ििष 1986 में भारत के प्रथम िीिमंडल आरषक्षत क्षेत्र (बायोथफीयर ररििष) के रूप में थथाषपत
ककया गया था। यद्यषप 1971 में यूनथे को द्वारा बायोथफीयर ररििष कायषक्रम की िुरुआत की गई थी, तथाषप भारत में 1986 में
प्रथम बायोथफीयर ररििष (नीलषगरर बायोथफीयर ररििष) की घोिर्ा की गई थी।
• फरिरी, 1992 में प्रोिेक्ट एषलफें ट की िुरुआत िंगली हाषथयों की आबादी के षलए थितंत्र क्षेत्र थथाषपत करने हेतु राज्यों की
सहायता करने के षलए की गई थी ताकक हाषथयों की उनके प्राकृ षतक पयाषिासों में एक षनधाषररत व्यिहायष आबादी के
दीघषकाषलक अषथतत्ि को बनाए रखा िा सके ।

Q 90.B
• “प्रिाषत षिषिधता (Species diversity)” एक ऐसा िब्द है षिसका उपयोग षिषिष्ट पाररषथथषतकीय समुदाय में पायी िाने
िाली िैषिक षिषिधता के मापन का िर्षन करने हेतु ककया िाता है। इसके अंतगषत एक पाररषथथषतक समुदाय में पाई िाने
िाली प्रिाषतयों की समृषद्ध या प्रिाषतयों की संख्या, प्रिाषतयों की प्रचुरता (या प्रषत प्रिाषत सदथयों की संख्या) और षितरर्
या समरूपता िाषमल होती है।
• िीतोष्र् क्षेत्रों की तुलना में उष्र्करटबंधीय क्षेत्रों में िैषिक षिषिधता अषधक पाई िाती है क्योंकक:

41 www.visionias.in ©Vision IAS


o िाषत उद्भिन (Speciation) सामान्य तौर पर समय-आधाररत पररघटना है। िीतोष्र् क्षेत्रों में प्राचीन समय से बारमबार
षहमनदन (glaciations) होता रहा है िबकक उष्र्करटबंधीय क्षेत्र में लाखों ििों से इस प्रकार के षिक्षोभ नहीं हुए हैं, इस
कारर् यहााँ प्रिाषत के षिकास और षिषिधता के षलए लंबा समय उपलब्ध होता है। इसषलए षिकलप 3 सही नहीं है।
o िीतोष्र् पयाषिरर् के षिपरीत उष्र् करटबंधीय पयाषिरर् में कम मौसमी पररितषन घरटत होते हैं और यह सापेषक्षक रूप से
अषधक षथथर और पूिाषनम
ु य
े होता है। इस प्रकार यह षथथर पयाषिरर् षनके त (niche) षिषिष्टीकरर् को उत्प्रेररत करता रहा
है षिसके कारर् अत्यषधक प्रिाषत षिषिधता को बढ़ािा षमला है। इसषलए षिकलप 1 सही नहीं है।
o उष्र् करटबंधीय क्षेत्र में अषधक सौर ऊिाष उपलब्ध होती है, षिससे उत्पादकता अषधक होती है; षिसके पररर्ामथिरूप
परोक्ष रूप से अषधक िैि-षिषिधता को बढ़ािा षमलता है। इसषलए षिकलप 2 सही है।

Q 91.C
• भारतीय िलमागष प्राषधकरर् ने षिश्व बैंक-सहायता प्राप्त ‘िल मागष षिकास पररयोिना’ के भाग के रूप में राष्ट्रीय िलमागष -1
(गंगा नदी) पर षनर्मषत होने िाले पहले बहु-मॉडल टर्मषनलों का उद्घाटन निंबर 2018 को िारार्सी में ककया था।
• इस िल मागष षिकास पररयोिना में तीन बहु-मॉडल टर्मषनलों (िारार्सी, साषहबगंि और हषलदया), दो अंतर-मॉडल टर्मषनलों,
पांच रोल-ऑन-रोल-ऑफ (Ro-Ro) टर्मषनल युग्मों, फरक्का, पषिम बंगाल में नए नेषिगेिन लॉक का षनमाषर् िाषमल है। इसमें
तलकिषर् की गहराई, एकीकृ त पोत मरममत और रखरखाि सुषिधा, अंतर िैषश्वक षथथषत प्रर्ाली (Differential Global
Positioning System: DGPS), नदी सूचना प्रर्ाली (River Information System: RIS) और नदी प्रषिक्षर् को
सुषनषित करना िाषमल है।

Q 92.D
• प्रायद्वीपीय भारत के कु छ भागों के षिितषषनक कक्रयाओं से पररिर्तषत होने के उपरांत भी इस भूखड
ं का िाथतषिक आधार तल
अप्रभाषित रहा है। इं डो-ऑथिेषलयन प्लेट का षहथसा होने के कारर् यह उध्िाषधर हलचलों ि खंड भ्रंि से प्रभाषित है।
• भ्रंि घाटी भूगभीय दरार या भ्रंि की कारषिाई से षनर्मषत की गई अनेक उच्च भूषमयों या पिषत िृंखलाओं के मध्य एक रै षखक
आकार की षनम्नभूषम है। भ्रंि घाटी एक अपसारी प्लेट सीमा, षिभंग षिथतार या सतह के अपसारी प्रसार के साथ बनती है,
षिसे बाद में अपरदनात्मक बल द्वारा और गहरा ककया िाता है।
• नमषदा, ताप्ती और दामोदर एक भ्रंि घाटी से होकर प्रिाषहत होती हैं। इसषलए षिकलप (d) सही है।
• नमषदा नदी अमरकं टक पठार के पषिमी पाश्वष से लगभग 1,057 मीटर की ऊंचाई से षनकलती है। यह दषक्षर् में सतपुडा और
उत्तर में सिंध्याचल िेषर्यों के मध्य यह भ्रंि घाटी से बहती हुई संगमरमर की चट्टानों में सुन्दर महाखड्ड, और िबलपुर के षनकट
धुआाँधार िलप्रपात का षनमाषर् करती है। यह लगभग 1,312 कक.मी. की दूरी तक प्रिाषहत होने के बाद यह भडौच के दषक्षर् में
अरब सागर में षमलती है और 27 कक.मी. लंबा ज्िारनदमुख षनर्मषत करती है।
• ताप्ती पषिम कदिा में बहने िाली एक अन्य महत्िपूर्ष नदी है। यह मध्य प्रदेि में बैतल
ू षिले में मुलताई से षनकलती है। यह 724
कक.मी. लंबी नदी है और लगभग 65,145 िगष कक.मी. क्षेत्र को अपिाषहत करती है। इसके अपिाह क्षेत्र का 79 प्रषतित भाग
महाराष्ट्र में, 15 प्रषतित भाग मध्य प्रदेि में और िेि 6 प्रषतित गुिरात में अिषथथत है।
• छोटानागपुर पठार के पूिी ककनारे पर दामोदर नदी बहती है और भ्रंि घाटी से होती हुई हुगली नदी में षगरती है। बराकर
इसकी मुख्य सहायक नदी है। कभी ‘बंगाल का िोक' कही िाने िाली इस नदी को अब दामोदर घाटी कॉपोरे िन नामक एक
बहुउद्देिीय पररयोिना के अंतगषत षनयंषत्रत ककया गया है।

Q 93.A
• द ज़ीरो िेथट इं टरनेिनल अलायंस को ठोस अपषिष्ट भराि और भथमीकरर् हेतु सकारात्मक षिकलपों को प्रोत्साहन प्रदान करने
के षलए तथा उन सामाषिक एिं आर्थषक लाभों के प्रषत सामुदाषयक िागरूकता को बढ़ाने हेतु थथाषपत ककया गया है , िो
अपषिष्ट को एक संसाधन आधार माने िाने पर प्राप्त होंगे तथा षिन पर रोिगार और व्यिसाय दोनों के अिसरों का षनमाषर्
ककया िा सकता है। द ज़ीरो िेथट इं टरनेिनल अलायंस:

42 www.visionias.in ©Vision IAS


o िून्य अपषिष्ट के प्रोत्साहन के षलए अनुसंधान एिं सूचना साझा करने की पहल और सुषिधा है।
o िून्य अपषिष्ट को प्रभािी ढंग से लागू करने की क्षमता का षनमाषर् करता है।
o िून्य अपषिष्ट की उपलषब्ध के मूलयांकन के षलए मानक षनधाषररत करता है।
• ििष 2002 में ज़ीरो िेथट इं टरनेिनल अलायंस के आयोिन के प्राथषमक लक्ष्यों में से एक षिश्व में िून्य अपषिष्ट के षिकास को
षनदेषित करने के षलए मानक षनधाषररत करना था।

Q 94.C
• समताप रे खाएं
o समताप रे खाएं िे कालपषनक रे खाएं हैं िो समान तापमान िाले थथानों को षमलाते हुए खींची िाती हैं। इन्हें खींचने के षलए
सभी थथानों को सागर तल पर मान कर (उं चाई के अन्तर को घटाकर) संिोषधत तापमान प्राप्त ककए िाते हैं , तत्पिात
समताप रे खाएं खीची िाती हैं। इस प्रकार, यह थपष्ट है कक समताप रे खाएं उन थथानों के िाथतषिक तापमान का
प्रषतषनषधत्ि नहीं करती हैं षिनसे होकर ये गुिरती हैं, बषलक ये उन थथानों के समुर्द्तल का तापमान दिाषती हैं। यही कारर्
है कक समताप रे खाओं के मानषचत्र ककसानों के षलए उपयोगी नहीं हैं क्योंकक उन्हें बढ़ती फसलों के षलए षििेि थथानों के
िाथतषिक तापमान की आिश्यकता होती है।
o सामान्यतः समताप रे खाएं पूिष-पषिम में गमन करती हैं और प्राय: अक्षांिों के समानांतर होती हैं। यह प्रिृषत्त तापमान के
क्षैषति षितरर् पर अक्षांिों का प्रबल षनयंत्रर् दिाषती है।
o प्राय: समताप रे खाएं सीधी होती हैं, लेककन िे भूषम और िल के षिभेदी तापन एिं िीतलन के कारर् महाद्वीपों तथा
महासागरों के संयोिन पर मुड िाती हैं।
o उत्तरी गोलाद्धष में महाद्वीपों की षििाल सीमा के कारर् समताप रे खाएं अषधक अषनयषमत हैं, लेककन िे महासागरों के
अषधक होने के कारर् दषक्षर्ी गोलाद्धष में अषधक षनयषमत हैं।
o प्राय: उत्तरी गोलाद्धष में समताप रे खाओं का रे खांतर कम होता है, लेककन दषक्षर्ी गोलाद्धष में िे अषधक षनयषमत होती हैं
और उनका रे खांतर अषधक होता है। इसषलए कथन 1 सही है। िे समताप रे खाएं षिनमें रे खांतर कम होता है, तापमान के
पररितषन की तेि दर और अत्यषधक तापमान प्रिर्ता को दिाषती हैं। दूसरी ओर, िे समताप रे खाएं षिनमें रे खांतर अषधक
होता है, तापमान के पररितषन की षनम्न दर और षनम्न तापमान प्रिर्ता को दिाषती हैं।
o दो समताप रे खाएं, परथपर षनकटता के बाििूद कभी एक-दूसरे को प्रषतच्छेकदत नहीं करती हैं। िैसे कक दो समताप रे खाओं
के चौराहे का अथष होगा कक ककसी षििेि क्षेत्र में एक समय में दो अलग-अलग तापमान होते हैं, िो असंभि है। इसषलए
कथन 2 सही है।
o औसतन, भूषम से महासागरों की ओर प्रिृत्त समताप रे खाओं का झुकाि गमी के दौरान भूमध्यरे खीय और िीतकाल के
दौरान ध्रुिीय रहता है। दूसरी ओर, महासागरों से महाद्वीपों की ओर िाने िाली समताप रे खाओं का झुकाि गर्मषयों के
दौरान ध्रुि की ओर तथा सर्दषयों के दौरान भूमध्यरे खीय रहता है।
Q 95.C
• पयाषिरर्, िन और िलिायु पररितषन मंत्रालय, भारत सरकार और संयुक्त राष्ट्र षिकास कायषक्रम (UNDP) के द्वारा उच्च िेर्ी
के षहमालय पाररषथथषतक तंत्र को सुरषक्षत करने, आिीषिका, संरक्षर् तथा सतत उपयोग की बहाली हेतु इस पररयोिना
(SECURE Himalaya) का िुभारंभ ककया गया। यह ग्लोबल िाइलडलाइफ प्रोग्राम का भाग है, िो िैषश्वक पयाषिरर्ीय
सुषिधा (Global Environment Facility: GEF) द्वारा षित्त पोषित और षिश्व बैंक के नेतृत्ि में सात ििष के एक कायषक्रम का
षहथसा है, षिसे िन्यिीिों में अिैध तथकरी के बढ़ते संकट की प्रषतकक्रया के रूप में षिकषसत ककया गया था। इसषलए कथन 2
सही है।
• "षसक्योर षहमालय" योिना का उद्देश्य उनके आिासों की रक्षा करके षहम तेंदओं
ु को संरषक्षत करना और षहमालय पिषतमाला
की पाररषथथषतकी में सुधार करना तथा पहाडी क्षेत्रों में रहने िाले समुदायों के िीिन में सुधार करना है , िो िलिायु पररितषन
के सिाषषधक पीषडतों में से हैं। इसषलए कथन 1 सही है।
• यह भारत सरकार द्वारा UNDP के साथ षमलकर महत्िपूर्ष पाररषथथषतक तंत्र सेिाओं को बनाए रखने और सामुदाषयक
आिीषिका को सुरषक्षत करके , सुभेद्य षहम तेंदओं
ु के संरक्षर्, प्रितषन को बढ़ाने, सामुदाषयक संथथानों को मिबूत करने तथा
43 www.visionias.in ©Vision IAS
पररदृश्य आधाररत संरक्षर् दृषष्टकोर्ों को बढ़ािा देने हेतु ज्ञान, पक्षपोिर् एिं सूचना प्रर्ाली में सुधार करके कायाषषन्ित ककया
िा रहा है। यह पररयोिना ग्लोबल स्नो लेपडष एंड इकोषसथटम प्रोटेक्िन प्रोग्राम (GLSEP) में सहायता प्रदान करे गी िो भारत
सषहत 12 कायषक्षेत्र देिों में षहम तेंदओं
ु के संरक्षर् का प्रयास है।

Q 96.C
• कथन 1 सही है: षहमाचल और उत्तराखंड षहमालय पषिम में रािी और पूिष में काली (घाघरा की सहायक नदी) के मध्य षथथत
है। यह भारत की दो मुख्य नदी तंत्रो, ससंधु और गंगा द्वारा अपिाषहत है। इस प्रदेि से बहने िाली नकदयों में ससंधु की सहायक
नकदयां रािी, ब्यास और सतलुि एिं गंगा की सहायक नकदयों में यमुना और घाघरा िाषमल हैं। भौषतक षिज्ञान की दृषष्ट से इस
क्षेत्र की दो महत्िपूर्ष थथलाकृ षतयां 'षििाषलक' और 'दून’ हैं। दून पषिमी षहमालय और षहमाचल तथा उत्तराखंड षहमालय के
दषक्षर्ी भाग में षथथत अनुदध्ै यष घारटयां हैं। इस क्षेत्र में षथथत कु छ महत्िपूर्ष दून िममू-दून, पठानकोट-दून, चंडीगढ़-कालका दून,
नालागढ़ दून, देहरादून, हररके दून और कोटा दून िाषमल हैं। इनमें देहरादून सबसे बडी घाटी है, षिसकी लमबाई 35 से
45 ककलोमीटर और चौडाई 22 से 25 ककलोमीटर है।
• कथन 2 सही है: दुआर पूिोत्तर भारत में िलोढ़ बाढ़ के मैदान हैं िो षहमालय की बाहरी तलहटी के दषक्षर् में और ब्रह्मपुत्र नदी
बेषसन के उत्तर में षथथत हैं। यह क्षेत्र लगभग 30 कक.मी. चौडा है और पषिम बंगाल में तीथता नदी से लेकर असम में धनषसरी
नदी तक लगभग 350 कक.मी. तक फै ला हुआ है। दार्िषसलंग और षसकक्कम षहमालय में दुआर थथलाकृ षतयां पाई िाती हैं षिनका
उपयोग चाय के बागान रोपर् के षलए ककया िाता है।

Q 97.B
• देि में पिुधन गर्ना ििष 1919-20 से ही समय-समय पर की िाती रही है। पिुधन गर्ना में सभी पालतू पिुओं और उनकी
कु ल संख्या को किर ककया िाता है। अब तक राज्य सरकारों और कें र्द् िाषसत प्रदेिों के प्रिासन की भागीदारी से इस तरह की
19 गर्नाएं आयोषित की गई हैं। 20िीं पिुधन गर्ना सभी राज्यों और कें र्द् िाषसत प्रदेिों की भागीदारी से आयोषित की
गई। यह मत्थय पालन, पिुपालन और डेयरी मंत्रालय के पिुपालन एिं डेयरी षिभाग की पहल है। इसषलए कथन 1 सही नहीं
है।
• 20िीं पिुधन गर्ना के प्रमुख षनष्किष षनमनषलषखत हैं:
o भारत में कु ल पिुधन आबादी षपछली िनगर्ना की तुलना में 4.6 प्रषतित बढ़कर लगभग 536 षमषलयन हो गई है।
o कु ल पिुधन में षिषभन्न पालतू पिुओं का प्रषतित षनम्नानुसार है: - मिेिी (35.94%), बकरी (27.80%), भैंस
(20.45%), भेड (13.87%) और सूअर (1.69%)। इसषलए, कथन 2 सही है।
o देि में गायों की आबादी षपछले सात ििों में 18 प्रषतित बढ़ी है, िबकक बैलों की संख्या 30 प्रषतित कम हुई है।
o इसके अषतररक्त, देि में पोलिी की आबादी 16.8 प्रषतित की िृषद्ध के साथ बढ़कर 851.81 षमषलयन हो गई, मुख्य रूप
देि में घरों के आंगन में पोलिी की कु ल संख्या 317.07 षमषलयन आंकी गई है िो षपछली गर्ना की तुलना में लगभग 46
प्रषतित अषधक है।
o राज्यों में, उत्तर प्रदेि में सबसे अषधक 67.8 षमषलयन (ििष 2012 में 68.7 षमषलयन) पिुधन हैं, इसके बाद इनकी
सिाषषधक संख्या रािथथान में 56.8 षमषलयन (ििष 2012 में 57.7 षमषलयन), मध्य प्रदेि में 40.6 षमषलयन (ििष 2012
में 36.3 षमषलयन) और पषिम बंगाल में 37.4 षमषलयन (ििष 2012 में 30.3 षमषलयन) है। इसषलए कथन 3 सही नहीं
है।
Q 98.B
• महासागर मुख्य रूप से िीिाश्म ईंधन की खपत से िातािरर् में ग्रीनहाउस गैसों की बढ़ती सांर्द्ता के पररर्ामथिरूप ऊष्मा को
बडी मात्रा में अििोषित करता है। ििष 2013 में इं टरगिनषमेंटल पैनल ऑन क्लाइमेट चेंि (IPCC) द्वारा प्रकाषित पांचिीं
आकलन ररपोटष में इं षगत ककया गया था कक 1970 के दिक से महासागरों ने ग्रीनहाउस गैस उत्सिषन से 93% से अषधक ऊष्मा
को अििोषित कर षलया था। इससे समुर्द् के तापमान में िृषद्ध हो रही है।

44 www.visionias.in ©Vision IAS


• औसत िैषश्वक समुर्द्ी सतह के तापमान (समुर्द् की ऊपरी सतह के कु छ मीटर का तापमान) में षपछले 100 ििों में लगभग
0.13°C प्रषत दिक की दर से िृषद्ध हुई है। महासागरीय तापन से डीऑक्सीिनेिन (समुर्द् में घुषलत ऑक्सीिन की मात्रा में
कमी) होता है और समुर्द् के थतर में िृषद्ध (षिसके पररर्ामथिरूप समुर्द् के िल का तापीय षिथतार होता है और महाद्वीपीय
षहम षपघलता है) होती है। समुर्द् के अमलीकरर् (CO2 में िृषद्ध होने के कारर् समुर्द् के pH की मात्रा में कमी) के साथ बढ़ रहा
तापमान, समुर्द्ी प्रिाषतयों और पाररषथथषतक तंत्र तथा अंततः मानि को महासागर से प्राप्त होने िाले मूलभूत लाभों को
प्रभाषित करता है ।
• समुर्द्ी मछषलयां, समुर्द्ी पक्षी और समुर्द्ी थतनपायी सभी बढ़ते तापमान से बहुत अषधक िोषखमों का सामना करते हैं, षिनमें
उच्च थतर पर षिलोपन, प्रिननक्षम थथलों की क्षषत और अनुकूल पयाषिरर्ीय पररषथथषतयों की ख़ोि के षलए प्रिाषतयों का
सामूषहक संचलन िाषमल है। प्रिाल षभषत्तयााँ भी बढ़ते तापमान से प्रभाषित होती हैं िो प्रिाल षिरं िन का कारर् बनती हैं और
उनकी मृत्यु दर को बढ़ाती हैं।
• इसषलए के िल कथन 1 सही नहीं है।
Q 99.C
• भूमध्यसागरीय प्रकार की िलिायु
o उष्र् ि िुष्क ग्रीष्म ऋतु तथा मृदु एिं ििाषयुक्त िीतकालीन मौसम इस िलिायु की षििेिता है और यह भूमध्य सागर के
चारों ओर तथा उपोष्र् करटबंध से 30° से 45° अक्षांिों के मध्य महाद्वीपों के पषिमी तट के साथ-साथ पाई िाती है।
o ग्रीष्मकाल के दौरान, पछु आ पिनों के प्रभाि से यह बेलट पीछे की ओर थथानांतररत हो िाता है और ििाष धारक व्यापाररक
पिनों के अपतटीय हो िाने की संभािना होती है। िीतकाल के महीनों में, पछु आ पिनें षििुित रे खा की तरफ थथानांतररत
हो िाती हैं और भूमध्यसागरीय क्षेत्र अपतटीय पछु आ पिनों के प्रभाि में आ िाता हैं।
o प्रषतचक्रिात के कारर् आस-पास की हिाएं अपसाररत हो िाती है तथा उनका अिरोहर् होता है षिसके कारर् थिच्छ
आकाि की दिाएं उत्पन्न होती हैं।
o िीतकालीन मौसम में बारी-बारी से गमष धूप िाले कदनों के साथ ििाष होती है षिससे भूमध्यसागरीय िलिायु क्षेत्र में
िलिायिीय पररितषन थपष्ट रूप से दृश्य होते हैं।
o भूमध्यसागरीय भूषम को षिश्व की उद्यान भूषम के रूप में भी िाना िाता है। संतरे , नींबू, षसिॉन और अंगूर िैसे खट्टे फलों
की एक षिथतृत िृंखला की कृ षि की िाती है। फलों के पेडों में , गमी के लमबे सूखे के दौरान काफी गहराई से पानी सोखने
के षलए लंबी िडें होती हैं।
o िार्िषक ििाष समान रूप से नहीं होती है। ििाष एक ही समय या एक षनषित अंतराल में नहीं होती है।
o तापमान भी ििष-दर-ििष पररिर्तषत होता रहता है और सर्दषयों का तापमान िून्य से कम हो सकता है तथा िुष्क क्षेत्रों में
50 षडग्री सेषलसयस से अषधक हो सकता है।
• चीन तुलय िलिायु
o यह उष्र् समिीतोष्र् पूिी सीमांत िलिायु का सबसे षिषिष्ट प्रकार है।
o गर्मषयों में षतब्बत सषहत एषिया महाद्वीप के आंतररक भाग में उच्च ताप के कारर्, षनम्न-दाब प्रर्ाली थथाषपत हो िाती है,
िो उष्र्करटबंधीय प्रिांत िायु धारा को आकर्िषत करती है।
o यह क्षेत्र दषक्षर्-पूिी मानसून से ििाष प्राप्त करता है।
o िीतकाल में, साइबेररया क्षेत्र पर अत्यषधक उच्च दाब का षनमाषर् होता है तथा महाद्वीपीय ध्रुिीय पिनें अत्यषधक िीत और
िुष्क उत्तर-पषिम मानसून के रूप में बाहर की ओर प्रिाषहत होती हैं।
o अलप ििाष होती है लेककन पयाषप्त मात्रा में षहमपात होता है।
o इस क्षेत्र में तीव्र उष्र्करटबंधीय चक्रिात आते हैं षिन्हें टाइफू न कहा िाता है। ये प्रिांत महासागर में उत्पन्न होते हैं और
इनकी बारमबारता ग्रीष्मकाल के उत्तराधष में अषधक होती हैं।
• समिीतोष्र् महाद्वीपीय (थटेपी) िलिायु
o इस क्षेत्र की िलिायु अत्यषधक तापमान के साथ महाद्वीपीय प्रकार की है।
o ग्रीष्मकाल अत्यषधक उष्र् होता है।
o यूरेषिया के महाद्वीपीय मैदानों में सर्दषयााँ अत्यषधक ठं डी होती हैं।
o इसके षिपरीत, दषक्षर्ी गोलाधष में िलिायु का थटेपी प्रकार कभी भी कठोर नहीं होता है।
o तापमान की िार्िषक सीमा में पयाषप्त अंतराल होता है, यह महाद्वीपीय िलिायु का प्रत्यक्ष पररर्ाम है।
o अपनी महाद्वीपीय षथथषत में, थटेपी िलिायु में हलकी िार्िषक ििाष होने की संभािना हो सकती है।
o औसत ििाष लगभग 20 इंच होती है, लेककन इसमें 10 इंच से 30 इं च तक थथानीय रूप से पररितषन होता रहता है।

45 www.visionias.in ©Vision IAS


• उष्र् मरुथथलीय और मध्य अक्षांि रे षगथतानी िलिायु
o उष्र् मरुथथलीय अश्व अक्षांि या उपोष्र् करटबंधीय उच्च िायुदाब पेरटयों के पास षथथत है, िहां िायु का अितलन होता
है, िो ककसी भी प्रकार की ििाष के षलए कम से कम अनुकूल होती है।
o तटीय क्षेत्रों में सापेषक्षक आर्द्षता 60 प्रषतित से कम है, िो रे षगथतान क्षेत्रों से 30 प्रषतित कम है।
o ििाष सामान्य रूप से संिहन प्रकार के प्रचंड झंझािात के रूप में होती है।
o ये रे षगथतान पृथ्िी पर सबसे गमष थथानों में आते हैं और यहााँ पूरे ििष उच्च तापमान बना रहता है।
o उष्र् रे षगथतानों में िीतऋतु नहीं होती है और ग्रीष्म ऋतु का औसत तापमान 86ºF होता है।
o यहााँ िार्िषक ताप पररसर 44ºF है।
o रे षगथतानों में दैषनक तापमान पररसर अषधक होता है।
Q 100.C
• सुिासन सूचकांक (Good Governance Index: GGI) सभी राज्यों के षलए एक समान उपकरर् है िो अषभिासन की
प्रषथथषत और राज्य सरकार तथा कें र्द् िाषसत प्रदेिों द्वारा उठाए गए षिषभन्न कदमों के प्रभाि का आकलन करता है। GGI का
उद्देश्य सभी राज्यों और कें र्द् िाषसत प्रदेिों में िासन की तुलना करने के षलए मात्रात्मक डेटा प्रदान करना है, िो राज्यों और
कें र्द् िाषसत प्रदेिों को सक्षम बनाने और िासन में सुधार के षलए उपयुक्त रर्नीषत तैयार करने में सहायक होगा। इसके
अषतररक्त यह पररर्ाम-उन्मुख दृषष्टकोर्ों और प्रिासन की ओर थथानांतरर् में सहायता करे गा। GGI को सेंटर फॉर गुड गिनेंस
द्वारा षडिाइन और षिकषसत ककया गया है। इसषलए कथन 1 सही है।
• GGI दस क्षेत्रों को ध्यान में रखता है:
o कृ षि और संबद्ध क्षेत्र,
o िाषर्ज्य और उद्योग,
o मानि संसाधन षिकास,
o सािषिषनक थिाथथ्य,
o सािषिषनक अिसंरचना और उपयोषगताएाँ,
o आर्थषक गिनेंस,
o समाि कलयार् और षिकास,
o न्याषयक और सािषिषनक सुरक्षा,
o पयाषिरर् और
o नागररक-के षन्र्द्त अषभिासन।
• इन 10 िासन क्षेत्रों को कु ल 50 संकेतकों के आधार पर मापा िाता है।
• कार्मषक मंत्रालय द्वारा प्रदान ककए गए आंकडों के अनुसार तषमलनाडु को सुिासन सूचकांक (GGI) की समग्र रैं ककं ग में िीिष
थथान षमला है, इसके बाद महाराष्ट्र और कनाषटक का थथान आता है। इसषलए कथन 2 सही है।
• सुिासन के षलए कें र्द्
o इसे अक्टूबर 2001 में आंध्र प्रदेि की तत्कालीन सरकार (Government of Andhra Pradesh: GoAP) द्वारा
अंतराषष्ट्रीय षिकास षिभाग (Department for International Development: DFID) और षिश्व बैंक के सहयोग से
थथाषपत ककया गया था ताकक राज्य के िांसफॉर्मिंग गिनेंस के लक्ष्य को प्राप्त करने में सहायता की िा सके ।
o यह कारष िाई समबन्धी अनुसंधान (एक्िन ररसचष) करता है, पेिेिर सलाह प्रदान करता है, और उनके सुधार एिेंडे के सफल
कायाषन्ियन को सक्षम करने के षलए सरकारी षिभागों एिं एिेंषसयों के षलए पररितषन प्रबंधन कायषक्रम आयोषित करता है।
CGG नीषत-षनमाषताओं िैसे मंषत्रयों, िररष्ठ अषधकाररयों, प्रबंधन षििेिज्ञों, संथथानों और अन्य षहतधारकों षििेिकर
नागररकों के साथ षमलकर लोगों पर के षन्र्द्त गिनेंस प्रथाओं के षनमाषर् के षलए कायष करता है।

Copyright © by Vision IAS


All rights are reserved. No part of this document may be reproduced, stored in a retrieval system or transmitted
in any form or by any means, electronic, mechanical, photocopying, recording or otherwise, without prior
permission of Vision IAS.

46 www.visionias.in ©Vision IAS

You might also like